.
. centre de ressources dilemmes et doutes le visage humain de mathématiques Qui sommes-nous Problème de mois activités de promotion babillard
Centrale des maths - centraledesmaths.uregina.ca
Dilemmes & doutes
« D & D »
. .
topic card  

Sujet:

us

liste de
sujets
. .
nouvelle recherche

855 articles trouvés pour ce sujet.
 
Page
1/1
une écriture plus simple des expressions 2011-01-13
jessie pose la question :

trouver une écriture plus simple des expressions suivantes sachant que : x + y = 3 et x - y =-2
A=x-1+y+2             B=x-1+y-2
C=x+1+y+2            C=x+1+y-2


Claude Tardif lui répond.
soustraction 2010-01-14
LUCAS pose la question :
comment faire l'operation suivant car je ne me souvient plus tres bien
D'avance merci
4h44mn53s-2h52mn45s

Pierre-Louis Gagnon et Claude Tardif lui répond.
Le plus grand commun diviseur 2009-11-11
Katie pose la question :
Question: Pendant mon cours de MAT 1300, mon enseignant m'a posé la question suivante:

(a, b) = 12
(a², b) = 24
(b, 72) = ?

Pouvez-vous m'aider s'il-vous-plaît?

Claude Tardif lui répond.
Un système de plusieurs équations et plusieurs inconnues 2009-01-22
Cédrick pose la question :
J'ai un problème écrit à résoudre est-ce que vous pouvez m'aider ?

Le premier est le tiers de la somme des deux autres.
La somme du premier et du deuxième est 13.
Le produit du deuxième et du troisième est 56.
Réponse: Le premier nombre est___________________
Le deuxième nombre est _________________
Le troisième nombre est _________________

Pierre-Louis Gagnon, Antoine Letarte at Claude Tardif lui répond.
calcul des hh,mn,sec 2008-01-19
gillot pose la question :
bonjour, mon fils à des additions et soustractions d'horaire et je n'arrive pas à trouver la règle pour lui espliquer comment il faut faire il y a t il une formule pour les additions et soustractions merci par avance pour vos réponses
Claude Tardif lui répond.
Illusion géométrique 2007-04-04
Dominique pose la question :
Je n'arrive pas expliquer la modofocation de surface.
Claude Tardif lui répond.
soustractions avec les heures 2006-03-01
Bossedi pose la question :
Je m'appelle Bossedi et malheureusement j'ai un petit souci en ce qui concerne le calcul des heures! Je ne me rappelle plus comment s'effectue une addition ou un soustraction avec les heures comme par exemples 22h30-7h00. Merci infiniment de bien vouloir m'aider.
P.S.: Je me souviens qu'une lectrice du surnom Jennifer avait deja poser ce genre de probleme, mais avec la seule difference que dans son cas, c'etait i j'ai bonne memoire, 09h45 moins 07h52! Quelques choses de ce genre. Dois-je +tot ercire 11h30-07h00?

Claude Tardif lui répond.
quel sont les plus grands diviseurs communs? 2005-09-07
Sylvain pose la question :
quel sont les plus grands diviseurs communs?
Claude Tardif lui répond.
Soustraction avec les heures 2003-04-08
Jennifer pose la question :
Je m'appelle Jennifer et malheureusement j'ai un petit souci. Pour le calcul des heures c'ets à dire je ne me rappelle plus comment s'effectue une addition ou un soustraction avec les heures comme par exemples 9h45-7h52. Si vous pouviez m'aider j'ens erais trés contente.
Diane Hanson and Claude Tardif lui répond.
Comment fait on pour justifier que... 2002-09-30
Lucie pose la question :
Comment fait on pour justifier que p au carré est égal à 2 fois q au carré?
Claude Tardif lui répond.
isomorphisme 2000-08-10
Romain Kroes pose la question :
Pour les beoins d'un ouvrage d'économie que je suis en train de terminer, pouvez-vous me dire qui est (sont) l'inventeur de l' "isomorphisme" en mathématiques (calcul tensoriel)?
Claude Tardif lui répond.
Derivées partielle 1999-10-19
Arnaud Flandin pose la question :
Quel est la definition des derivées partielle
Claude Tardif lui répond.
Velocity and acceleration 2021-01-09
yen pose la question :
The motion of a stone projected upward is given by the relation s=112t - 16t^2. What are the values of velocity and acceleration when t=3 sec?
Penny Nom lui répond.
Two equations with fractions 2020-10-22
mia pose la question :
I am supposed to look for x and y. my teacher didn't explain very well.
please help : ((
2x - y = 5
x/4 + y/3= 2

Penny Nom lui répond.
A confusing problem 2020-10-01
Kgaugelo pose la question :
a triangle has the vertices A(-4;-3),B(x;y)and C(2;1). Determine the coordinates of B if the equation of the line BC is y=5x-9
Harley Weston lui répond.
A problem involving distance, time, and rate. 2020-09-15
Ruby pose la question :
Amy drove on the highways at 70 miles/hour to pick up fruits from her grandma’s garden in the morning.
She stayed there for two hours. When coming back in the afternoon, she drove on the local roads at 40 miles per hour to avoid traffic.
She spent a total of 8 hours for this trip, and drove a total 300 miles.
How many miles did she drive each way?

Penny Nom lui répond.
Simultaneous equations 2020-09-05
Paa pose la question :
a+b=8
a+c=13
b+d=8
c-d=6
find the value of each letter

Penny Nom lui répond.
Filling holes with stone dust 2020-07-30
Zach pose la question :
I’m looking for amount of stone dust needed for thirteen holes with a 3ft depth and 12in diameter with a 4x4 post. I would need the measurement in KG.
Harley Weston lui répond.
A system of equations. 2020-04-27
solomon pose la question :
xy + x =28
x + y +4

Penny Nom lui répond.
Building a house in one year 2020-04-22
Fify pose la question :
If it takes 8 men to build a house in 450 days. How many men would it take to build the house in 365 days? Is there a specific method to calculate this please?
Penny Nom lui répond.
Two chords 2020-04-14
Frank pose la question :
How to find the length of the radius of a circle if you know the lengths of two chords
Penny Nom lui répond.
Form a square and a triangle from a wire 2020-04-08
Raahim pose la question :
2. A 2 meter piece of wire is cut into two pieces and once piece is bent into a square and the other is bent into an equilateral triangle. Where should the wire cut so that the total area enclosed by both is minimum and maximum?
Penny Nom lui répond.
The center and radius of a circle 2020-03-25
Riley pose la question :
so it says find the center and the radius of the below circle
x^2+(y-6)^2=121

i need help learning how to solve this

Penny Nom lui répond.
Sipping juice from a Tetra Pak 2020-02-11
Anjhelic pose la question :
Karen is sipping juice from a 1 in. by 3 in. by 6 in tetra pack at the rate of 0.5in³/sec. How fast is the height of juice in the pack decreasing?
Harley Weston lui répond.
Investigating y = (-2)^x 2020-01-13
Gonzalo pose la question :
This is not precisely a maths question, but it is formulated based on my maths curiosity. Fidgetting with my new graphic calculator, I started graphing things and had the idea to graph $y=(-2)^x.$ The result surprised me, and I thought a little bit about it, stored it on the back of my brain, and promised myself to look deeper into it someday.
Harley Weston lui répond.
Simultaneous equations 2019-10-16
deepak pose la question :
This is too complicated please help me
8/x - 10/y = 1 and x+y=9

Penny Nom lui répond.
What is -5 squared? 2019-09-10
Pori pose la question :
What is -5 squared?
Penny Nom lui répond.
A negative minus a negative 2019-09-03
Maggie pose la question :
Why is a negative minus a negative a negative?
Penny Nom lui répond.
The radius of a circle of given area 2019-08-14
shelby pose la question :
What's the radius of a circle that has an area of 803.84cm2? I also need examples of how you got the answer.
Penny Nom lui répond.
The volume of a frustum 2019-06-24
Abdulganiy pose la question :
A right pyramid on a base 10cm square is 15m high
a)find the volume of the pyramid
b)if the top 6m of the pyramid is removed what is the volume of the remaining frustum?

Penny Nom lui répond.
Maximize monthly revenue 2019-05-23
a student pose la question :
A real-estate firm owns 100 garden type apartments. At RM400 per month, each apartment can be rented. However, for each RM10 per month increase, there will be two vacancies with no possibility of filling them. What rent per apartment will maximize monthly revenue?
Penny Nom lui répond.
Volume and Surface area of a sphere 2019-05-03
Caitlin pose la question :
Why does the surface area formula of a sphere have a squared radius while the volume of a sphere has a cubed radius?
Penny Nom lui répond.
Is every rectangle a rhombus? 2019-04-25
Danny pose la question :
Is a rectangle a rhombus??? True or False
Penny Nom lui répond.
Salary Plus Commission 2019-04-05
Herschel pose la question :
Fire Fighting Equipment pays salespeople as follows: $452 per week plus a commission of .9% on sales between $15,000 and $25000, with 1.1% paid on sales in excess of $25,000. Find the gross earnings for an salesperson whose Total Sales are $28,400. (No commission is paid on the first $15,000 of sales)
Penny Nom lui répond.
Misuse of greater than 2019-03-07
Kenneth pose la question :
I have an old business mathematics textbook. The authors have indicated that the following expressions indicate multiplication:

? is 2/3 greater than 90; ? is 2/3 smaller than 90. They also indicated that the following expression would indicate division: 30 is 2/3 greater than ? and 30 is 2/3 smaller than ?.

How can these phrases indicate multiplication and division? How can 60 be 2/3 greater than 90 and also smaller than 90 as indicated above. What were the authors thinking? I have added the page from the book that indicates what I have explained in my message Kenneth

Harley Weston lui répond.
Simultaneous equations with fractions 2018-12-14
zaheer pose la question :
solve simultaneous equations and give answer in fractional form
3x - 2 = 4y +5/3
y + 7 = 2x + 4
would really appreciate some help on this please

Penny Nom lui répond.
The top of a truncated cone 2018-12-02
Sameer pose la question :
Bottom cone radius 1124.height is 200. Top radius 15degree decree to bottom radius. Top radius how much ?
Penny Nom lui répond.
A locus of points 2018-09-03
Ericka pose la question :
Find the locus of points three times as far from (0,4) as from (2,0)
Penny Nom lui répond.
Gauss' Addition of whole numbers. 2018-04-30
Brad pose la question :
I found this on your site. Question: what is the sum of the first 100 whole numbers?? Is there a different formula if the numbers begin at a number other than one? For example What is the series I want to add is goes from 7 - 53?
Harley Weston lui répond.
The limit of (e^x-1)^(1/x) as x tends to 0 2018-02-27
ARGHA pose la question :
Find the limit of (e^x-1)^(1/x) as x tends to 0.
Penny Nom lui répond.
Dividing a region in half 2018-02-24
mandy pose la question :
There is a line through the origin that divides the region bounded by the parabola y=4x−5x2y=4x−5x2 and the x-axis into two regions with equal area. What is the slope of that line?
Penny Nom lui répond.
A parallelogram, a rhombus and a rectangle 2018-02-07
Sambo pose la question :
what do parallelogram, rhombus and rectangle have in common?
Penny Nom lui répond.
A tangent to a curve 2017-10-22
Jasem pose la question :

Suppose that
f(x)=(3x-3)^1/2.

(A) Find an equation for the tangent line to the graph of f(x) at x=2

(B) Find all values of xx where the tangent line is horizontal, and enter them as a comma-separated list (e.g., 2,-3,6). If there are none, enter none.

Values of x


Penny Nom lui répond.
Salary plus commission 2017-10-16
Natasha pose la question :
Veronika works at Future Shop and earns ₱10.50/h plus 6% commission on sales. Last week Veronika worked 40 hours. What was Veronika’s weekly gross salary if her total sales were ₱2050?
Penny Nom lui répond.
The distance between the origin and a moving point on a graph 2017-10-16
Paulina pose la question :
Find the rate of change of the distance between the origin and a moving point on the graph of y=x^2 +1 if dx/dt=2 centimeters per second
Penny Nom lui répond.
The volume of a cone without calculus 2017-10-02
Akash pose la question :
How to find the volume of a cone without the knowledge of calculus?
Penny Nom lui répond.
Simultaneous equations with fractions 2017-09-09
Farah pose la question :
Hi, my name is Farah. I hope you can help me with this question . X/2 + g/5= 3 and 2g - f = 10
Penny Nom lui répond.
Simultaneous equations 2017-09-02
keto pose la question :
x-y=2,x^2+xy=12
Penny Nom lui répond.
Simultaneous equations with fractions 2017-06-02
Jamal pose la question :
1/x + 1/y =5 and 1/y - 1/x =-1
Penny Nom lui répond.
Forming a cone from a circle 2017-04-15
Tasha pose la question :
A sector of a circle subtends an angle of 216 degrees at the centre, If this sector is used to form a cone of vertical height ,8cm, calculate the base radius of the cone
Penny Nom lui répond.
Is the square root of 2 plus the square root of 2 irrational? 2017-03-29
haya pose la question :
how can i prove the the square root of 2 plus the square root of 2 is an irrational number?
Penny Nom lui répond.
Differentiate y = x^x^x 2017-03-19
Nafis pose la question :
differentiate y = x^x^x
Penny Nom lui répond.
The average rate of change of cot(t) 2017-01-18
Brianna pose la question :
Hello! It's been a while since I've taken a math course, and I'm stuck on a problem in my calculus course.
The question is this:

Find the average rate of change of the function over the given interval.
h(t)=cot(t) a) [5pi/4, 7pi/4]

Penny Nom lui répond.
Five equations 2017-01-16
Muhammed pose la question :
A x 4 = E
B ÷ 4 = E
C + 4 = E
D - 4 = E
___________
& A + B + C + D = 100
what is the value of E

Penny Nom lui répond.
Simultaneous equations with fractions 2016-12-21
Kimi pose la question :
I am stuck on this,can someone please help me????

1/2x+1/3y=11
8x+2/5y=102
Can someone please explain how to solve questions like these?? Or else I will never learn...
Thank you

Penny Nom lui répond.
Two concentric circles 2016-12-21
shrestha pose la question :
Two concentric circles have radii of 14 cm and 7 cm respectively. Find the area of space between them.
Penny Nom lui répond.
The area and radius of a circle 2016-12-19
india pose la question :
How do you find the radius of a circle if only the area is given to you?
Penny Nom lui répond.
Is every rectangle a rhombus? 2016-11-28
Robert pose la question :
not every rectangle is a rhombus.true or false? if false please leave a short explanation. thank you:)
Penny Nom lui répond.
Simultaneous equations 2016-11-25
Rohan pose la question :
x-2/3 + y - 1/4 = 13/12
2-x/2 + 3 + y/3 = 11/6

Penny Nom lui répond.
Radius angle and arc length 2016-11-24
pavidthra pose la question :
Length or arc 11 and angle of subtended 45.need to find a radius
Penny Nom lui répond.
Volume of liquid remaining in a tilted cylinder 2016-11-08
Brian pose la question :
I am trying to determine the amount of a liquid remaining in a 55 gallon drum when it is tilted at 45 degrees and the liquid level is low enough so that the liquid does not completely cover the bottom of the drum.

Your help is greatly appreciated.

Harley Weston lui répond.
A circle formed from bending a wire 2016-10-16
Deszaree pose la question :
The same piece of a wire is bent to form a circle
calculate:
a, the radius of the circle
b, the area of the circle

Penny Nom lui répond.
2,006-1-2-3-4-...-48-49-50 2016-09-24
Mimi pose la question :
Compute the following:

2,006-1-2-3-4-…-48-49-50

Penny Nom lui répond.
The distance around a warehouse 2016-09-21
Chuck pose la question :
How far do I walk when I walk around a 751,000 sq. ft. warehouse?
Penny Nom lui répond.
Integrate (x^2 - 4x + 4) ^4/3 2016-09-15
Ifah pose la question :
Hi i have questions please answer
Integral 2 sampai 3 dari (x² - 4x + 4) ^4/3 dx

Penny Nom lui répond.
The radius of a coffee cup 2016-09-12
Brett pose la question :
What is inside radius, in centimeters of a coffee cup if it holds 350g of coffee when filled to a depth of 9.5 cm? Assume coffee has the same density of water, 1.00g/cm3. A numeric value is expected and not an expression.
Penny Nom lui répond.
A suspension bridge 2016-08-18
sai pose la question :
The main cable of a suspension bridge has the shape of a parabola. The cables are strung from the top of two towers, 61 metres apart, each 15.25 metres high above the roadway. The cable is 1.5 metres above the roadway at the point that is directly between the towers.

h(x)=a(x-30.5)^2+1.5
where a is the constant and its value to be determined.

a) determine the value of the constant a by using that the fact that height of the suspension cable, h(x) is 15.25 metres at each tower. Hence, write the updated model equation.

Penny Nom lui répond.
The modulus of a complex number 2016-07-29
Cheyenne pose la question :
There's a question on my Summer Assignment I cant figure out. Here it is:

Find the absolute Value of the complex number. -5i

Please help?

Penny Nom lui répond.
Two equations with fractions 2016-07-22
kanesh pose la question :
p/2+q/3=3

p/4+2q/3=3

Penny Nom lui répond.
Simultaneous equations 2016-06-25
sena pose la question :
2x/3+3y/2=-1
4x-5y=22

simultaneous equation

Penny Nom lui répond.
Implicit differentiation 2016-06-06
Pranay pose la question :
Is a circle x^2+y^2=2 a function? If it is not a function, why is it possible to do implicit differentiation on it?
Thanks.

Penny Nom lui répond.
The volume of a truncated rectangular pyramidal pond 2016-05-13
Paul pose la question :
How do you calculate a partially filled truncated rectangular pyramid if you always know the bottom rectangle, the maximum height top rectangle perimeter, but have a varying height. Similar to filling up a pond you know the current height and dimensions at the max rectangle how do you calculate it half full i.e. 10x30 outside perimeter with a 2x8 base and a max height of 6ft how do you calculate it at 3ft without re-measuring the top perimeter. Thanks, Paul
Harley Weston lui répond.
A pair of equations with fractions 2016-05-11
tiya pose la question :
hello, i want to know how to solve this question.

m/6+2n/3=6
-m/10=2n/5=2

Penny Nom lui répond.
Covering a site with 6 inches of rock 2016-04-26
Carla pose la question :
if a pump house that measures 20'x16' sits in the middle of a site that is 50'x60' and you apply crushed rock to the site at depth of 6", how many yards will you need to haul?
Penny Nom lui répond.
Filling a hole with 14 tons of rock 2016-04-08
Barry pose la question :
If I had 14 tons of inch and one half sized aggregate rock, how large of a square or rectangular shaped hole would I need to hold that amount?
Penny Nom lui répond.
Maximizing the area of a two lot region 2016-04-03
yousef pose la question :
A man wishes to enclose two separate lots with 300m of fencing. One lot is a square and the other a rectangle whose length is twice its width. Find the dimensions of each lot if the total area is to be a minimum.
Penny Nom lui répond.
Solve for x and y 2016-02-27
ntshidi pose la question :
Y=1/2x+4and1/4x-6
Penny Nom lui répond.
A bonus of 8% of sales 2016-02-22
Barghavi pose la question :
A man earns $325 per week plus an additional 8% on any sales over $500. If he makes $6,250 in sales, how much money in total does he earn that week? thank you!
Penny Nom lui répond.
Two equations in two unknowns 2016-02-13
Anumba pose la question :
4x + 2y = 4
7x - y = -11

Penny Nom lui répond.
A chord of a circle, the central angle and the radius 2016-01-26
Nishan pose la question :
If chord length is given along with angle then how to calculate the radius.
Penny Nom lui répond.
A system of linear equations 2016-01-24
kareem pose la question :
my name is kareem
and i am a student i have a math puzzle and i tried to solved it but it always have same mistake
x-y=9
x+z=12
z-n=14
y+n=2

Penny Nom lui répond.
A Max/Min problem with an unknown constant 2016-01-17
Guido pose la question :
Question:

The deflection D of a particular beam of length L is

D = 2x^4 - 5Lx^3 + 3L^2x^2

where x is the distance from one end of the beam. Find the value of x that yields the maximum deflection.

Penny Nom lui répond.
Integration of dx/(x^2+1)^3 2016-01-07
Ishank pose la question :
Integration of dx/(x^2+1)^3
Penny Nom lui répond.
A relative maximum and a relative minimum 2015-12-28
kemelo pose la question :
show for the following function f(x)=x+1/x has its min value greater than its max value
Penny Nom lui répond.
Constructing simultaneous linear equations 2015-12-28
Deborah pose la question :
Linda thinks of a two-digit number. The sum of the digits is 8. If she reverses the digits, the new number is 36 greater than her original number. What was Linda's original number?

Thank you!

Penny Nom lui répond.
A bus trip 2015-12-15
Michelle pose la question :
I do not know what equation(s) should be used in order to obtain the answer(s) to this word problem.

A bus was rented for a trip, the cost was dived equally amongst the passengers. During the trip, someone mentioned that if there were 9 more passengers, they would have paid each 5$ less. Another mentioned that if there were 6 less passengers, they would have each paid 5$ more. What is the cost of renting the bus and how much must each passenger pay.

Penny Nom lui répond.
Linear equations in two variables 2015-12-13
priya pose la question :
I have problem in solving these equations please help me today itself very urgent:
I)2x+y=y
II)pie*x+y=9

Penny Nom lui répond.
A tangent line to a parabola 2015-12-02
pei pose la question :
Given that the line y=mx-5 is a tangent to the curve y=2x^2+3 find the positive value of M.
Penny Nom lui répond.
The diagonal of a rhombus 2015-11-14
Om pose la question :
In a rhombus ABCD, angle A=60° and side AB=6 cm. Then diagonal BD is ?
Penny Nom lui répond.
2.236... 2015-10-13
Ann pose la question :
2.236...
What is the most specific category of numbers does this fall into? Rational or Irrational? Does the .... mean that it repeats?

Harley Weston lui répond.
Why is the area of square not conserved when it changes to a rhombus? 2015-06-28
shubham pose la question :
Why is the area of square not conserved when it changes to a rhombus, both have equal sides still rhombus have less area than square.??
Penny Nom lui répond.
1 + 1 + 1 + 1 + 1 + 1 + 1 + 1 + 1 + 1 + 1 + 1 x 0 + 1 = ? 2015-06-18
Sharon pose la question :
1 + 1 + 1 + 1 + 1 + 1 + 1 + 1 + 1 + 1 + 1 + 1 x 0 + 1 = ?

I got 1 as my answer despite BODMAS making it 12 because logic tells me I ought to place brackets around the first set of repeated addition. Could you please clarify this for me? Thank you 😊

Harley Weston lui répond.
A tangent to y = x^3 2015-05-31
Brayden pose la question :
Show that a tangent line drawn to the curve y=x^3 at the point (d,f (d)), where d>0, forms a right triangle with the x and y axes in quadrant 4 whose area is (2/3)d^4.
Penny Nom lui répond.
Two lorries approaching an intersection 2015-05-15
Nuraini pose la question :
Two straight roads intersect at the right angles. Lorry A, moving on one of the roads, Approaches the intersection at 50mi/h and lorry B, moving on the other roads, approaches the intersection at 20mi/h. At what rate is the distance between the lorry changing when A is 0.4 mile from the intersection and B is 0.3 mile from the intersection?
Penny Nom lui répond.
A calculus optimization problem 2015-05-14
Ali pose la question :
Given an elliptical piece of cardboard defined by (x^2)/4 + (y^2)/4 = 1. How much of the cardboard is wasted after the largest rectangle (that can be inscribed inside the ellipse) is cut out?
Robert Dawson lui répond.
The number of possible musical notes using an n-key instrument 2015-05-04
Farihin pose la question :
Lets say that i have keys, and each key is for notes of a musical instrument, So i wanted to find out the number of notes i can get for a certain number keys, of course in the form of an equation. Notes can use as many keys, it can use 1, or 2, or 3, or even 100.
Notes in real life is not as such, but ignore reality. I tried doing this but i can't seem to find a formula for it. For example, i have 4 keys, say A, B, C, and D. so, for notes that uses one key are 4, which is A, B, C, and D themselves. for notes that uses two keys are 6,
AB, AC, AD, BC, BD and CD.
for notes that uses three keys are 4,
ABC, ABD, ACD and BCD.
lastly for notes that uses all four keys is 1, ABCD.
So, the total will be 4+6+4+1=15#

The nth term for the first equation is n, the second is [(n^2)-n]/2 the third and the fourth, i don't know but the final answer should be like,
n + [(n^2)-n]/2 + [3rd] + [4th]

Sorry for the long question though...

Penny Nom lui répond.
The method of elimination 2015-05-01
oreanna pose la question :

Question from oreanna, a student:

How do u solve 2x+9y=3

7x-4y=-25 in elimination


Penny Nom lui répond.
The volume of a sphere 2015-04-30
Cassidy pose la question :
How do you find the radius of a sphere that has volume 36pI?
Penny Nom lui répond.
Constructing a box of maximum volume 2015-04-14
Margot pose la question :
I need to do a PA for maths and I'm a bit stuck. The PA is about folding a box with a volume that is as big as possible. The first few questions where really easy but then this one came up.

8. Prove by differentiating that the formula at 7 does indeed give you the maximum volume for each value of z.

Penny Nom lui répond.
A word problem with fractions 2015-04-09
Lorraine pose la question :
If the numerator of a certain fraction is doubled and the denominator is increased by 1, the fraction becomes 1/2.

If the numerator of the original faction is squared and the denominator is decreased by 2, the fraction becomes equal to 1.

Let x be the numerator and let y be the denominator of the original fraction. Write down two simultaneous equation in x and y.

Solve these equations to find two possible values for the given fraction.

Penny Nom lui répond.
The area of the ring between two concentric circles 2015-04-08
Conner pose la question :
The area of the ring between two concentric circles is 25pi/2 square inches. The length of a chord of the larger circle tangent to the smaller circle is?
Robert Dawson lui répond.
Is a rhombus a square? 2015-03-24
Justin pose la question :
Is a rhombus a square?
Penny Nom lui répond.
Extraneous solutions 2015-03-07
Emily pose la question :
I have a question about Extraneous Solutions, Because I was recently researching to figure out on how to determine that a solution is extraneous and many of the answers talked about how if a answer is negative that it should always be extraneous but now I found out that positive solutions can also be considered as extraneous solutions so that is why I am really confused Extraneous Solution's.
I would really appreciate it if you could clearly explain to me how to determine an extraneous solution from a normal solution.
Thanks,
Emily

Chris Fisher lui répond.
The radius of a cylinder 2015-02-26
Rose pose la question :
Hi. I want to know how to find the radius of cylinder.
When I have height (35cm) and area of the curved surface(880sq.cm).
I know the formula is 2πr(h+r). But I can't understand how to find it's radius.
Please help me. I really need your help.

Penny Nom lui répond.
Two equations with fractions 2015-02-26
Pulane pose la question :
Hi math centre I've been trying to solve these equations for days now please help (6/x)-(1/y)=4
(9/x)+1=(-2/y)

Please help me solve them simultaneously
Thank you

Penny Nom lui répond.
f(x)=(x^2-1)/(x-1) 2015-02-21
Ahmed pose la question :
Is f(x)=[(x^2-1)/(x-1) and x=2 at x=1] differentiable at x=1 ? Why ?
Penny Nom lui répond.
Two equations 2015-02-16
nigel pose la question :
2x+1/2y=1
6x-3/2y=21

Penny Nom lui répond.
The center and radius of a circle 2015-02-06
ariana pose la question :
I need to find the center and the radius of this circle. I don't know how to put 9/2 than square it into a fraction.

x^2 + 2x + y^2 +9y=5

Penny Nom lui répond.
128/(-16)/(-2) 2015-01-28
jackie pose la question :
128/(-16)/(-2) I was wondering if you can show me how to work this question out
Harley Weston lui répond.
Is a square a trapezoid? 2015-01-26
Katie pose la question :
Can a trapezoid sometimes be a square?
Penny Nom lui répond.
Rates, percentages and units 2014-12-30
Kenneth pose la question :
Hello:

If percentages have no units, why are some percentages called rates, as in interest rate, or perhaps a tax rate of 7% as an example? A rate has units of different quantities.

I thank you for your reply.

Robert Dawson lui répond.
4 card hands 2014-11-02
Ronaldo pose la question :
How many Mus hands (combination of 4 cards allowing repetitions) can be made from a deck containing 8 types of cards?

RRRR, RRRC, RRRS, RRR7, RRR6, RRR5, RRR4, RRRA,
RRCC, RRCS, RRC7, RRC^, RRC5, RRC4, RRCA,
RRSS, RRS7, etc.

I think 330. If so, what is the formula?

Penny Nom lui répond.
A linear system 2014-10-14
pheter pose la question :
4/x - 1/y = 3 .... equation (1)
6/x - 2/y = 5 .... equation (2)

Penny Nom lui répond.
How does pir^2 = 1/4pid^2? 2014-10-14
al pose la question :
Hi I cant work out the algebra. How does pir^2 = 1/4pid^2 Thanx
Penny Nom lui répond.
Two equations in x and y 2014-09-25
seyilogo pose la question :
solve y=2x - 3 and (4x - 2y) / x + y = 1 simultaneously
Harley Weston lui répond.
Continuity on a closed interval 2014-09-21
Pragya pose la question :
The trouble I'm having is as follows :
a continuous function is most of the times defined on a closed interval, but how is it possible to define it on a closed interval ,because to be continuous at endpoints of the interval the function's limit must exist at that endpoint,for which it has to be defined in its neighborhood,but we don't know anything about whether the function is always defined in the neighborhood.
Please help...

Penny Nom lui répond.
Two equations in two variables 2014-09-18
Susan pose la question :
(28x + 36y) - [20000 - .75(28x + 36y) + 60000] = 5000
x + y = 10000
solving two equations involving variables

Penny Nom lui répond.
A tangent to a curve passing through a point not on the graph 2014-09-15
Aquilah pose la question :
For the curve y = x2 + 3x, find the equations of all tangent lines for this graph that also go through the point (3, 14).
Penny Nom lui répond.
Water usage in Ames 2014-08-29
Abigail pose la question :
The City of Ames water treatment plant treats 7 million gallons of water per day on average to meet water demands. Hypothetically, if water is withdrawn from a groundwater system with an area of 51.6 square miles (approximate area of Ames and surrounding area), what is the depth of water in inches that would have to be added to the groundwater system annually to replace the water that is withdrawn by the city?
Penny Nom lui répond.
A car passing a bus 2014-08-24
Athish pose la question :
the driver of the car is travelling at 36 km/hr and spots a bus 80 m ahead of him after 1 hr the bus is 120 m behind the car what is the speed of the bus
Penny Nom lui répond.
Filling three holes with stones 2014-08-20
mark pose la question :
how many tonnes of hardcore/crushed stone would it take to fill 1 hole 9ft diameter 5ft deep and 2 holes both 3ft diameter and 5ft deep
Penny Nom lui répond.
The sum of the first 50 terms of an arithmetic progression 2014-07-26
Joshua pose la question :
Hello ...my is Joshua...I'm a grade 11 student...I got a question

Calculate the sum of the first 50 terms of an arithmetic progression: 112:98:84

Penny Nom lui répond.
The method of elimination 2014-07-05
leo pose la question :
please explain how can i solve this problem

3x-6y=-38
6x-9y=44

using elimination and simultaneous method thank you :)

Penny Nom lui répond.
Differentiate ln[x(2x-4)^1/2] 2014-06-28
Igwe pose la question :
If y=In[x(2x-4)^1/2],find dy/dx at x=3
Penny Nom lui répond.
Simultaneous equations 2014-06-20
rana pose la question :
solve the simultaneous equations
a)3x=7y
12y=5x-1

Penny Nom lui répond.
The derivative of sin(x) 2014-04-26
Lucky pose la question :
f(x)=Sin(x), by first principle its f'(x)...show me how to solve such problem.
Penny Nom lui répond.
Simultaneous equations with fractions 2014-04-19
Maryam pose la question :
I looked at your example of simultaneous equations with fractions and applied it to my question from an educate exam papers but I couldn't get it to work. The question is:

x/8 - y = -5/2
3x + y/3 = 13

Penny Nom lui répond.
A frustum of a pyramid with a square base 2014-04-18
tuba pose la question :
a pyramid has a base of 10 m and is 15 m high.what is the volume? if 6m is removed from top what is the volume of the remaining frustum?
Penny Nom lui répond.
The area bounded by the X-axis and y=x^(2)-4 from -5 to 0 2014-04-15
Lexie pose la question :
Determine the area that is bounded by the following curve and the x-axis on the interval below. (Round your answer to three decimal places)

y=x^(2)-4, -5 ≤ x ≤ 0

The answer is 32.333 but I have no idea how to get there.

Penny Nom lui répond.
A tangent of the curve (x/a)^n+(y/b)^n =2 2014-04-15
sudhir pose la question :
the equation of tangent of the curve (x/a)^n+(y/b)^n =2. at(a,b) is
Penny Nom lui répond.
The locus of a point 2014-04-04
srishti pose la question :
A point P moves such that the difference between its distance from the origin and from the axis of x is always a constant c . what is the locus of the point?
Penny Nom lui répond.
A cable around the Earth 2014-03-13
fikile pose la question :
By how much must an equatorial cable be extended in order that it runs 1meter above the ground?
Penny Nom lui répond.
A parabolic suspension bridge 2014-03-11
jeffrey pose la question :
the towers of a parabolic suspension bridges 200 meter long are 40 meter high and the lowest point of the cable is 10 meter above the roadway.Find the vertical distance from the roadway to the cable at 50 meter from the center.
Penny Nom lui répond.
What are the possible lengths of the hypotenuse? 2014-03-07
audrey pose la question :
The three sides of a right angles triangle measure x-2, x+5, and 2x-1 in length. What are the possible lengths of the hypotenuse?

... I'm doing the equation c2=a2+b2 and subbing in the numbers but nothing makes sense

Penny Nom lui répond.
Simultaneous fractional equations 2014-02-15
benjamin pose la question :
hi math central. benjamin here. during class, i had problem with this topic. normally i wont have problems with math but this topic i just too hard for me. please help i am having exam and test next week on this topic

here is the question:
using substitution method, solve the simultaneous equation.
(x+1)/(y+2)=0.5

(x-2)/(y-1)=1/3

Penny Nom lui répond.
The volume of a frustum 2014-02-02
mike pose la question :
volume of frustum R23", r 18", h 16"
Penny Nom lui répond.
Conics 2014-02-01
Kassidy pose la question :
Hey, I have searched through all the questions about conics and how people use them in the real world, but none of them were very specific on how they are applied and the process, why it's so important etc. I have a project due asking these questions and it's been very difficult finding the right answer, if you could name jobs, how they are use and specifically applied that would be greatly appreciated.
Penny Nom lui répond.
Two nonlinear equations 2014-01-26
Naryn pose la question :
(1÷x) + (1÷y) = (7÷12)
xy = 12

Penny Nom lui répond.
An inequality 2014-01-25
LANELL pose la question :
this is a problem to solve: 1/3 + 2/7 >=x/21 -- part of the answer is (-oo) not exactly that similar--it is on a calculator as a symbol- sure you know what it is I am talking about- the x will be a number
Penny Nom lui répond.
25% profit 2014-01-02
Finn pose la question :
Hello,
The question is all about buy-and-sell business.
Problem:
Pencil - $6 for whole sale price
     $8 if I sell the item How do I get the 25% profit? (you can change the whole sale price and the retail price[if i sell the item])
if I buy the pencil at 24 pieces and sell it at 24 pieces.

Penny Nom lui répond.
The popcorn box problem 2013-11-07
Dave pose la question :
We know that calculus can be used to maximise the volume of the tray created when cutting squares from 4-corners of a sheet of card and then folding up.

What I want is to find the sizes of card that lead to integer solutions for the size of the cut-out, the paper size must also be integer. EG 14,32 cutout 3 maximises volume as does 13,48 cutout 3.

I have done this in Excel but would like a general solution and one that does not involve multiples of the first occurence, as 16, 10 cutout 2 is a multiple of 8,5 cutout 1.

Walter Whiteley lui répond.
Substitution type simultaneous equations 2013-11-03
Kayla pose la question :
I am having problems with substitution type simultaneous equations, when the variable you are substituting is a algebraic one:
y=x^2-3x+4 and 3x-2y=1
I have rearranged 3x-2y=1 to get x=(1+2y)/3 but when I substitute this x value into the other equation, I get the wrong answers!
Would appreciate any help! Thank you.

Penny Nom lui répond.
Water flowing out of a tank 2013-11-03
Carolyn pose la question :
The flow of water out of a hole in a tank is known to be proportional to the square root of the height of water above the hole. That is,

dV/dt (proportional to) sq root (h)

The tank has a constant cross-sectional area A, show that the height of water in the tank is given by

h = ((-kt+C)/2)^2

If the tank is 9 metres high, and it takes 5 hours for it to drain from full to half full, how much longer will we have to wait until it is completely empty?

Penny Nom lui répond.
Extraneous solutions 2013-10-22
tom pose la question :
i need an equation where x=-2 is the correct answer and x=-3 is an extraneous solution. can you provide me with such an equation??
Harley Weston lui répond.
A frustum 2013-10-12
Lily pose la question :
A cone of height 6in. and radius of base 4in. has its top cut off by a plane parallel to its base and 4in from it. Find the volume of the remaining frustum.

I have worked out the volume of the entire cone but I don't know how to work out the radius of the top of the frustum.

Thanks

Penny Nom lui répond.
Proportional rates 2013-10-10
Varsha pose la question :
A province's Ministry of Social services has found that both the number of people needing social assistance and the province's total expenditures on social assistance are proportional to the rate of unemployment. Last August when the provincial unemployment rate was 8.4 %, the province provided assistance to 89,300 individuals at a total cost of 4107.4 million. The forecast unemployment rate for next August is 7.9%. How many people can the province expect to need social assistance next August? What amount should the province budget for social assistance in August?
Penny Nom lui répond.
Maximize the volume of a cone 2013-10-09
Conlan pose la question :
Hi I am dong calculus at school and I'm stumped by this question:

A cone has a slant length of 30cm. Calculate the height, h, of the cone if the volume is to be a maximum.

If anyone can help me it would be greatly appreciated.

thanks.

Penny Nom lui répond.
The sum of all whole numbers from 1 to X 2013-09-06
Tim pose la question :
How do I develop a rule for the sum of all whole numbers from 1 to X when I have no idea how to do this
Penny Nom lui répond.
Ordering crushed stone 2013-09-03
Prakash pose la question :
Dear Sirs, I am working in a Soft Landscaping contracting company. If I need to purchase crushed stone with the size 50-70mm for $53,429 m^2$ area, how many 20feet containers should I need to order to my suppliers? The 20foot container has internal dimensions 5,897 mm by 2,348 mm by 2,285 mm and the $53,429 m^2$ area is to be covered by 10 cm of stone.
Harley Weston lui répond.
Equal ordinate and abscissa 2013-08-15
sonit pose la question :
the slope of tangent to the curve y=(4-x^2)^1/2 at the point, where the ordinate and abscissa are equal, is
Penny Nom lui répond.
Differentiate x^x - 2^sinx 2013-08-09
tarun pose la question :
derivative of x^x - 2^sinx
Penny Nom lui répond.
Practical uses of trigonometry 2013-08-06
tharindu pose la question :
use of trigonometry
Penny Nom lui répond.
What is the value of |2((i)^(1/2))|? 2013-07-22
Delilah pose la question :
What is the value of |2((i)^(1/2))| ?
i.e. absolute value of 2 multiplied by square root of i.

Penny Nom lui répond.
Simultaneous equations 2013-07-10
Warren pose la question :
solve this simultaneous equation:
xy=4
2x+3y=14

Penny Nom lui répond.
Water use in a rectangular flush tank 2013-05-10
milo pose la question :
A rectangular flush tank 22" by 71/4 contains water to depth of 17" how many gallons of water will be saved if a conservation device reduces the capacity to 3/5 of this amount? And reduced to the nearest tenth
Penny Nom lui répond.
A cyclic rhombus 2013-04-16
Marisa pose la question :
I know that the only rhombus that can be inscribed in a circle is a square, but why is that? I've been racking my brain and the internet for solutions, but have found no logical explainations in relation to the arc degrees and angles. Please help.
Chris Fisher lui répond.
4 linear equations with 3 unknowns 2013-04-12
Marian pose la question :
how to solve for 3 unknowns in 4 simultaneous equations
Penny Nom lui répond.
Simultaneous equations with fractions 2013-03-31
Terence pose la question :
5/x-6/y=1 17/x+30/y=16 I been spending whole day to solve this question. Would be very grateful if you can help I try The denominator value is a equations term which make is simultaneous equations so hard.
Penny Nom lui répond.
Tangents to the curve y = x^3 2013-03-24
Ethan pose la question :
How many tangent lines to the curve y = x^33 pass through the point (2, 4)? For each such line, and the exact coordinates of the point of tangency on the curve.
Penny Nom lui répond.
Extraneous solutions 2013-02-18
Eileen pose la question :
(5x+4)^1/2-3x=0
Penny Nom lui répond.
Related rates 2013-02-17
Ishaak pose la question :
A hemispherical bowl is filled with water at a uniform rate. When the height of water is h cm the volume is π(rh^2-1/3 h^3 )cm^3, where r s the radius. Find the rate at which the water level is rising when it is half way to the top, given that r = 6 and the bowl fills in 1 minute.
Penny Nom lui répond.
The continuity of f(x,y)=ln(x^2+y^2) 2013-02-17
anu pose la question :
the question says we have to find the points in the plane where the function is continuous: f(x,y)=ln(x^2+y^2) . here we aren't given a particular point (x,y) where we have to check a function's continuity. what is to be done if we have to check continuity over the whole domain of the function? please help .
Harley Weston lui répond.
A word problem involving toys 2013-02-14
sandy pose la question :
Each boy gets 5 toys.Each girl gets 3 toys.There are 150 pupils.
The boys had 74 more toys than girls.
How many boys?
How many girls?

Penny Nom lui répond.
Simultaneous equations 2013-02-10
Michael pose la question :
2P + 1/3V =8
3P - 2/V=5

Penny Nom lui répond.
Integration from 0 to 2pi of 1/(3cos x + 2) dx 2013-02-04
ankit pose la question :
Integration from 0 to 2pi of 1/(3cos x + 2) dx
Harley Weston lui répond.
Maximize profit 2013-01-19
Chris pose la question :
A firm has the following total revenue and total cost function.
TR=100x-2x^2
TC=1/3x^3-5x^2+30x
Where x=output
Find the output level to minimize profit and the level of profit achieved at this output.

Penny Nom lui répond.
5 1/2 cubic feet of sawdust 2013-01-19
Vina pose la question :
I have a bale of sawdust that containes 5 1/2 cubic feet. How many will I need to make a cubic yard?
Penny Nom lui répond.
A triangular island 2012-12-29
Udit pose la question :
A long time ago Mr Gibson found an island shaped as a triangle with three straight shores of length 3 km,4 km and 5 km. He declared an 'exclusion zone' around his island and forbade anyone to come within 1 km of his shore. What was the area of his exclusion zone?
Penny Nom lui répond.
An integral 2012-12-16
Slavena pose la question :
integration of (lnx)^2 / x dx
Penny Nom lui répond.
An area bounded by lines 2012-12-16
sidra pose la question :
find area bounded by functions:
y=x
y=2x
and y=5-x

Penny Nom lui répond.
A max/min problem 2012-12-14
bailey pose la question :
A right angled triangle OPQ is drawn as shown where O is at (0,0). P is a point on the parabola y = ax – x^2 and Q is on the x-axis.

Show that the maximum possible area for the triangle OPQ is (2a^3)/(27)

Penny Nom lui répond.
The derivative of y = sin (30º + x) 2012-11-07
Saskia pose la question :
derivative of y = sin (30º + x)
Harley Weston lui répond.
An implicit differentiation problem 2012-10-26
Katie pose la question :
find y' of x^2y-2y^3=3x+2y
Harley Weston lui répond.
How fast is the distance between the aircraft and the car increasing? 2012-10-24
Steven pose la question :
At a certain instant an aircraft flying due east at 240 miles per hour passes directly over a car traveling due southeast at 60 miles per hour on a straight, level road. If the aircraft is flying at an altitude of .5mile, how fast is the distance between the aircraft and the car increasing 36 seconds after the aircraft passes directly over the car?
Penny Nom lui répond.
A label to cover a plastic cup 2012-10-23
Kevin pose la question :
I'm trying to make a label to cover the entire outer area or a plastic cup. I know there must be a way to figure out the dimensions needed, but I can't seem to figure it out. The circumference of the bottom of the cup is 21.4cm and the circumference at the top of the cup is 29.8cm. The cup is 14.5cm tall. What should the height of the arc from the plane connecting the two ends of the 21.4cm arc. I attached a diagram where x is the value I'm looking for. I'm guessing there is some simple relationship between the length of a line and the arc needed to turn that line into a perfect circle, but I don't know what it is. Can you figure this out and share it with me? Thanks.

-Kevin

Penny Nom lui répond.
Differentiation rules 2012-10-23
Morgan pose la question :
Use the derivative rules to differentiate each of the following:
1. f(x)=1/x-1
2. f(x)= sqrt(x)

Penny Nom lui répond.
A word problem involving a fraction 2012-10-12
Derrick pose la question :
If the numerator and denominator of a fraction are both decreased by 1 the fraction becomes 2/3. If the numerator and denominator are both increased by 1 the fraction will be 3/4. Find the original fraction. How to do?
Penny Nom lui répond.
The hypotenuse 2012-09-06
Jeevan pose la question :
how can i find the height and base of a right angle triangle if i have the hypotenuse only ?
Penny Nom lui répond.
A tangent to f(x) = 1/x 2012-09-04
Steven pose la question :
Consider the graph of the function f(x) = 1/x in the first quadrant, and a line tangent to f at a point P where x = k. Find the slop of the line tangent to f at x = k in terms of k and write an equation for the tangent line l in terms of k.
Penny Nom lui répond.
The length and depth of a chord 2012-08-16
Tim pose la question :
Can I find the radius of a circle, if I have a chord of 2400mm, and a depth between the center of the chord and the perimeter of the circle of 150mm ?
Thanks
Tim

Penny Nom lui répond.
Fence post holes 2012-07-19
Gerry pose la question :
Hello, I'm digging 30 8" dia holes, 5 ft deep for fence posts that are 4"x 4" Can you please help me figure out how much stone dust I should order for all 30 holes. Thanks
Penny Nom lui répond.
A volume of revolution 2012-07-15
Tewodros pose la question :
Let f(x) = e^x and g(x) = x^1/2 both be defined on [0,1]. Consider the region bounded by f(x), g(x), x = 0, x = 1. Rotate this region about the y-axis and determine the volume using the shell method.
Harley Weston lui répond.
Two cars approach a right-angled intersection 2012-04-10
Michael pose la question :
Two cars approach a right-angled intersection, one traveling south a 40km/h and the other west at 70km/h. When the faster car is 4km from the intersection and the other case if 3km from the intersection, how fast is the distance between the car cars changing?
Penny Nom lui répond.
A maximization problem 2012-04-09
Nancy pose la question :
After an injection, the concentration of drug in a muscle varies according to a function of time, f(t). Suppose that t is measured in hours and f(t)=e^-0.02t - e^-0.42t. Determine the time when the maximum concentration of drug occurs.
Penny Nom lui répond.
The spread of a rumor 2012-04-09
Roohi pose la question :
The function f(t) = a/(1+3e^(-bt)) has also been used to model the spread of a rumor. Suppose that a= 70 and b=3 0.2. Compute f(2), the percentage of the population that has heard the rumor after 2 hours. Compute f'(2) and describe what it represents. Compute lim t approaches infinity and describe what it represents.
Penny Nom lui répond.
The period T of a pendulum 2012-03-27
Ashley pose la question :
The period T of a pendulum is given in terms of its length, l, by T=2pi sqrt(l/g) where g is the acceleration due to gravity(a constant)
a. find dT/dl
b. what is the sign of dT/dl
c. what does the sign of dT/dl tell you about the period of the pendulums?

Penny Nom lui répond.
The derivative of x^-(1/2) 2012-01-14
Eric pose la question :
I have an problem figuring out the derivative of the negative square root of x i.e. x^-(1/2) using the first principle.
Could someone please show me?
Thanks in advance!

Harley Weston lui répond.
Lost in the woods 2012-01-12
Liz pose la question :
I am lost in the woods. I believe that I am in the woods 3 miles from a straight road. My car is located 6 miles down the road. I can walk 2miles/hour in the woods and 4 miles/hour along the road. To minimize the time needed to walk to my car, what point on the road should i walk to?
Harley Weston lui répond.
The radius of a circle 2012-01-12
Janie pose la question :
Find the radius of a circle knowing that a chord of 24.6 inches has a corresponding arc of 70°.
Penny Nom lui répond.
A volume of revolution 2012-01-11
john pose la question :
find volume of solid generated by revolving the region in the first quadrant bounded by the curve y squared=x cubed, the line x=4 and the x-axis about the line y=8. The answer in the back of the book is 704 pi divided by5
Penny Nom lui répond.
A wire spiral 2012-01-07
Pinar pose la question :
I am trying to help my daughter with very challenging maths questions which sometimes I get stuck If someone help me with one of them I would appreciate it.

Jane is making a spiral out of wire. She bends the wire after 1 cm, then bends the wire after 2cm, then 3cm and continues in this manner. After 4 bends she used 15 cm of wire For each bend how many cm wire was used? After 6 bends how many cm wire will she have used? How many bends will she have made if she uses 66 cm of wire?

I would appreciate if somebody would help me with this.

Thank you!
Pinar

Penny Nom lui répond.
The volume of a frustum of a cone 2011-12-24
CV pose la question :
If I know the height, taper, and volume of a frustum cone, what are its Radii?

Calculating frustum cone volume is straight forward.
v=Pi/3*h*(R*R+r*r+R*r) where;
v=volume;
h=frustum height;
R=major radius;
r=minor radius;
Pi=3.1415926;
t=taper, ('slant angle' where t=0 is a disk)

Here is the tricky part.
Knowing 'h', 't', & 'v';
Calculate one or both of 'R' & 'r'.

Specifically, what is the formula for 'R=' or 'r='?

Penny Nom lui répond.
Water is flowing into a cup 2011-12-19
Tim pose la question :
A cup has a radius of 2" at the bottom and 6" on the top. It is 10" high. 4 Minutes ago, water started pouring at 10 cubic " per minute. How fast was the water level rising 4 minutes ago? How fast is the water level rising now? What will the rate be when the glass is full?
Penny Nom lui répond.
The circumference and area of a circle 2011-12-13
Mable pose la question :
A circle that going 70mi across using 22/7 I need the area,radius, and the circumference and how to set up the steps can you help?
Penny Nom lui répond.
A cube of ice is melting 2011-12-05
Emily pose la question :
a cube of ice (i.e.) each side is of the same length) is melting at a rate such that the length of each side is decreasing at a rate of 5cm per hour. how fast is the volume of the cube decreasing (in cubic cm per hour) at the instant the length of each side is 25cm?
Penny Nom lui répond.
A suspension bridge 2011-11-30
jennifer pose la question :
suspension bridges like the golden gate bridge, are used to span large distances. when the main curved cables are attached to the deck by vertical cables they will end up in the shape of parabola. assume that we need to build a bridge that spans 2,400 feet. the two towers 165feet tall each where placed at 400feet from either side. the lowest point of the center of the bridge at 10feet. vertical suspension cables where placed at 25foot interval. how many feet of cable are needed to connect the deck to the main cables between the two towers? show all working.
Penny Nom lui répond.
Four carpenters can build eight houses in 10 days. 2011-11-23
Kenneth pose la question :
Four carpenters can build eight houses in 10 days. Two carpenters can build how many houses in 15 days?
Penny Nom lui répond.
Water pouring into a conical tank 2011-11-21
Patience pose la question :
Hi my name is patience and I'm having a problem with this question.
Water pours into a conical tank of semi vertical angle 30 degrees at the rate of 4 cm^3/s, where h is the depth of the water at time t. At what rate is the water rising in the tank when h = 10 cm?
Thank you

Penny Nom lui répond.
Lines tangent to y^2=4x 2011-11-11
Reuchen pose la question :
Find equations of the lines tangent to y^2=4x and containing (-2,1).
Penny Nom lui répond.
A scale drawing 2011-10-30
aretha pose la question :
i have a scale drawing of a house floor plan, don't understand how to find the answer. the drawing is 1in: 3ft. need to find the length / width/and the actual length/width of the living room,kitchen, 3bedrooms deck,sitting room, entry, and closet
Penny Nom lui répond.
A spherical ball in a conical wine glass 2011-10-26
Jules pose la question :
A heavy spherical ball is lowered carefully into a full conical wine glass whose depth is h and whose generating angle (between the axis and a generator) is w. Show that the greatest overflow occurs when the radius of the ball is (h*sin(w))/(sin(w)+cos(2w)).
Claude Tardif lui répond.
Implicit differentiation 2011-10-20
Monica pose la question :
Find dy/dx in terms of x and y, if sin(xy)=(x^2)-y.
Penny Nom lui répond.
One central circle and three tangent circles 2011-10-16
Margaret pose la question :
You have one central circle and three or more circles tangent to the outside of the circle of varying radii. You know the x,y coordinates of the centers of the other circles. If you now remove that central circle (and pretend you never knew where it was), can you calculate its center in x,y coordinates?
Chris Fisher lui répond.
Building a custom range hood 2011-10-08
Bill pose la question :
I'm building a custom range hood for a customer with special order material that matches their newly installed cabinets and I need it to be perfect. The hood is basically a pyramid but the 4th side is the flat wall at the back and a flat, rectangular top. I need to calculate the bevel and miter of the three sides but I never was very good with geometry functions (although I am fairly good with other math fields). I either need the calculations from you at least (shudder) a formula or set of formulas so that I can calculate them myself.
Harley Weston lui répond.
A hemispherical bowl with a lead ball inside 2011-09-27
Jean pose la question :
"(a) Water is being poured into a hemispherical bowl of radius 3 inch at the rate of 1 inch^3/s. How fast is the water level rising when the water is 1 inch deep ?

(b) In (a), suppose that the bowl contains a lead ball 2 inch in diameter, and find how fast the water level is rising when the ball is half submerged."

Penny Nom lui répond.
The derivative of f(x) = (x+1)^1/2 2011-09-05
Carla pose la question :
Find the derivative using the limit process of
f(x) = (x+1)^1/2

Harley Weston lui répond.
The height of a fluid in a horizontal tank 2011-07-24
jason pose la question :
Same set up as many others, cylindrical tank on its side, but I am interested in defining the change in volume and/or fluid level as a function of time at a constant volumetric outflow. I plan on hooking a pump to the tank so "gpms' will be constant. I have a couple different sized tanks and pumps so I want a general equation. Thanks for your help.
Harley Weston lui répond.
A line tangent to f(x)=1/x 2011-06-05
Michael pose la question :
A line tangent to f(x)=1/x in the first quadrant creates a right triangle with legs the x-axis and the y-axis. Prove that this triangle is always 2 square units regardless of where the point of tangency is.
Penny Nom lui répond.
An exclusion zone around a triangle II 2011-05-03
Aishwarya pose la question :
A long time ago Mr gibson found an island shaped as a triangle with three straight shores of length 3km, 4km and 5km. He declared an exclusion zone around his island and forbade anyone to come within 1km of his shore. What was the area of his exclusion zone? This question was answered but did not understand the explanation.
Robert Dawson lui répond.
The radius of a cylinder 2011-04-27
Jazmin pose la question :
Hi, I don't understand how to find the radius in a cylinder with only the surface area (143.7) and the height (0.8)? I know that the formula is 2pir2+2pirh, but I don't see how to isolate the r? I appreciate your help.
Penny Nom lui répond.
An antiderivative of the square root of (8t + 3) 2011-04-19
Caitlyn pose la question :
I know how to take an antiderivative. But this one's stumping me. I need it to finish a problem. What's the antiderivative of the square root of (8t + 3)

~Caitlyn=

Penny Nom lui répond.
Eliminate y 2011-04-07
Lynn pose la question :
2x + y = 8
y + 3z =5
z + 2w =1
5w + 3x = 9

Form three equations with y eliminated

Penny Nom lui répond.
Designing a tin can 2011-03-31
Tina pose la question :
A tin can is to have a given capacity. Find the ratio of the height to diameter if the amount of tin ( total surface area) is a minimum.
Penny Nom lui répond.
A stone is dropped into a lake 2011-03-24
AnneMarie pose la question :
A stone is dropped into a lake, creating a circular ripple that travels outward at a speed of 25 cm/s. Find the rate at which the area within the circle is increasing after 4s.
Penny Nom lui répond.
At what rate is the grain pouring from the chute? 2011-02-26
MJ pose la question :
Suppose that grain pouring from a chute forms a conical heap in such a way that the height is always 2/3 the radius of the base. At the moment when the conical heap is 3 m high, its height is rising at the rate of 1/2 m/min. At what rate (in m^3/min) is the grain pouring from the chute?
Penny Nom lui répond.
Mathematics and a musical dilemma 2011-01-19
rahul pose la question :
how is mathematics applied in entertainment?
Harley Weston lui répond.
Integrating ln^3x/x 2011-01-14
ken pose la question :
y=ln^3x/x from x=1 to x=11
Penny Nom lui répond.
How do I prove that the quadrilateral is a Rhombus? 2010-12-16
Matthew pose la question :
Quadrilateral KLMN has vertices K(2,3), L(7,3), M(4,7) and N(-1,7). How do I prove that the quadrilateral is a Rhombus??
Robert Dawson and Penny Nom lui répond.
Simultaneous equations 2010-12-05
ryan pose la question :

Question from ryan, a student:

3         4
--   -    --      =     1      (1)
x         y

7       2             11
--   -  --        =   --       (2)
x       y              12


Chris Fisher and Stephen La Rocque lui répond.
A 400 gallon drum 2010-11-04
Jerry pose la question :
Question from Jerry:

I want to build A tank. able to hold 400 gallons of asphalt sealer it will be round . basically like A old fuel oil drum but made with heavy material . math ? was I want tank to hold 400 gallons I think the length of 60" would be perfect. I don't know what diameter or radius of tank needs to be . to make it A 400 Gallon tank

Harley Weston lui répond.
The angles in an m-gon and genrealizations 2010-10-16
Michael pose la question :
Hello: In answer to a student's question, someone named Penny from your organization provided a proof that the sum of the interior angles of a triangle in the plane is pi radians (or 180 degrees).

I am interested (and I'm sure many other people would be too) in 3 potential generalizations of this basic fact in plane geometry:

Walter Whiteley lui répond.
What is the maximum weekly profit? 2010-10-10
Joe pose la question :
A local artist sells her portraits at the Eaton Mall. Each portrait sells for $20 and she sells an average of 30 per week. In order to increase her revenue, she wants to raise her price. But she will lose one sale for every dollar increase in price. If expenses are $10 per portrait, what price should be set to maximize the weekly profits? What is the maximum weekly profit?
Stephen La Rocque and Penny Nom lui répond.
A Taylor polynomial for (lnx)/x 2010-09-29
Dave pose la question :
I have a series problem that I cannot solve. The problem asks for you to compute a Taylor polynomial Tn(x) for f(x) = (lnx)/x. I calculated this poly out to T5(x) and attempted to use this to identify a pattern and create a series in order to calculate Tn(x). However, the coefficients on the numerator out to F5prime(x) are as follows: 1, -3, 11, -50, 274... Ok, so the negative is an easy fix -> (-1)^n-1. But the other coefficients are stumping me. I can't see any sort of pattern there and I've tried every trick I know. Is there another way to go about this? Thanks!
Chris Fisher lui répond.
limit as x approaches a of ((x^(1/2))-(a^(1/2)))/(x-a)? 2010-09-29
emily pose la question :
limit as x approaches a of ((x^(1/2))-(a^(1/2)))/(x-a)?
Penny Nom lui répond.
A limit 2010-09-27
norma pose la question :
I have a problem like this one but I can get it right. please help me to answer find the constant a such that the function is continuous on the entire line.
g(x)= {x^2 - a^2 / x-a if x is not = a
         {6 if x = a

Penny Nom lui répond.
Continuity 2010-09-18
Carina pose la question :
Hi. My name's Carina and I'm currently a sophomore in high school. I'm having a lot of difficulties in AP Calculus with continuity, one-sided limits, and removable discontinuities. Basically, I have no idea how to do them or even what they are. I read the lesson but I still don't get it. Can someone put it in simpler terms so I can understand how to complete my questions? Thank you!
Robert Dawson lui répond.
Maximizing the volume of a cylinder 2010-08-31
Haris pose la question :
question: the cylinder below is to be made with 3000cm^2 of sheet metal. the aim of this assignment is to determine the dimensions (r and h) that would give the maximum volume. how do i do this? i have no idea. can you please send me a step-to-step guide on how t do this? thank you very much.
Penny Nom lui répond.
A max min problem 2010-08-19
Mark pose la question :
a rectangular field is to be enclosed and divided into four equal lots by fences parallel to one of the side. A total of 10000 meters of fence are available .Find the area of the largest field that can be enclosed.
Penny Nom lui répond.
The suspension cables of a bridge 2010-07-29
Mike pose la question :
what is the formula for the suspension cables of a bridge. The towers are 200 ft above the roadway The towers are 3400 ft apart The cable if at 8ft in the middle of the span
Robert Dawson lui répond.
Maximize the floor area 2010-07-07
shirlyn pose la question :
A rectangular building will be constructed on a lot in the form of a right triangle with legs of 60 ft. and 80 ft. If the building has one side along the hypotenuse, find its dimensions for maximum floor area.
Penny Nom lui répond.
A max/min problem 2010-06-12
valentin pose la question :
What is the maximum area of an isosceles triangle with two side lengths equal to 5 and one side length equal to 2x, where 0 ≤ x ≤ 5?
Harley Weston lui répond.
The capilano suspension bridge 2010-06-03
nida pose la question :
the capilano suspension bridge in north vancouver is the world's highest footbridge of its kind. the bridge is 140m long . from the ends of the bridge the angles of depression of a point on the river under the bridge are 41 degrees and 48 degrees. how high is the bridge above the river to the nearest metre
Penny Nom lui répond.
Integration of sin^3 (2x) 2010-05-29
ascher pose la question :
how do you integrate this equation ∫ sin^3 (2x) dx
Robert Dawson and Penny Nom lui répond.
More on a truncated cone 2010-05-28
Mike pose la question :

Question from Mike, a parent:

I was reviewing this question and answer:
http://mathcentral.uregina.ca/QQ/database/QQ.02.06/phil1.html

But I have trouble with this part:
Now if we express the radius of the inside circle as r and the outside circle's radius is R, then this means r/R is 911/1728. But earlier we said that the outside radius R is simply w more than the inside radius r, so R = r + 282. That means that r/R = r/(r + 282). Now we can simply solve the equation for r:
r/(r+282) = 911/1728
This means r = 314 mm (with rounding).

Can I get more detail on the method to solve for r?

Thank you,
Mike


Penny Nom lui répond.
An optimization problem 2010-05-23
Marina pose la question :
Hello, I have an optimization homework assignment and this question has me stumped..I don't even know

A hiker finds herself in a forest 2 km from a long straight road. She wants to walk to her cabin 10 km away and also 2 km from the road. She can walk 8km/hr on the road but only 3km/hr in the forest. She decides to walk thru the forest to the road, along the road, and again thru the forest to her cabin. What angle theta would minimize the total time required for her to reach her cabin?
I'll do my best to copy the diagram here:

                             10km
Hiker_ _ _ _ _ _ _ _ _ _ _ _ _ _ _ _ _ _ _ _ _Cabin
      \                           |                              /
       \                          |                             /
     f  \                      2km                          /
         \                        |                           /
theta   \___________________________ /
                            Road


Penny Nom lui répond.
The hypotenuse of a triangle 2010-05-22
linda pose la question :
find the length of the hypotenuse of a triangle with legs of 12in. and 17in. round to the nearest hundredth
Tyler Wood lui répond.
Extraneous solutions 2010-05-22
Joe pose la question :

Question from Joe, a parent:

w+3    2w
----- - ----- = 1
w2-1   w-1

W2 is = w squared

                            -4
The answer is w= ------
                             3

but have no idea how this was solved. any help is appreciated. Thanks.


Penny Nom lui répond.
The rate of change of y with respect to x 2010-04-29
Tom pose la question :
I just had a quick calc question about wording that wasn't ever addressed in class. When the book says "the rate of change of y with respect to x", should it be considered how fast y is changing in comparison to x?

I ask because the textbook says that "y is changing 3 times faster than x, so the rate of change of y with respect to x is 3." I'm use to rate being like velocity, as in units of distance per units of time. All we're told in class is that it's the slope of the tangent line, I was hoping you could clarify for me what exactly is meant by the wording of a "rate of change of something with respect to something else". More specifically, what "rate" and "with respect to" mean within this context?

Thanks for your time

Harley Weston lui répond.
A rectangular garden 2010-04-25
Billy pose la question :
Tanisha wants to make a rectangular garden with a perimeter of 38 feet. What is the greatest area possible that tanisha can make the garden?
Penny Nom lui répond.
Integrate the ((4th root of x^3)+1) dx 2010-04-12
Bridget pose la question :
integrate the ((4th root of x^3)+1) dx
Tyler Wood lui répond.
The derivative of y=x^x 2010-04-09
David pose la question :
So, its David, and I was wondering about the derivative of y=x^x. I have often seen it be shown as x^x(ln(x)+1), but when I did it through limits it turned out differently. Here's what I did:
It is commonly know that df(x)/dx of a function is also the limit as h->0 of f(x+h)-f(x)/h. To do this for x^x you have to start with lim h->0 ((x+h)^(x+h)-x^x)/h. The binomial theorem then shows us that this is equal to lim h->0 (x^(x+h)+(x+h)x^(x+h-1)h+...-x^x)/h
This is also equal to lim a->0 lim h->0 (x^(x+a)+(x+h)x^(x+h-1)h...-x^x)/h.
Evaluating for a=0 you get lim h->0 (x^x+(x+h)x^(x+h-1)h...x^x)/h
Seeing as the last 2 terms on the numerator cancel out you can simplify to a numerator with h's is each of the terms, which you can then divide by h to get:
lim h->0 (x+h)x^(x+h-1)... which when evaluated for h=0 gives us: x(x^(x-1)). This statement is also equal to x^x.
This contradicts the definition of the derivative of x^x that is commonly shown. So, my question is: can you find any flaws in the logic of that procedure? I do not want to be shown how to differentiate x^x implicitly because I already know how to do that.

Robert Dawson lui répond.
A max min problem 2010-04-06
Terry pose la question :
The vertex of a right circular cone and the circular edge of its base lie on the surface of a sphere with a radius of 2m. Find the dimensions of the cone of maximum volume that can be inscribed in the sphere.
Harley Weston lui répond.
The derivative of cos^3x 2010-04-06
Erson pose la question :
Find y' of the given function: y = cos^3x.
Harley Weston lui répond.
Sand falling off a conveyer 2010-04-02
Katherine pose la question :
sand is falling off a conveyer onto a pile at the rate of 1.5 cubic feet per minute. The diameter of the base is approximately twice the altitude. At what rate is the height of the pile changing when it is 10 feet high?
Penny Nom lui répond.
A negative times a negative 2010-03-25
priya pose la question :
why is minus into minus plus?
Harley Weston lui répond.
A 14 side well house cover 2010-03-12
Kenneth pose la question :
I am 35 yr I am wanting to build a well house cover. I'm trying to figure out how long the pieces need to be and what angle they need to be for a 4 ft dia with 14 side well house. I would love an answer but would also like to know how to figure it in the future. Thanks Kd
Harley Weston lui répond.
The integral of X^3/the square root of 1-x^2 dx 2010-03-07
William pose la question :
The integral of X^3/the square root of 1-x^2 dx.
Harley Weston lui répond.
The volume of a frusta of a hexagonal based pyramid 2010-03-04
sarah pose la question :
Volume of a frusta of a hexagonal based pyramid
Penny Nom lui répond.
Lissajous curve 2010-03-03
Nikki pose la question :
I'm interested in information about a particular mathematical figure. My memory is that it is called a "liciju figure", but obviously my spelling of this is incorrect because a google search of this and it's variants has revealed nothing. I believe it's related to the Moebius strip and probably connected with radio waves. It is used as the logo for our national broadcaster (The Australian Broadcasting Corporation) and you see exactly what I'm talking about by going on their website: www.abc.net.au. I have tried contacting them directly, but have received no response in over a month now!
Harley Weston lui répond.
The hypotenuse 2010-02-27
Dannielle pose la question :
how do you find the hypotenuse if a=8 and b=6?
Penny Nom lui répond.
A square corner 2010-02-11
Trevor pose la question :
I am building a new house and wish to set it out on site with the use of profile boards and string. I want to be certain it is correct in terms of squareness. I have a vague idea that the square on the hypotenuse should be equal to the sum of the squares of the other two sides.

I get a little lost here and need some help. The building is a rectangle measuring 40x30 feet to exterior brickwork. I guess that the length of the hypotenuse should be exactly the square root of the combined squares of the two sides.

Using the above measurements could you give me calculations from nuts to soup as to the correct length of the diagonal. And what adjustments are needed if everthing is not in accord.

Trevor.

Robert Dawson lui répond.
The distance from a chord to an arc 2010-02-11
matt pose la question :
hello, I have to layout a radius without being able to pull from the center my radius is 3819.53 feet and i have a chord length of 275.59 feet if i broke that up into 25.05 feet sections how would i calculate the lengths from my chord to that radius?
Robert Dawson lui répond.
A tunnel from Toronto to Montreal 2010-01-25
Dave pose la question :
I want to make a tunnel from Toronto to Montreal (for example)

Something like this http://mathcentral.uregina.ca/QQ/database/QQ.09.09/h/grant1.html
-------------
My coordinates are 45.442455,-73.861340 (Montreal) and 43.442455, -79.861340 (Toronto)

I need to know how to find arc distance, chord distance and radius What websites can i find for this subject Google has many but they are useless (blah blah) websites
LOL
Thanks

Chris Fisher and Robert Dawson lui répond.
The inradius of an icosahedron 2010-01-21
fnavroth pose la question :
Suppose you have an equilateral triangle. The area of the triangle is exactly 1200 square centimetres. Now suppose you have twenty of those triangles. It's possible to assemble those twenty triangles into a closed three-dimensional shape, a regular polyhedron. What would be the volume, in cubic centimetres, of the largest sphere that could fit inside the shape?
Chris Fisher lui répond.
A cone circumscribed about a given hemisphere 2010-01-19
Neven pose la question :
The cone of smallest possible volume is circumscribed about a given hemisphere. What is the ratio of its height to the diameter of its base?
(G.F.Simmons, Calculus with Analytic Geometry, CH4 Applications of Derivatives)

Chris Fisher lui répond.
A push-button padlock 2010-01-17
Vince pose la question :
Hi,

I have a push-button padlock using ten buttons (1-2-3-4-5-6-7-8-9-0). Five digits must be pushed in order to open the lock. Each digit can only be used once. Order is not supposed to matter.

How many different possible combinations? And what are they?

Harley Weston lui répond.
A pair of simultaneous equations 2010-01-09
Yumiko pose la question :
Solve the following pair of simultaneous equations.

x^2 -4x = y^2-4
3y=2x - 3

Penny Nom lui répond.
A question from a boat builder 2010-01-01
Grant pose la question :
I am a boat builder, trying to lay out shape of side's elevation. My question is, how do I define the length of a circle's radius, if I know the chord length (20 ft) and the segment of the radius between the chord and the circle is known (7 inches)?
Penny Nom lui répond.
Chord length given the length & radius of an arc 2009-12-31
Wayne pose la question :
Given the length & radius of an arc, is there a formula that will accurately calculate the chord length? I'm an architectural designer, and would need it explained in layman's terms. Thank you.
Wayne

Penny Nom lui répond.
The volume of a frustum 2009-12-29
dave pose la question :
I have a frustum top 1.7r bottom .55r and 2.14 h I have to calculate cement in a silo every week. I am not very good at maths but i have been adding the top and bottom to get an average so as to turn it into a cylinder and i come up with 8.5m3 I know that the correct volume is 9.24 m3. Can you tell me why getting an average width on the top and bottom of a frustum doesn't work.
Thank you.

Chris Fisher lui répond.
f(x)=x+2sinx 2009-12-12
amroziz pose la question :
for which values of x does the graph of f(x)=x+2sinx have horizontal tangent
Harley Weston lui répond.
How fast is the distance between the two cars decreasing? 2009-12-08
Jenny pose la question :
Two cares are on a collision course toward point P. The paths of the two cars make a 30 degree angle with each other. The first car is 40 km from P, and traveling toward P at 16 km/hour. The second car is 50 km from P, traveling at 20 km/hour. How fast is the (straight line) distance between the two cars decreasing. (Hint: Law of Cosines)
Harley Weston lui répond.
Solving two equations, one with a square root 2009-11-23
kacie pose la question :
y = square root of x+3
x-4y = -7

im having trouble with this problem...i have to find where they intersect.

Harley Weston lui répond.
The triangle formed by the tangent and the coordinates axes 2009-11-23
Nirmala pose la question :
Given that y=1/x, x is not equal to zero. Prove that the area of the triangle formed by the tangent and the coordinates axes is 2.
Harley Weston lui répond.
Excluded values 2009-11-14
Janie pose la question :
I have to State the excluded values for this equation and then solve, but not sure how to do this. Here is the problem
(x+6)/x+3=(3)/(x+3)+2

Harley Weston lui répond.
f(x)= (e^x) / [(e^x)+(ex^2)] 2009-11-10
natalie pose la question :
I'm trying to graph the function, f(x)= (e^x) / (e^x)+(ex^2) [e to the x divided by e to the x plus e times x squared] I know that there aren't any vertical asymptotes, but is there a horizontal asymptote? and also, I'm stuck on finding the concavity for this graph. I tried to find f "(x), but it came out to be really long and I am not sure how to find the x values for f "(x) without using a graphic calculator. thanks, natalie
Chris Fisher and Harley Weston lui répond.
At what rate are the people moving apart? 2009-11-01
saira pose la question :
A man starts walking north at 4 ft/s from a point P. 5 minutes later a woman starts walking south at 5 ft/s from a point 500 ft due east of P. At what rate are the people moving apart 15 minute after the woman starts walking ?
Harley Weston lui répond.
A path around a pond 2009-10-31
adeniji pose la question :
find the area of a concrete path 2m wide surrounding a circular pond 12m in diameter
Penny Nom lui répond.
Painting a dome 2009-10-30
Jessica pose la question :
A hemispherical dome with a radius of 50 ft will be given a coat of paint .01 inch thick. The Contractor for the job wants to estimate the number of gallons of paint needed. Use a differential to obtain an estimate (231 cubic inches/gallon) HINT: Approximate the change in volume of hemisphere corresponding to increase of .01 inch in the radius.
Robert Dawson lui répond.
Graphing y=(4-x^2)^5 2009-10-25
natalie pose la question :
I want to graph the curve of y=(4-x^2)^5 without using a graphing calculator. To do this, I'm suppose to find: domain, y and x intercepts, asymptotes, intervals of increase/decrease, local max/min, concavity and points of inflection. I got all the way to the step where I'm solving the concavity and I'm stuck. I found the f"(x) and it came out to be really large polynomial. I want to know how I can solve for the x of f"(x) without the use of a graphing calculator, when the polynomial has x^6 and x^8. Thank you so much, natalie
Harley Weston lui répond.
The hypotenuse of a right triangle 2009-10-18
steven pose la question :
the perimeter of a right triangle is 20 cm. its area is 15 sq cm. find its hypotenuse.
Penny Nom lui répond.
A max/min problem 2009-10-12
avien pose la question :
a rectangle has a line of fixed length Lreaching from the vertex to the midpoint of one of the far sides. what is the maximum possible area of such a rectangle? SHOW SOLUTION USING CALCULUS
Penny Nom lui répond.
A line tangent to a parabola 2009-10-01
kanchan pose la question :
for what value of c a line y=mx+c touches a parabola y^2=4a(x-a)
Penny Nom lui répond.
Sawdust 2009-09-29
joel pose la question :
What is the density of saw dust
Harley Weston lui répond.
Probability 2009-09-27
Ed pose la question :
My mother died 3 years to the day after her daughter died. what are the odds of that happening by chance? thanks
Chris Fisher lui répond.
Extraneous solutions 2009-09-20
iyana pose la question :
what is an extraneous solution? what must you do to determine whether a extraneous solution?
Stephen La Rocque and Harley Weston lui répond.
solve integral of ( x^2+x+1)^5 2009-09-18
jaka pose la question :
solve integral of ( x^2+x+1)^5
Robert Dawson lui répond.
Sagitta 2009-09-10
Robert pose la question :
Can you please tell me if there is a formula to figure out the Sagitta of an arc when you know the radius, chord length, and arc length?
Chris Fisher and Harley Weston lui répond.
A circular border around a pool 2009-09-08
Calvin pose la question :
A pool in the shape of a circle measures 10 feet across. One cubic yard of concrete is to be used to create a circular border of uniform width around the pool. If the border is to have a depth of 3 inches, how wide will the border be? ( 1 cubic yard=27 cubic feet )
Stephen La Rocque lui répond.
Simultaneous equations 2009-08-28
onias pose la question :
solve 3/a - 2/b = 1/2 , 5/a + 3/b = 29/12
Robert Dawson lui répond.
The layout of an arch 2009-08-18
Steven pose la question :
I am trying to layout a large radius between 2 points in a building and need a formula to figure different senarios for example: radius is 187'6" distance between 2 points is 34'8" need points 16" apart along the line between the 2 points to create the radius please help
Stephen La Rocque lui répond.
An antiderivative problem 2009-08-13
Indrajit pose la question :
∫4e^x + 6e^-x/(9e^x + 4e^-x)dx = Ax + Bloge(9e2x - 4) + C

then A=?......B=?.....C=?

plz solve it...."^" stands for "to the power of"....

Harley Weston lui répond.
A rectangular pen 2009-08-13
Kari pose la question :
A rectangular pen is to be built using a total of 800 ft of fencing. Part of this fencing will be used to build a fence across the middle of the rectangle (the rectangle is 2 squares fused together so if you can please picture it). Find the length and width that will give a rectangle with maximum total area.
Stephen La Rocque lui répond.
Torricelli's trumpet 2009-07-29
Gary pose la question :
I was reading about torricelli's trumpet which is described by the equation1/x which is then rotated around the x axis which results in a figure which looks like a trumpet. Now in order to find the volume the integral 1/x^2 dx is used which diverges when integrated so the volume is finite.However if you integrate 1/x dx which is the formula on the plane the answer diverges. Now if you took an infinite area then rotated it around the x axis shouldn't you get an infinite volume? Notice the area I am talking about is under the line 1/x not the surface area of the trumpet which is what the painters paradox is about What am I missing? Thanks
Robert Dawson lui répond.
Annular sector 2009-07-20
Ed pose la question :
What is the name for the section of a flat ring (annulus)? Similar to a section on a radar screen, a quadrilateral but the top and bottom are curved.
Robert Dawson lui répond.
Simultaneous Equations 2009-07-06
Mukulu pose la question :
Solve the equation simultaneously X/5=(Y+2)/2= (Z-1)/4 ……………….eqt 1 3X+4Y+2Z-25=0 ………………eqt 2
Janice Cotcher lui répond.
The integral of x^x 2009-06-18
ANGIKAR pose la question :
what would be the integration of (X^Xdx)?

give answer in details.

Robert Dawson and Harley Weston lui répond.
The radius of an arc 2009-06-12
Billy pose la question :
I have tried using the formula 4h2(squared)plus L2(squared)divided by 8h to find the radius of an arc, but I must be doing something wrong since I keep getting the wrong answer. Can you tell me what I am doing wrong. The height is 37.75 in. and the length is 18.875 in. Thank you for any help you can give me.
Stephen La Rocque lui répond.
Extraneous solutions 2009-06-02
Ayana pose la question :
solve and check for extraneous solutions.

3x+6/ x²-4 = x+1/ x-2

x can not = {-2,2}

Penny Nom lui répond.
Two ships and a lighthouse 2009-05-27
Chelsey pose la question :
I have a question in regards to how do I know when to use tangent or cosine when determining angles. The question is: Looking north from the observation deck of a lighthouse 60 m above the sea, a lighthouse keeper sees two ships. The angles of depression to the ships is 5 degrees and 10 degrees. How far apart are the ships?

I don't understand which one to use when solving the equation.

Harley Weston lui répond.
differentiate y sin[x^2]=x sin[y^2] 2009-05-11
mamiriri pose la question :
derivate y sin[x^2]=x sin[y^2]
Harley Weston lui répond.
The integral of a to power x squared 2009-04-28
JIM pose la question :
WHEN I ATTENDED U.OF T. (TORONTO ) MANY YEARS AGO WE WERE TOLD THE FOLLOWING INTEGRAL COULD NOT BE SOLVED : a to power x squared . is this still true ?

CURIOUS , JIM

Robert Dawson lui répond.
A max-min problem 2009-04-20
Charlene pose la question :
A fixed circle lies in the plane. A triangle is drawn inside the circle with all three vertices on the circle and two of the vertices at the ends of a diameter. Where should the third vertex lie to maximize the perimeter of the triangle?
Penny Nom lui répond.
Sand falls from a conveyor belt 2009-04-01
Tracy pose la question :
Sand falls from a conveyor belt at the rate of 10 cubic feet per minute onto a conical pile. The radius of the base is always equal to half the pile's height. How fast is the height growing when the pile is 5ft high?
Stephen La Rocque lui répond.
A spherical Tootsie Roll Pop 2009-04-01
Tracy pose la question :
A spherical Tootsie Roll Pop you are sucking on is giving up volume at a steady rate of .8 ml/min. How fast will the radius be decreasing when the Tootsie Roll Pop is 20 mm across?
Harley Weston lui répond.
Uses of Pythagorean theory 2009-03-27
Britta pose la question :
Please, give me some complex real life situation examples where the pythagorean theory is used. It must be a grade 8 or grade 9 level of thinking as that is what is my teacher's demands.
Robert Dawson lui répond.
An isosceles triangle 2009-03-26
sela pose la question :
An isosceles triangle has two equal sides of length 10 cm. Theta is the angle between two equal sides.
a) Express area of a triangle as a function of theta
b) If theta is increasing at a rate of 10 degrees/minute, how fast is area changing at the instant theta=pi/3?
c) at what value of theta will the triangle have the maximum area?

Penny Nom lui répond.
The rate of change of the volume of a sphere 2009-03-25
Kaylin pose la question :
why the rate of change of volume of a sphere is not constant even though dr/dt is constant?
Walter Whiteley lui répond.
A max-min problem 2009-03-24
Jay pose la question :
Determine the area of the largest rectangle that can be inscribed between the x-axis and the curve defined by y = 26 - x^2.
Harley Weston lui répond.
The diameter of a roll of plastic 2009-03-24
truong pose la question :
hi. i have trouble to calculate the diameter of the plastic roll. the sheet is 765 m long and 0.8 mm to wrap around the core 400 mm in dia. please help me with the formula to calculate the dia of plastic roll, thanks in advance
Harley Weston lui répond.
The radius of a circle 2009-03-22
Justin pose la question :
Find the radius of a circle with a circumference of 9.43cm.
Penny Nom lui répond.
The angles of a triangle 2009-03-11
Marissa pose la question :
The angles in a triangle measure 7x-1, 18x+2, and 5x+10. Determine whether the triangle is acute, obtuse, or right. State your reasons clearly.
Robert Dawson lui répond.
A common tangent to two curves 2009-03-02
Jay pose la question :
For what values of a and b will the parabola y = x^2 + ax + b be tangent to the curve y = x^3 at (1,1)?
Penny Nom lui répond.
Implicit differentiation 2009-03-01
Emily pose la question :
determine the derivative y' at the point (1,0)
y= ln(x^2+y^2)

y'(1)= ??

Stephen La Rocque lui répond.
Implicit differentiation 2009-02-18
Sunny pose la question :
Find slope of the tangent line to the curve 2(x^2+y^2)2=25(x^2–y^2) at (3,-1)
Robert Dawson and Harley Weston lui répond.
The area between the x-axis and a curve 2009-02-18
Lauren pose la question :
This is from a homework question I can't figure out.

Let R be the region in the fourth quadrant enclosed by the x-axis and the curve y= x^2 - 2kx, where k > 0. If the area of the region R is 36 then what is the value of k?

Robert Dawson lui répond.
The second derivative of h(x)=f(g(x)) 2009-02-16
Kristina pose la question :
If h(x)=f(g(x)), and is differentiable, then find h"(x).
Robert Dawson lui répond.
A definite integral 2009-02-09
Mathata pose la question :
Evaluate: integral from 0 to 1, x^2 e^x^3dx
Harley Weston lui répond.
A trig limit 2009-02-05
Samantha pose la question :
lim x-> 0 ( ( r*cos(wt +h) + r*cos(wt) )/ h )

Where r & w are constants.

Harley Weston lui répond.
A point on -8x^2+5xy+y^3=-149 2009-02-04
Vivian pose la question :
Consider the curve defined by -8x2+5xy+y3=-149
a) find dy/dx
b) Write an equation for the line tangent to the curve at the point (4,-1)
c) There is a number k so that the point (4.2,k) is on the curve. Using the tangent line found in part b), approximate the value of k.
d) write an equation that can be solved to find the actual value of k so that the point (4.2,k) is on the curve
e) Solve the equation found in part d) for the value of k

Harley Weston lui répond.
Simultaneous equations with an xy term 2009-02-01
angelee pose la question :
xy+5x-2y-10=0 2x+y=1
Penny Nom lui répond.
limit sinx/x 2009-01-30
Jackie pose la question :
how to evaluate limit sinx/x as x tends to zero if x is in degrees
Stephen La Rocque and Harley Weston lui répond.
An integral from 1 to infinity 2009-01-24
Ray pose la question :
Determine the area bounded by the x-axis and the curve y=1/(x^2) from x=1 to x=infinity.
A. 1.00
B. infinity
C. indeterminate
D. 2.00

Harley Weston lui répond.
Archimedes' formula for parabolic arches 2009-01-23
La pose la question :
Use calculus to verify Archimedes' formula for y=9-x^2. Prove Archimedes' formula for a general parabolic arch.
Harley Weston lui répond.
In the shadow of a flagpole 2009-01-22
La pose la question :
How fast is the length of the shadow of an 18 foot flagpole growing when the angle of elevation of the sun is 45 degrees and is decreasing at a rate of 10 degrees per hour?
Harley Weston lui répond.
The parabola with vertex (7,-2) and directrix y = -3 2009-01-21
Deann pose la question :
Find an equation of the parabola with vetrex (7,-2) and directrix y =(-3)
Penny Nom lui répond.
Partial derivatives 2009-01-17
Meghan pose la question :
I have a question I've been working at for a while with maxima/minima of partial derivatives.

"Postal rules require that the length + girth of a package (dimensions x, y, l) cannot exceed 84 inches in order to be mailed. Find the dimensions of the rectangular package of greatest volume that can be mailed. (84 = length + girth = l + 2x + 2y)"

Harley Weston lui répond.
Negative rate of change 2009-01-12
hemanshu pose la question :
when i have to find rate of change of decrease in any value my ans comes in negative why??????????
Penny Nom lui répond.
What is the maximum revenue? 2009-01-09
Kristy pose la question :
A skating rink manager finds that revenue R based on an hourly fee x for skating is represented by the function R(x) = -200x^2 + 1500x

What is the maximum revenue and what hourly fee will produce maximum revenues?

Harley Weston lui répond.
A max/min problem 2009-01-09
Angelica pose la question :
have 400 feet of fence. Want to make a rectangular play area. What dimensions should I use to enclose the maximum possible area?
Robert Dawson lui répond.
The area of a region bounded by two curves 2009-01-07
Rogerson pose la question :
Find the area, S, enclosed by the given curve(s) and the given line.
y = x^2 - x - 1, y = x+2

Harley Weston lui répond.
A kennel with 3 individual pens 2009-01-06
Jean pose la question :
An animal clinic wants to construct a kennel with 3 individual pens, each with a gate 4 feet wide and an area of 90 square feet. The fencing does not include the gates. Write a function to express the fencing as a function of x. Find the dimensions for each pen, to the nearest tenth of a foot that would produce the required area of 90 square feet but would use the least fencing. What is the minimum fencing to the nearest tenth?
Harley Weston lui répond.
The area enclosed by a curve and the x-axis 2009-01-04
Rogerson pose la question :
Find the area, S, enclosed by the curve y = -x^2 + 6x - 5 and the x-axis in the interval 0≤x≤4.
Harley Weston lui répond.
Determine y'' by implicitly differentiating twice 2009-01-04
Walter pose la question :
Given x^3 - 3xy + y^3 = 1 , determine y'' by implicitly differentiating twice. I cannot solve this. Would you be kind enough to perform the mathematics and show the steps involved in obtaining the solution?
Harley Weston lui répond.
The area of a region in the plane 2009-01-03
Rogerson pose la question :
Find the area, S, of the shaded region enclosed by the given cureve, the given line and the x-axis.

y = -x^2 + 1
line x = 2

Harley Weston lui répond.
The radius of a cone 2009-01-02
kalpaj pose la question :
A conical funnel holds 100ml. If the height of the funnel is 10 cm, determine its radius, to the nearest tenth of a centimeter.
Penny Nom lui répond.
The volume of a pipe elbow 2008-12-31
mhd pose la question :
how i can find volume of the pipe elbow of 4inches?
Stephen La Rocque lui répond.
Pouring angles for a crucible 2008-12-20
Richard pose la question :
I am trying to work at pouring angles and volume left in during pouring a crucible, The crucible is cylindrical and flat bottomed.

I know the diameter, radius and volume of the crucibles. and the volume of liquid going into it.

So lets say the crucible is only half full firstly I need to work out the angle just before its going to pour. ( I can work this out as long as there is a certain volume of liquid if its not enough I cant do it)

Now the problem I also need to work out how much I should tilt the crucible to allow a certain amount out and be able to do this untill the volume reaches 0 at 90' turn. This is where I am stuck.

The reason for needing to be able to work this out is so i can develop a constant flow for example 10Kg of metal per second. Thank you very much for you time

Harley Weston lui répond.
Integral of cos^2 X between pi/2 and 0 2008-12-18
Wanda pose la question :
Integral or Area of cos^2 X between pi/2 and 0.

The answer that I got is -pi/4. Is this correct? If not, how did you come up with your answer?

Robert Dawson lui répond.
Solve for x 2008-12-16
Melissa pose la question :
I have a test tomorrow and I'm hoping you can help me before then. I can never seem to solve the "RESOLVE X" problems, or in french resous pour x/
They look like this. 3x+2\6=2x-5\3. I only understand NOTHING from that. Another equation is 2(x+1)=3(x+2).

Robert Dawson and Penny Nom lui répond.
The middle term of an arithmetic sequence 2008-12-15
Leigh pose la question :
Find the sum of the first fifteen terms of an arithmetic series if the middle term is 92
Penny Nom lui répond.
A sphere in a can of water 2008-12-12
Meghan pose la question :
A cylindrical can open at the top has (inside) base radius equal to 1. The height of the can is greater than 2. Imagine placing a steel sphere of radius less than 1 into the can, then pouring water into the can until the top of the sphere is just covered.

What should be the radius of the sphere so the volume of water used is as large as possible?

Harley Weston lui répond.
The third vertex of a right triangle 2008-12-10
prashant pose la question :
how to calculate coordinates of third vertex in a right angle triangle given the coordinates of hypotenuse as (1,3) and (-4,1)
Robert Dawson lui répond.
What is so important about quadratics? 2008-11-29
zoe pose la question :
what is so important about quadratics?
Harley Weston lui répond.
How fast is the distance between the airplanes decreasing? 2008-11-10
Crystal pose la question :
At a certain instant, airplane A is flying a level course at 500 mph. At the same time, airplane B is straight above airplane A and flying at the rate of 700 mph. On a course that intercepts A's course at a point C that is 4 miles from B and 2 miles from A. At the instant in question, how fast is the distance between the airplanes decreasing?
Harley Weston lui répond.
A trig limit 2008-11-04
Teri pose la question :
Although I have this problem completely worked out in front of me I still cannot understand how it was done. The problem is:
Find the limit.
lim x->0 sin2x/tan7x.

Harley Weston lui répond.
Separating variables 2008-11-04
Terry pose la question :
by separating variables solve the initial value problem

(x+1)y' + y = 0 y(0) = 1

Harley Weston lui répond.
Taxes in Taxylvania 2008-10-22
April pose la question :
Taxylvania has a tax code that rewards charitable giving. If a person gives p% of his income to charity, that person pays (35-1.8p)% tax on the remaining money. For example, if a person gives 10% of his income to charity, he pays 17 % tax on the remaining money. If a person gives 19.44% of his income to charity, he pays no tax on the remaining money. A person does not receive a tax refund if he gives more than 19.44% of his income to charity. Count Taxula earns $27,000. What percentage of his income should he give to charity to maximize the money he has after taxes and charitable giving?
Harley Weston lui répond.
Antiderivative of 1/(x(1 - x)) 2008-10-22
Matt pose la question :
derivative of dx/(x(1-x))

From what I've seen I should break apart the equation as such derivative of dx/x - dx/(1-x) and then get the 2 corresponding log functions.

If that is correct why does this factoring work, if that is incorrect what is the proper way to find the derivative.

Harley Weston lui répond.
The slope of a tangent line 2008-10-18
Amanda pose la question :
If f(x)=square root of (x+4), and the slope of the tangent line at x=21 was 1/n for some integer n, then what would you expect n to be?
Stephen La Rocque lui répond.
Two equations in two unknowns 2008-10-17
Dushayne pose la question :
Please help me in solving this problem:
a. 3x-4y=32
5x+2y=10

b. 2x+3y=11
4x+3y=10

Penny Nom lui répond.
Two modular equations 2008-10-08
Mhiko pose la question :
please solve this Chinese remainder problem..and give me a solution or rule in order to solve this problem/

x=2mod15
x=1mod25

Stephen La Rocque lui répond.
Finding the Distance Between Two Latitudes 2008-10-02
Samua pose la question :
Assuming that the Earth is a sphere of radius 4000 miles and that the cities are on the same longitude (one city is due north to the other). Find the distance between the cities with the latitudes of 37 degrees 47'36'' and another city with 47 degrees 37'18''. Heeeeeeeeeeelp!
Janice Cotcher lui répond.
Extraneous solutions 2008-09-25
crystal pose la question :
/6x+7/=5x+2
Penny Nom lui répond.
The region between two circles 2008-09-24
Carol pose la question :
Good day! Here is a picture of the problem that we need to solve. (I send the picture through e-mail.) A small circle is inside a larger circle, the only given in the problem is the chord of the larger circle tangent to the smaller circle which measures 16cm. The question is, what is the area of the shaded region? Can you answer this question? Thanks! :)
Harley Weston lui répond.
The biggest right circular cone that can be inscribed in a sphere 2008-09-08
astrogirl pose la question :
find the volume of the biggest right circular cone that can be inscribed in a sphere of radius a=3
Harley Weston lui répond.
An exclusion zone around a triangle 2008-09-07
Awrongo pose la question :
A long time ago Mr Gibson found an island shaped as a triangle with three straight shores of length 3 km,4 km and 5 km. He declared an 'exclusion zone' around his island and forbade anyone to come within 1 km of his shore. What was the area of his exclusion zone?
Stephen La Rocque and Penny Nom lui répond.
A string around the world 2008-08-18
Terry pose la question :
I heard this on T.V. and was wondering if it was true. Place a string around the world, you would need 25,000 miles of string. If you raised the string off of the ground by 1 foot all the way around. How much more string would you need?
Penny Nom lui répond.
[f(x)-f(1)]/(x-1) 2008-08-14
katie pose la question :
Evaluate (if possible) the function of the given value of the independent variable:

f(x)=(x^3)-x:

[f(x)-f(1)]/(x-1)

Penny Nom lui répond.
Arc-length and sector-angle 2008-08-06
Benson pose la question :
If chord length, radius are given, How to find the sector angle and arc-length
Janice Cotcher lui répond.
Integral of X^2 2008-07-28
Hemanshu pose la question :
Integral of X^2
Janice Cotcher lui répond.
Simultaneous equations 2008-07-23
Franco pose la question :
Solve

3 D + E - F = -10
-2 D - F = -4
-3 D - 4 E - F = -25

Franco

Penny Nom lui répond.
The maximum range of a projectile 2008-07-22
kwame pose la question :
the range R of projectile fired with an initial velocity Vo ,at an angle of elevation (@ )theta from the horizontal is given by the equation R = (Vo(squared) sin2theta)/g. where g is the accelation due to gravity . Find the angle theta such that the projectile has maximum range .
Harley Weston lui répond.
A square and a circle 2008-07-20
kobina pose la question :
4 ft of a wire is to be used to form a square and a circle. how much of the wire is to be used for the square and how much should be used for the square in order to enclose the maximum total area
Harley Weston lui répond.
Chords and arcs 2008-07-11
Ronnie pose la question :
We are trying to build a semi life size ark decoration , and we are trying to cut the sides out . The curved sides and we can't figure our radius , all we know is that our chord length is 24ft. any suggestions on how to find the radius or maybe even the arc length or circumference or diameter?
Harley Weston lui répond.
A difference quotient 2008-07-10
Rita pose la question :
Find the difference quotient of f, that is, find [f (x + h) - f (x)]/h, where h does not = 0 for the given function. Be sure to simplify.

f(x) = 1/(x + 3)

Janice Cotcher lui répond.
A dog tied to a round building 2008-07-08
maitham pose la question :
i have this question which i don't know how to solve it :

One dog was linked to the outer wall of a building round of 20 meters in diameter. If the length of chain linking the dog sufficient turnover of half the distance around the building, What area can guard dog?

they said that we can solve it by integral .. can you solve it for me?

Harley Weston lui répond.
CIRCLES 2008-07-07
daryl pose la question :
Find the equation of the smaller circle that is tangent to the axes and the circle x(squared)+y(squared)=2x+2y-1?
Penny lui répond.
If the arc is 75mm, what is the radius? 2008-06-12
malcolm pose la question :
If the are is 75mm, what is the radius?
Janice Cotcher and Harley Weston lui répond.
Two rhombi 2008-06-12
Malik pose la question :
("rhombus" diagram is attached) ABCD and EJCD are rectangles. EFHG and HIJK are similar rhombus. HIJK is 4times bigger than EFHG. In each rhombus the larger diagonal is double of the smaller one. If AB = 30cm, what is the lenght of the diagonals of the smaller rhombus ?
Penny Nom lui répond.
The rate of change in the depth of the water 2008-06-12
Liz pose la question :
A rectangular pool 50ft long and 30ft. wide has a depth of 8 ft. for the first 20 ft. for its length and a depth of 3 ft. on the last 20ft. of its length and tapers linearly for the 10 ft in the middle of its length. the pool is being filled with water at the rate of 3ftcubed/ min at what rate is the depth of the water in the pool increasing after 15 hours?
Harley Weston lui répond.
The radius of a sphere if you know the volume 2008-06-11
Cey pose la question :
how to i find the radius of a sphere with a volume of 1000cm cubed using the formula v=4/3 pi r squared???????/
Penny Nom lui répond.
Two circles 2008-06-10
cey pose la question :
the diameter of the larger circle is 20cm, and the smaller 10cm. what is the shaded area??
Janice Cotcher lui répond.
The length of a shadow 2008-05-27
Simon pose la question :
A figure skater is directly beneath a spotlight 10 m above the ice. IF she skates away from the light at a rate of 6m/s and the spot follows her, how fast is her shadow's head moving when she is 8m from her starting point? The skater is (almost) 1.6m tall with her skates on.
Stephen La Rocque and Harley Weston lui répond.
The weight of a concrete column 2008-05-11
russell pose la question :
a cylindrical form is filled with a slow curing concrete. The base of the form is 10 ft in radius, and height is 25 ft. while the concrete hardens, gravity causes the density to vary from a density of 90 lbs/ft^3 at the bottom to a density of 50 lb/ft^3 at the top. Assume that the density varies linearly from the top to the bottom, and compute the total weight of the resulting concrete column
Harley Weston lui répond.
A lidless box with square ends 2008-04-28
Chris pose la question :
A lidless box with square ends is to be made from a thin sheet of metal. Determine the least area of the metal for which the volume of the box is 3.5m^3. I did this question and my answer is 11.08m^2 is this correct? If no can you show how you got the correct answer.
Stephen La Rocque and Harley Weston lui répond.
At what value of t is the maximum acceleration? 2008-04-25
Mary pose la question :
Velocity of a function (which is the first derivative of its position) is defined over the interval 0 to 12 using the following piecewise function: v(t)=-1 from 0 to 4, v(t)=x-5 from (4 to 8 and v(t)=-x+11 from (8 to 12. At what value of t is the maximum acceleration?
Stephen La Rocque lui répond.
The radius of a circle 2008-04-25
kathy pose la question :
How do you find the radius of a circle if the area is 803.84 and using 3.14 for pi.
Penny Nom lui répond.
A volume of revolution 2008-04-24
Sabahat pose la question :
Hi, i have a region enclosed by both axes, the line x=2 and the curve y=1/8 x2 + 2 is rotated about the y-axis to form a solid . How can i find the volume of this solid?. (Please note that y equation is read as y =1 over 8 times x square plus 2.) I will be really grateful if you answer this question. :)
Harley Weston lui répond.
An open box 2008-04-23
Le pose la question :
Metal Fabrication; If an open box is made from a tin sheet 8 in square by cutting out identical squares from each corner and bending up the resulting flaps, determine the dimensions of the largest box that can be made.
Harley Weston lui répond.
What is the integral of 13sin^3(x)*cos^7(x)dx? 2008-04-22
Cathrine pose la question :
I am having trouble integrating this problem. It says to evaluate the integral but I don't know what to do or how to do it.

It is the integral of
13sin^3(x)*cos^7(x)d

Harley Weston lui répond.
f(x)=sin^3(3x^2) find f ' (x) 2008-04-21
Michael pose la question :
f(x)=sin^3(3x^2) find f ' (x)
Harley Weston lui répond.
The perimeter of a rhombus 2008-04-18
susana pose la question :
how do you find the perimeter of a rhombus?
Penny Nom lui répond.
The area bounded by 3 curves 2008-04-13
Sabahat pose la question :
Hi, I have enclosed a diagram.
The diagram shows the curve y=(2x-5)4. The point P has co-ordinates (4,81) and the tangent to the curve at P meets the x-axis at Q.

Find the area of the region (shaded in the diagram) enclosed between the curve, PQ and the x-axis . (Please note that the equation y is read as y=2x -5 whole raise to power 4.)

Stephen La Rocque lui répond.
f(x) =ax^blnx 2008-04-13
charles pose la question :
supposef(x) =ax^blnx is a real- valued function. Determine exact values(not decimal approximations) fro nonzero constants a and b so that the function f has a critical point at x=e^3 and a maximum value of 1/2e
Harley Weston lui répond.
A volume of revolution 2008-04-04
ted pose la question :
Consider the region bounded by y=x^2 + 1, y=5-3x and y=5. Sketch and shade the given region; then set up but dont evaluate teh integrals to find the following:

a) The volume of the solid generated by rotating the region about the line y=5

b) the volume of the solid generated by rotating the region about the y-axis

Penny Nom lui répond.
lim as x approaches infinite of 5x + 2/x-1 2008-04-04
Jordan pose la question :
how to solve this.

lim as x approaches infinite of 5x + 2/x-1

Stephen La Rocque and Harley Weston lui répond.
Finding the radius when only given chord length 2008-04-03
Lorraine pose la question :
There are two chords in a circle, an 8 inch chord and a 10 inch chord. The 8 inch chord is twice the distance from the center as the 10 inch chord. What is the radius?
Stephen La Rocque lui répond.
The integral of dx / (4x^2 - 25)^3/2 2008-04-01
Meghan pose la question :
I have a question from the trigonometric substitution of my calculus course.

integral of dx / (4x^2 - 25)^3/2

Harley Weston lui répond.
A max-min problem 2008-03-27
LSL pose la question :
show that of all rectangle with a given area, the square has the smallest perimeter.
Penny Nom lui répond.
The radius of a circle 2008-03-22
danny pose la question :
waht is the radius of a circle, if the circumference is 800?
Penny Nom lui répond.
A train and a boat 2008-03-15
Sabrina pose la question :
A railroad bridge is 20m above, and at right angles to, a river. A person in a train travelling at 60 km/h passes over the centre of the bridge at the same instant that a person in a motorboat travelling at 20km/h passes under the centre of the bridge. How fast are the two people separating 10s later?
Harley Weston lui répond.
The centre and radius of a circle 2008-03-12
Ryan pose la question :
hello and thank you for such a wonderful service. This problem I think needs to be checked could you take a gander at it and tell me if i get it correct thanks find the center and the radius of this circle x^2+y^2=8x-2y+15=0 I cam up with center -2, 1/2 and a radius of 11 3/4
Harley Weston lui répond.
What point on the graph y = e^x is closest to the origin? 2008-03-03
elvina pose la question :
What point on the graph y = e^x is closest to the origin? Justify your answer.
Stephen La Rocque lui répond.
Simultaneous equations 2008-02-29
CONOR pose la question :
I was wondering if you could help me with this problem

7x - 5y = -1
3y = 4x

Penny Nom lui répond.
The radius of a circle 2008-02-28
SteVonee pose la question :
Estimate the radius of a circle with the given circumference that is 192ft
Penny Nom lui répond.
I cut the cylinder at a 45 degree angle 2008-02-26
Shannon pose la question :
I have a cylinder with a radius of 2' 1 5/8". How do I calculate the radius increase when I cut the cylinder at a 45 degree angle?
Harley Weston lui répond.
A Norman window 2008-02-25
Jason pose la question :
If the perimeter of a Norman window is 20 feet, what is the maximum area of the window?
Stephen La Rocque lui répond.
A ball bearing is placed on an inclined plane 2008-02-15
Leah pose la question :
A ball bearing is placed on an inclined plane and begins to roll. The angle of elevation of the plane is x. The distance (in meters) that the ball bearing rolls in t seconds is s(t) = 4.9(sin x)t^2. What is the speed of the ball bearing, and what value of x will produce the maximum speed at a particular time?
Penny Nom lui répond.
Two regions with equal area 2008-02-13
James pose la question :
There is a line through the origin that divides the region bounded by the parabola y=3x-5x^2 and the x-axis into two regions with equal area. What is the slope of that line?
Harley Weston lui répond.
The circumference and radius of a circle 2008-02-10
Ray pose la question :
How do you find the circumference or radius of an area presuming it is a circle. Or in other words how do you find the c or r given only the area is 50 sq metres
Penny Nom lui répond.
Classifying a triangle 2008-02-07
kevin pose la question :
scalene triangle 8 ft base right side 9.5 left side 12 ft what is the angles
Penny Nom lui répond.
Integration by parts 2008-01-30
seth pose la question :
hi i really dont understand integr ation by parts. for example, the integral(t^2sintdt. i have u=t^2 and v'=sint also u'=t^/3 v=-cost for the formula i have uv-integralvu' dx this is all well and good but i cant get it right.
Harley Weston lui répond.
Belled-out pier 2008-01-28
Gina pose la question :
I need to know how to find the total yards needed to fill a concrete pier that is 54"/ 108" and 26' deep. That is...54" @ the top of the pier belled to 108" @ the bottom...26' deep.
Stephen La Rocque lui répond.
Inflection points 2008-01-25
Armando pose la question :
Hi, Im trying to write a program that takes an equation ( f(x) = 0 ) and returns a list of the inflexion points in a given interval. there must be (I think) a mathematical method or algorithm to do this, probably involving the (second) derivate of the function. However I have not found such a method yet. Any help on this will be much appreciated.
Stephen La Rocque and Harley Weston lui répond.
Finding the area of an isosceles triangle given one angle and the inradius 2008-01-24
Saurabh pose la question :
Given an isosceles Triangle, whose one angle is 120 and inradius is √3. So area of triangle is?
Stephen La Rocque lui répond.
A parallelogram and a rhombus 2008-01-22
miguel pose la question :
i have a problem proving a parallelogram a rhombus.. if a diagonal of a parallelogram bisects an angle of the parallelogram , then its a rhombus prove
Stephen La Rocque and Walter Whiteley lui répond.
Maximize income 2008-01-18
Chris pose la question :
Lemon Motors have been selling an average of 60 new cars per month at $800 over the factory price. They are considering an increase in this markup. A marketing survey indicates that for every $20 increase, they will sell 1 less car per month. What should their new markup be in order to maximize income?
Stephen La Rocque and Harley Weston lui répond.
The radius of a planet 2008-01-06
Ben pose la question :
Two people who are both h feet tall are standing on a spherical planet. One person walks a distance d in feet away from the the other person. At this point, the person walking turns around and can no longer see the top of the other persons head. What is the radius of the planet?
Stephen La Rocque and Harley Weston lui répond.
The integral of 1/ (x(x+1)^0.5) 2007-12-29
Nooruddin pose la question :
Integral of
dx / x(x+1)^0.5
(boundaries are 5 and 3)

Harley Weston lui répond.
Differentiate 2007-12-28
taiwo pose la question :
i am finding it difficult to use first principle to differentiate this question: y=xcos2x. can u help me.
Penny Nom lui répond.
lim sinx/(x +tanx) 2007-12-16
shimelis pose la question :
i have problem how do you solve this equation lim sinx/(x +tanx)
Harley Weston lui répond.
A 45-45-90 triangle 2007-12-13
Aaditya pose la question :
explain to me please how to do the 45-45-90 theorem when one of the legs(not the hypotenuse) is 3. How do you find the remaining two sides? please help me out.
Leeanne Boehm lui répond.
A right triangle 2007-12-06
Shubhomoy pose la question :
The co-ordinates of a hypotenuse are (1,3) and (-4,1). Find the equations of the perpendicular sides.
Harley Weston lui répond.
System of equations 2007-12-06
Jenn pose la question :
change the equation,x-y=4 to form y=mx+b the solution to the system of equations y=2x and y=-x+3 is
Stephen La Rocque lui répond.
Chicken and goat feet 2007-12-05
Kim pose la question :
Old McDonald raises goats and chickens. The animals have a total of 100 heads adn 360 feet. How many goats and how many chickens does Mr. McDonald have?
Stephen La Rocque and Penny Nom lui répond.
A radius and a tangent to a circle 2007-12-04
elizabeth pose la question :
show that the radius of a circle meets a tangent line to the circle in a 90 degree angle. hint: start by assuming they are not perpendicular and at a contradiction.
Penny Nom lui répond.
Proving a quadrilateral is a rhombus 2007-12-03
Jeanie pose la question :
How do you prove that a quadrilateral is a rhombus because the diagonals of the quadrilateral are perpendicular and bisect each other using the 2-column proof method?
Stephen La Rocque lui répond.
Maximize the product 2007-11-25
David pose la question :
Hi i have this site call calcchat.com, but i dont understand how they explained this can you take a look? The question is: Direction: Find two positive numbers that satisfy the given requirements. The sum is S and the product is a maximum this is what they did 1) Let x and y be two positive numbers such that x + y = S 2)P = xy 3) = x (S - x) 4) =Sx - x^2 5)...etc. the thing i dont get is how did they go from step 2 to step 3 and also i know this sound dumb but how did they get step 2? =)
Harley Weston lui répond.
A rectangular plot of farmland 2007-11-25
Christy pose la question :
A rectangular plot of farmland will be bounded on one side by a river and on the other three sides by a single-strand electric fence. With 800m of wire at your disposal, what is the largest area you can enclose, and what are its dimensions?
Harley Weston lui répond.
A curve sketch 2007-11-22
Ahson pose la question :
Find critical points, determine the monotonicity and concavity and sketch a graph of f(x) with any local maximum, local minimum and inflection points labeled:

1. f(x) = x^4 - x^3 - 3x^2 + 1

Harley Weston lui répond.
Elimination of mayan prisoners 2007-11-19
Jim pose la question :
An evil Mayan emperor decides to make an example of 1,000 prisoners. He stands them in a circle with numbers one to a thousand marked on their shirts. He then starts counting: "one in, two, in three out" The third man is immediately executed. This continues round and round and round the circle. While the number in the circle shrinks, every third prisoner is pushed out and executed. And it continues even when there are only two prisoners left alive. What number is on the last prisoner's shirt?
Victoria West lui répond.
A rectangle in an ellipse 2007-11-18
David pose la question :
I need to find the max area of a rectangle inscribed in an ellipse with the equation x^2+4y^2=4.. What I have so far is f(x,y)=4xy g(x,y)=x^2+4y^2-4=0,

y=sqrtx^2-4/4 f'(x)=2x^2/sqrt-4x^2+2(sqrt-4+x^2).

What I need to know is how to finish the problem and find the actual mas area of the rectangle. David

Penny Nom lui répond.
Find the radius of a circle given the center and a point on the circle 2007-11-18
Raymund pose la question :
Find the radius if the center is at (0, -5) and one point on the circle is (2,3)
Stephen La Rocque lui répond.
lim [x + squareroot(x^2 + 3)] as x->-inf 2007-11-16
David pose la question :
Find the limit. (Hint: treat the expression as a fraction whose denominator is 1, and rationalize the numerator.)

lim [x + squareroot(x^2 + 3)] as x->-inf

i got to

lim -3/(x - squareroot(x^2 + 3)) as x->-inf

but i'm having trouble understanding why the answer is 0 plz explain thx

Harley Weston lui répond.
Local maxima, minima and inflection points 2007-11-13
Russell pose la question :
let f(x) = x^3 - 3a^2^ x +2a^4 with a parameter a > 1.

Find the coordinates of local minimum and local maximum

Find the coordinates of the inflection points

Harley Weston lui répond.
Maximize his profit 2007-11-12
apoorva pose la question :
During the summer months Terry makes and sells necklaces on the beach. Last summer he sold the necklaces for $10 each and his sales averaged 20 per day. When he increased the price by $1, he found that he lost two sales per day.

a. Find the demand function, assuming it is linear.
b. If the material for each necklace costs Terry $6, what should the selling price be to maximize his profit?

Penny Nom lui répond.
Family of functions 2007-11-12
Russell pose la question :
Consider the family of functions
f(t)= Asin3t + Acos3t +Bsin8t + Bcos8t

find exact values of parameters A and B so that f(0) = 2 and f ' (0) = 1

Stephen La Rocque lui répond.
The radius of an arch 2007-11-10
Mark pose la question :
How do you determine the raduis or diameter of a circle based on the folowing information:
1. The distance along the circle between two points is 35'-2". This creates an arch.
2. The (chord) distance between the two points is 30'-8".
3. The distance from the center of the chord (on a 90 degree) to the arch is 6'-10 3/4".

Harley Weston lui répond.
Two integrals 2007-11-09
Akilan pose la question :
how to integrate these (tan x)^6(sec x)^4 and sinh(x)(cosh(x))^2.

Please send me how to do this question. Having exams on Monday. Please help.

Harley Weston lui répond.
Increasing and decreasing for functions 2007-11-09
David pose la question :
Direction: Identify the open intervals on which the function is increasing or decreasing.

f(x)=1/(x^2)
f'(x)= -2/(x^3)

i understand how to get up until there, and the undf. is x=0, but now i'm having problem setting up the number table chart. i cant remember how, and where to place the increase and decrease + - the chart, for example <---------0----------> where would the increase and the decrease be place?

Harley Weston lui répond.
f(x+y) = f(x) + f(y) + 2xy 2007-11-01
Marcia pose la question :
For all real numbers x and y, let f be a function such that f(x+y) = f(x) + f(y) + 2xy and such that the limit as h -> 0 of f(h) / h = 7, find: f(0), use the definition of the derivative to find f'(x), and find f(x).
Penny Nom lui répond.
How to solve related rates problems 2007-10-27
David pose la question :
Can you plz explain how and where you come up with an equation to solve this?
Find the rate of change of the distance between the origin and a moving point on the graph of y = sin x if dx/dt = 2 centimeters per second.

Stephen La Rocque lui répond.
lim x->1 (root x - x^2)/{1 - root x) 2007-10-16
Meghan pose la question :
Hi! I have a question from my Calculus textbook that I've been picking at for a while and I'm stuck.

lim x->1 (root x - x^2)/{1 - root x).

Stephen La Rocque and Penny Nom lui répond.
How do you find the radius of a circle if you only know its area 2007-10-15
s pose la question :
how do you find the radius of a circle if you only know the area of the circle. Do you somehow reverse the Pi formula.
Penny Nom lui répond.
Four triangles in a square 2007-10-15
Kristina pose la question :
A square with side lengths of 6 cm is divided into 3 right triangles and a larger isosceles triangle. If the three right triangles have equal area, find the exact area of the isosceles triangle.
Stephen La Rocque lui répond.
13 year and 17 year locusts 2007-10-12
stefan pose la question :
how many years pass between the years when both 13 year and 17 year locusts are out at the same time?
Penny Nom lui répond.
The average rate of change of a function 2007-10-11
vern pose la question :
Find the average rate of change of the function over the given interval. Compare this average rate of change with the instantaneous rates of change at the endpoints of the interval. f(X)=sinX for the inverval [0,pi/6]?
Harley Weston lui répond.
Substitution method 2007-10-11
Kevin pose la question :
3xx+2y=-36-y=11
Stephen La Rocque lui répond.
Given the arc length and chord length, what is the radius? 2007-10-10
Wayne pose la question :
I have the actual length of an arc plus the length of the cord. How do I determine the radius of the arc.
Harley Weston lui répond.
Parabolic suspension bridge 2007-10-09
Jessica pose la question :
A suspesion bridge with weight uniformly distributed along its length has twin towers that extend 75 meters abouve the road surfce and are 400 meters apart.The cables are parabolic in shape and are suspended from the tops of the towers. The cables touch the road surface at the center of the bridge. Find the height of the cables at a point 100 meters from the center. (Assume that the road is level.)
Stephen La Rocque lui répond.
Coin jar 2007-10-07
a student pose la question :
Sally empties his jar of coins. It contains $3.75 in nickels, dimes, and quarters. The number of dimes is twice the number of nickels and the number of quarters is three less than the number of nickels. Determine how many nickels, dimes, and quarters were in the in the jar.
Stephen La Rocque lui répond.
Solving four simultaneous equations (system of four linear equations) 2007-10-07
Johan pose la question :
I need some help in solving this question
x + 2y - 3z + 4w = 12
2x + 2y - 2z + 3w = 10
0 + y + z + 0 = -1
x - y + z - 2w = -4

Stephen La Rocque lui répond.
Arc lengths, central angles and radii 2007-10-04
Ashutosh pose la question :
Jose can remember that the length of an arc is 440cm, but he cannot remember the radius of the arc or the angle at the center. He does know that the angle was a whole number of degrees and the radius was less than 100cm. Find three possible angles and write down the size of each of the possible radii.
Stephen La Rocque lui répond.
How many ten thousands makes one million? 2007-10-02
Payton pose la question :
how many ten thousands makes one million?
Penny Nom lui répond.
Finding equations, intersection point of two lines at right angles 2007-09-22
Yaz pose la question :
Find the equation of the line joining A(-1,-9) to B(6,120). Another line passes through C(7,-5) and meets AB at rigth angle of D. Find the euation of CD and calculate the co-ordinates of D.
Stephen La Rocque lui répond.
The hypotenuse 2007-09-20
Kasey pose la question :
What is the hypotenuse of 96 squared and 108 squared?
Penny Nom lui répond.
How many two digit numbers contain at least one 7? 2007-09-06
Janet pose la question :
How many two digit numbers contain at least one number seven?
Penny Nom lui répond.
The area of a circle knowing only the length of a chord 2007-09-05
James pose la question :
I need some help in the right directions with a problem. I was presented with a problem where I need to find the area of a circle knowing only the length of a chord.

the is a circle in the center of a larger circle (which the size of either could change) the only thing that matter is that the chord is 100 ft long and rests on top of the smaller circle.

Stephen la Rocque and Brennan Yaremko lui répond.
The tangent to y = x^3 at x = 0 2007-09-04
Amit pose la question :
consider the equation = x^3. The equation of tangent to this curve (which is smmetrical in Ist and IVth quadrant) at (0,0) is y=0, which is x-axis. but graphically one can visulize that x-axis intersects the curve, so how can it be the tangent to the curve. Please help.
Harley Weston lui répond.
A frustum of a right pyramid 2007-08-24
Andrew pose la question :
Find the volume of a frustum of a right pyramid whose lower base is a square with a side 5 in., whose upper base is a square with a side 3in., and whose altitude is 12 in. Round your answer to the nearest whole number.
A. 47cu in.     C. 226 cu in.
B. 196 cu in.     D. 1036 cu in.

Stephen la Rocque lui répond.
Where do you use trigonometry? 2007-08-21
jenny pose la question :
where do you use trigonometry besides architecture and engineering?
Stephen La Rocque lui répond.
A geometry problem 2007-08-20
samhita pose la question :
ABC is a triangle. Let D be a point on side BC produced beyond B such that BD=BA. Let M be the mid-point of AC. The bisector of angle ABC meets DM at P. Prove that angle BAP=angle ACB.
Chris Fisher lui répond.
A right triangle 2007-08-11
Peter pose la question :
Ok, is it possible to find the height and base of a right triangle when all the information you are given is the length of the hypotenuse?
I also know that the angle between side b and the hypotenuse is 45 degrees. please help.

Walter Whiteley lui répond.
The swaying of a building in the wind 2007-08-11
San pose la question :
During a strong wind, a tall builing, such as the CN Tower, can sway back and forth as much as 100cm, with a period of 10 seconds. Please help me to determine the equation for this function, in the form y=asinkx
Stephen La Rocque lui répond.
Diameter of an octagon 2007-08-07
Bree pose la question :
I am trying to find the diameter of a octagon with 20' sides . What formula do I use?
Stephen La Rocque lui répond.
A complex number in polar form 2007-07-23
roland pose la question :
write the given complex number z in polar form lzl(p+qi) where lp + qil=1 for 3 - 4i.
Harley Weston lui répond.
f(x) = (x^4) - 4x^3 2007-07-22
Michael pose la question :
I'm a student who needs your help. I hope you'll be able to answer my question. Here it is: Given the function f(x)=(x^4)-4x^3, determine the intervals over which the function is increasing, decreasing or constant. Find all zeros of f(x) and indicate any relative minimum and maximum values of the function.
Any help would be appreciated. Thank you for your time.

Harley Weston lui répond.
A normal to a curve 2007-07-16
Samantha pose la question :
The function f is defined by f:x --> -0.5x^2 + 2x + 2.5
Let N be the normal to the curve at the point where the graph intercepts the y-axis. Show that the equation of N may be written as y = -0.5x + 2.5.
Let g:x--> -0.5x + 2.5
(i) find the solutions of f(x) = g(x)
(ii) hence find the coordinates of the other point of intersection of the normal and the curve

Penny Nom lui répond.
The isosceles triangle of largest area with perimeter 12cm 2007-07-16
sharul pose la question :
find the dimension of isosceles triangle of largest area with perimeter 12cm
Harley Weston lui répond.
Implicit Derivatives 2007-07-13
Charles pose la question :
I need help computing y' by implicit differentiation the question is: y^2 + x/y + 4x^2 - 3
Stephen La Rocque lui répond.
Derivative of a Function 2007-07-09
Bob pose la question :
What is the derivative of the function a sub n = [n/(n+1)]^n ?
Stephen La Rocque lui répond.
Finding the radius of an inscribed circle 2007-07-05
Maria pose la question :
I need to find the radius of a circle which is inscribed inside an obtuse triangle ABC. I know all the angles and all the lengths of the triangle.
Stephen La Rocque and Chris Fisher lui répond.
Using calculus to prove the formula for the area of a triangle 2007-07-04
Apratim pose la question :
Using calculus how can one show that the area of any triangle is 1/2 times its base times its height?
Stephen La Rocque lui répond.
A rhombus with all right angles 2007-06-29
Rachel pose la question :
Can a rhombus with all right angles still be tagged as a rhombus, or is it then said to be a square?
Stephen La Rocque, Harley Weston and Walter Whiteley lui répond.
A question on continuity 2007-06-28
Mac pose la question :
f(x) = (1/x) + (1/(2-x)) be the function and [0,2] be the interval.

1) It is continuous at the end points ?
2) is f(0) equal to f(2) ?

Harley Weston lui répond.
Log base 2 of log base 2 of x 2007-06-27
alex pose la question :
y = log base 2 of lag base 2 of x The slope of the tangent to the given curve at its x-intercept is..?
Harley Weston lui répond.
sin|x| and cos|x| 2007-06-25
Mac pose la question :
Can anyone tell me whether sin|x| and cos|x| is differentiable at x=0 ? As far as i know, cos(x) and sin(x) is differentiable at all x.
Penny Nom and Stephen La Rocque lui répond.
Limits as x approaches a constant 2007-06-25
Mac pose la question :
can you please tell me what is the reason they say "denominator is a negative quantity" in the solution 11 and "denominator is a positive quantity" solution 10 ??
If i guess correctly, for solution 10, its because of x^2 in the denominator.

Penny Nom lui répond.
Simultaneous equations : the Elimination method 2007-06-21
Patricia pose la question :
I need to find the value of X and Y using the Elimination method.

5/x + 3/y=4
25/x-2/y=3

Stephen La Rocque lui répond.
Simultaneous inequalities 2007-06-18
Freddy pose la question :
Watson Electric has production facilities in Valley Mills, Marlin,and Hillsboro. Each one produces radios,stereos,and TV sets. There production capacities are

Valley Mills: 10 radios, 12 stereos, and 6 TV sets per hour Marlin: 7 radios, 10 stereos, and 8 TV sets per hour Hillsboro: 5 Radios, 4 Stereos, amid 13 TV sets per hour

QUESTION

How many hours should each plant be scheduled to fill an order of 1095 radios, 1230 stereos, and 1490 TV sets?

Stephen La Rocque lui répond.
Area of a circular garden 2007-06-18
Cynthia pose la question :
Hi is this the correct formula for this problem? What is the approximate area of a circular garden that is 20 feet in diameters?
Stephen La Rocque lui répond.
Using the Pythagorean Theorem 2007-06-18
cynthia pose la question :
Hi, If I have a question with a right triangle and it asks.... If ABC is say 400 miles. How much shorter will the miles be if I travel from BC? I don't exactly remember the question but, I would I solve a problem similiar to this one?
Stephen La Rocque lui répond.
Angles of depression 2007-06-13
Phonda pose la question :
The pilot of a small private plane can look forward and see the control tower for a small airstrip. Beyond that is a large factory that is 3 milies from the airstrip. The angles of depression are 12.5 degrees and 4.8 degrees respectively. Find the airplane's altitude, to the nearest ten feet.
Stephen La Rocque lui répond.
Two tangent lines to y=x^3 2007-06-07
stephanie pose la question :
find the equations of two tangent lines to the y=x^3 function through the point (2,8)
Penny Nom lui répond.
The limit of a rational function 2007-05-28
Imad pose la question :
     3 _______ 3 _______
lim \/ 1 + x -    \/ 1 - x
x->0 ---- ----------------------------
              x

Penny Nom lui répond.
A circular blob of molasses 2007-05-28
Julie pose la question :
A circular blob of molasses of uniform thickness has a volume of 1 m^3. The thickness of the molasses is decreasing at a rate of 0.1 cm/hour. At what rate is the radius of the molasses increasing when the radius is 8 m?

Thanks,
Julia

Penny Nom lui répond.
More on quadrilateral shape names 2007-05-26
Don pose la question :
If North Americans call a quadrilateral with no parallel sides a trapezium, is a kite merely a special type of trapezium? Can a rhombus be a kite?
Walter Whiteley and Penny Nom lui répond.
System of equations 2007-05-24
Chris pose la question :
Find all real solutions (x,y,z,w) of the system of equations:
2y= x + x/17, 2z= y + y/17, 2w = z + z/17, 2x= w + w/17

Penny Nom lui répond.
Set up two simultaneous equations 2007-05-21
Admire pose la question :
The cost of producing windscreen wipers blades at a factory ais partly fixed (due to operating overheads) and partly dependent on the number of blades produced. It costs $300 to produce 1000, and $600 to produce 5000 blades. How much would it cost to produce 24000 blades?
Penny Nom lui répond.
Finding the hypotenuse without Pythagorus 2007-05-11
Shelbie pose la question :
How do i find the hypotenuse of a right traingle not using the pythagorean thereom if i have the measurements of the legs?
Stephen La Rocque lui répond.
Pattern for a truncated cone 2007-05-11
Mike pose la question :
I have been trying to get this cone flat so I can build this column. Can you please help me so I can figure this out? Thanks for your help.
Stephen La Rocque lui répond.
A ton of sawdust 2007-05-10
David pose la question :
I am trying to do a conversion. A tractor trailer is loaded with 165 cubic yards of sawdust (I do not know the actual weight). I knw the wholesale cost as $1,000 for this amount, but would like to convert this to find out what the equivalent cost for a metric ton would be.
Stephen La Rocque lui répond.
Area of region between circle and inscribed octagon 2007-05-07
amy pose la question :
I have to find the area of the shaded region where there is an octagon inscribed in a circle The radius is 4 inches. The shaded region is everything besides the octagon inside the circle. How can I find the area of the shaded region? Thank you!
Stephen La Rocque lui répond.
Optimization - carrying a pipe 2007-05-05
A student pose la question :
A steel pipe is taken to a 9ft wide corridor. At the end of the corridor there is a 90° turn, to a 6ft wide corridor. How long is the longest pipe than can be turned in this corner?
Stephen La Rocque lui répond.
Hypotenuse 2007-05-03
ashley pose la question :
how do you find the hypotenuse
Penny Nom lui répond.
Edging surrounding a round pool 2007-05-03
Carol pose la question :
Hello, I am new at this and very rusty on my math. I am getting a 24 Ft. round pool and would like to put 2ft width stone (small) edging around it. How much would I need to buy. I have to buy it by the yard (cubic yard) I am not looking for an exact, just an approx. even would be great. Thanks Carol
Stephen La Rocque lui répond.
Continuity of y = |x| 2007-05-02
moulipriya pose la question :
Is the curve y = | x | continuous everywhere?
Penny Nom lui répond.
Two concentric circles form an annulus 2007-05-02
A student pose la question :
In the diagram below, two concentric circles form an annulus. The vertical line is tangent to the inner circle, and forms the diameter of a third circle.

Explain why the areas of the annulus and third circle are the same.

Penny Nom lui répond.
A thousand, is it M or K? 2007-05-01
Larry pose la question :
I have heard that Million is annotated as MM. But Ihave heard two answers for Thousands (K, M). Which is correct?
Penny Nom lui répond.
A tugboat's speed 2007-04-30
Amanda pose la question :
a tugboat must travel 24 miles against a 4 mile per hour current on the Potomac River and return. At what constant speed must the tugboat travel to make the trip in 12 hours. Round answer to the nearest tenth mph.
Stephen La Rocque lui répond.
The area of a pyramid 2007-04-28
Alexander pose la question :
Total area of the plate required to fabricate a vessel(pyramid) the base is 0.6mx0.6m and height of 1.0m.
Stephen La Rocque lui répond.
Maximize the volume of a cone 2007-04-27
ashley pose la question :
hello, I've been stumped for hours on this problem and can't quite figure it out. The question is: A tepee is a cone-shaped shelter with no bottom. Suppose you have 200 square feet of canvas (shaped however you like) to make a tepee. Use calculus to find the height and radius of such a tepee that encloses the biggest volume. Can you help??
Stephen La Rocque and Penny Nom lui répond.
A cylinder inside a sphere 2007-04-25
Louise pose la question :
i need to find the maximum volume of a cylinder that can fit inside a sphere of diamter 16cm
Penny Nom lui répond.
Liquid is being poured into the top of a funnel 2007-04-19
neroshan pose la question :
Liquid is being poured into the top of a funnel at a steady rate of 200cm^3/s. The funnel is in the shape of an inverted right circular cone with a radius equal to its height. It has a small hole at the bottom where the liquid is flowing out at a rate of 20 cm^3/s. How fast is the height of the liquid changing when the liquid in the funnel is 15 cm deep? At the instant when the height of the liquid is 25cm, the funnel becomes clogged at the bottom and no more liquid flows out. How fast does the height of the liquid change just after this occurs?
Penny Nom lui répond.
Minimum cost for a fixed volume 2007-04-18
James pose la question :
My question goes: A silo is to be constructed and surmounted by a hemisphere. The material of the hemisphere cost twice as much as the walls of the silo. Determine the dimensions to be used of cost is to be kept to a minimum and the volume is fixed.
Penny Nom lui répond.
Simultaneous equations 2007-04-16
kyrie pose la question :
simultaneous equation 4x + 3y = 21 2x * y = 8
Penny Nom lui répond.
The second derivative 2007-04-14
Gerry pose la question :
In mathematical context,what do you understand by the term "Second Derivative"
Penny Nom lui répond.
An arc shaped groove into a peice of metal 2007-04-12
daniel pose la question :
hello i work at an engineering workshop the other night i was asked to machine an arc shaped groove into a piece of metal the cord length was 6 mm and the height from the middle of the cord to the arc was 1mm i was hoping to find the diameter of the cutter needed to do the job and also the formula to work out how to find the diameter. i believe it is 10mm dia thankyou for your time and knowledge
Stephen La Rocque lui répond.
Pythagoras was right 2007-04-11
Vineet pose la question :
in a right angle triangle, hypotenuse side is less than the sum of other two sides, how the square of hypotenuse is equal to the sum of squares of other two sides?
Stephen La Rocque lui répond.
What is the limit of 3.x^(3/x) as x approaches +infinity? 2007-04-11
Teodora pose la question :
What is the limit of 3.x^3/x as x approaches +infinity ?
Haley Ess lui répond.
Find the volume of the solid 2007-04-07
tricia pose la question :
a solid is constructed so that it has a circular base of radius r centimeters and every plane section perpendicular to a certain diameter of the base is a square, with a side of the square being a chord of the circle. find the volume of the solid at first i thought the length of a side of the square would be r, but that isn't awlays be true- only when the chord is in the center. so how can i solve this without any values? i dont understand the relationship between the chord and radius, except that the radius intercepts the chord at the midpoint. i know i hav to take the integral to get the volume, but how do i even find the area of one of the squares? please help, thanks, tricia
Penny Nom lui répond.
What is the hypotenuse of a right traingle 2007-04-04
debbie pose la question :
what is the hypotenuse of a right triangle with sides of 38 meters and 24.2 meters.
Stephen La Rocque lui répond.
A set of points in space 2007-04-04
Lenny pose la question :
What is a set of points in space the same given distance from its center point called?
Stephen La Rocque lui répond.
A beam on a lighthouse 2007-03-28
Lisa pose la question :
A beam on a lighthouse 2000 metres away from the nearest point P on a straight shoreline revolves at the rate of 10 pi radians per minute. How fast is the beam of the light moving along the shoreline when it is 500 metres from P?
Stephen La Rocque lui répond.
The foci of an ellipse 2007-03-27
Brad pose la question :
I am trying to figure out how to find the foci of an ellipse x^2/7 + y^2/16 = 1. Since 16 is the largest denominator I know the major axis is going to be the y axis. Do I now take 7-c^2=16. c^2=16-7, c^2=9, c=3. So is my foci (0,+-3).
Penny Nom lui répond.
y = sin(2x) 2007-03-22
bader pose la question :
sin(2x) find dx/dy
Penny Nom lui répond.
A rhombus 2007-03-04
Sally pose la question :
As a kindergarten teacher, I am trying to introduce the term rhombus to my class. What would be the best mathematical, but simple language to use? The diamond shape, which I am trying to label as rhombus, is still "a dimaond" to my kindergarten students. Help!
Steve La Rocque and Diane Hanson lui répond.
Mutually exclusive events 2007-03-01
kalyssa pose la question :
will you me an example of two events that are mutually exclusive and could you explain to me what mutually exclusive means?
Steve La Rocque, Pam Fowler and Penny Nom lui répond.
Simultaneous equations with fractions 2007-02-28
Alyca pose la question :
Hello Math Central, I am a grade 10 student taking Academic math. Our unit right now is method of substitution and elimination. I'm stuck on this one question that I've been doing forever. Please help =)

*For this equation I have to do method of elimination, but it's so much harder with fractions...could some one please explain to me how to do it step by step?*
x y 2
-- - -- = - --
3 6 3

x y 1
-- - -- = 1---
12 4 2

Steve La Rocque and Ashley Mang lui répond.
At what rate is the area of the triangle changing? 2007-02-24
mac pose la question :
two sticks 3.5 feet long are hinged together and are stood up to form an isosceles triangle with the floor. The sticks slide apart, and at the moment when the triangle is equilateral, the angle is increasing at the rate of 1/3 radian/sec. At what rate is the area of the triangle increasing or decreasing at that moment?
Mac

Penny Nom lui répond.
Find the area of the triangle 2007-02-20
Christina pose la question :
Graph the function f(X)= x+1/x-1 and graph the tangent line to the function at the points A:(2,3) and B:(-1,0). The point of intersection of the two tangent lines is C. Find the area of the triangle ABC.
Stephen La Rocque lui répond.
Optical illusions 2007-02-18
Jami pose la question :
Hi, I'm Jami and I'm in 10th grade.I'm doing a geometry research project on optical illusions and need to know how math is involved.I have an idea already of how our eyes percept 2 dimensional images and construct them into 3D images but, that isn't my question.There are many books that contain optical illusion pictures that have secret messages in them or have objects popping out. Is there a mathematical way in that optical illusion pictures are constructed?
Walter Whiteley and Harley Weston lui répond.
An augmented matrix 2007-02-13
Mary pose la question :
I've been trying for quite some time now to figure this out. I have to solve this by using the Gauss-Jordan Method: 3x - y = 15 2x + 3y = 10 Can anyone help me?
Penny Nom and Gabriel Potter lui répond.
Exponential form of complex numbers 2007-02-12
Austin pose la question :
When dealing with imaginary numbers in engineering, I am having trouble getting things into the exponential form. The equation is -1+i now I do know that re^(theta)i = r*cos(theta) + r*i*sin(theta). Just not quite understanding the order of operations. Thanks
Penny Nom lui répond.
Volume of an inner tube 2007-02-10
Bubba pose la question :
For a science project, I'm collecting methane gas in an inner tube. In addition to measuring psi of the inner tube, I'd like to calculate the volume of gas collected. What is the formula for or how would I calculate the volume of an inner tube? I appreciate any help you can give me.

Thanks so much. Bubba

Penny Nom lui répond.
Simultaneous equations with envelopes 2007-02-08
Mick pose la question :
There were 17 envelopes bought, some were brown, some were white. The brown envelopes cost one cent more per envelope than the white ones. The total cost was 80 cents. How much of each type of envelope was bought? --Many thanks!
Stephen La Rocque lui répond.
The substitution method 2007-01-31
Victoria pose la question :
how do i solve this problem using the substitution method?
2x-5= -14
-7x+14y= -5

Stephen La Rocque lui répond.
The elimination method 2007-01-31
Addrianna pose la question :
x-2y=2
3x-5y=7

Stephen La Rocque lui répond.
The centre and radius of a circle 2007-01-27
A student pose la question :
x^2+y^2=121 is the equation of the Circle C

(1) Write down the center and the radius of C.

Stephen La Rocque lui répond.
How many locations for the lampposts are possible? 2007-01-21
Madeeha pose la question :
Maria's backyard has two trees that are 40 feet apart, as shown in the accompanying diagram. She wants to place lampposts so that the posts are 30 feet from both of the trees. Draw a sketch to show where the lampposts could be placed in relation to the trees. How many locations for the lampposts are possible?
Penny Nom lui répond.
The volume of a frustum of a pyramid 2007-01-17
Sam pose la question :
Find the volume of a frustum of a pyramid with square base of side b, square top of side a, and height h.
Penny Nom lui répond.
An arc, a cord and the radius of a circle 2007-01-14
Kevin pose la question :
I have the length of the cord and the distance from the cord to the arc, is it possible to find the radius with just these parameters?
Penny Nom lui répond.
An octagonal bird house 2007-01-13
Soren pose la question :
I'm in the process of building a birdhouse that is an octagon (based on previous questions, looks like that's a familiar tune). The essential elements are known, but I get stuck when trying to determine the angle for the cuts that would be made to the thickness of the wood so that they all fit together when assembled. Each octagonal section is 7 inches in width and the peak of the roof will be 2 inches higher than the sides. My sense is that the angle cuts that need to be made to the 'height' of each piece of wood. By height I mean the thinnest part of the wood that is neither the length nor the width to use colloquial terms. While it's clear that a slight angle is needed, it would seem that the angle would necessarily change as the distance from the top of any one side to the peak changes. Please advise if more clarification is needed. The 2 inches is random and can be changed if more convenient. Whew!
Harley Weston lui répond.
Integrate x^8 (x^8 + 2)^2 ((x^8 + 2)^3 + 1)^4 2007-01-09
James pose la question :
How do you integrate x^8 (x^8 + 2)^2 ((x^8 + 2)^3 + 1)^4
Penny Nom lui répond.
What are the dimensions of the most economical container? 2007-01-04
Ashely pose la question :
A cylindrical container costs $2.00 per square foot for the sides and $3.00 a square foot for the top and bottom. The container must hold 100 cubic feet of material. What are the dimensions of the most economical container.
Stephen La Rocque lui répond.
An octagonal birdhouse 2006-12-30
Verner pose la question :
I am building a octagon birdhouse,what degree would I cut each side of each piece of wood to assemble the birdhouse?
Penny Nom lui répond.
A rhombus 2006-12-26
Jose pose la question :
show mathematically that a quadrilateral whose vertices are A(2,1),B(6,-2) C(10,1),and D(6,4) is a rhombus
Penny Nom lui répond.
Rolle's Theorem 2006-12-07
Erika pose la question :
If f(x) = (x^2)(square root of [3-x]) on the interval [0,3] is given, Does Rolle's Theorem apply? If yes, find any values of c such that f '(c)=0
Penny Nom lui répond.
A Norman window 2006-11-30
Joe pose la question :
a norman window is a rectangle with a semicircle on top. If a norman window has a perimeter of 28, what must the dimensions be to find the maximum possible area the window can have?
Stephen La Rocque lui répond.
The radius of a hemisphere 2006-11-29
Emma pose la question :
how do you calculate the radius of a hemisphere when you are given the volume?
Stephen La Rocque lui répond.
The radius of an arch 2006-11-15
Kelly pose la question :
I am trying to achieve an arc height of .375 on the length of 17.375.
Penny Nom lui répond.
Tangent lines 2006-11-09
Melissa pose la question :
let f be a function with f(1)=4 such that for all points (x,y) on the graph of f the slope is given by (3x^(2)+1)/(2y)

a.)Find the slope of the graph of f at the point where x=1.
b.)Write an equation for the line tangent to the graph of f at x=1 and use it to approximate f(1.2)
c.) Find whether f is concave up or concave down when x=1. Is your answer in part b an overestimate or an underestimate?

Stephen La Rocque lui répond.
Simultaneous equations 2006-11-06
An other pose la question :
e^2y-x+2=0
ln(x+3)-2y-1=0

Penny Nom lui répond.
Water is being pumped into the pool 2006-10-24
Jon pose la question :
A swimming pool is 12 meters long, 6 meters wide, 1 meter deep at the shallow end, and 3 meters deeps at the deep end. Water is being pumped into the pool at 1/4 cubic meters per minute, an there is 1 meter of water at the deep end.

a) what percent of the pool is filled?

b) at what rate is the water level rising?

Stephen La Rocque lui répond.
An approximation 2006-10-22
Ellen pose la question :
consider the curve -8x^2 +5xy+y^3 +149 =0 Write an equation for the line tangent to the curve at (4, -1) use this equation to approximate the value of K at the point (4.2, K)
Penny Nom lui répond.
How fast is the water level rising when the water is 1 meter deep? 2006-10-19
Don pose la question :
The cross section of a 5-meter trough is an isosceles trapezoid with a 2-meter lower base, a 3-meter upper base and an altitude of 2 meters. Water is running into the trough at a rate of 1 cubic meter per minute. How fast is the water level rising when the water is 1 meter deep?
Stephen La Rocque lui répond.
The hypotenuse 2006-10-02
Ashley pose la question :
How do you find the hypotenuse of a right triangle? I don't understand how to find c.
Stephen La Rocque lui répond.
The focus of a parabola 2006-10-01
Lily pose la question :
I have a mathematical assignment which includes applications of parabolas, hyperbolas and ellipses in the real world. I have been searching the internet and now I am ware that most of the applications of parabolas have a connection with what people call "the focus". However, I do not think I clearly understand what "the focus" of a parabola is. Would you please explain it to me?
Penny Nom lui répond.
The area of a rhombus 2006-09-10
Lillian pose la question :
In a rhombus, each side is 14 in. long. Two of the sides form a 60 degree angle. Find the area of the rhombus. Round your answer to the nearest square inch.
Stephen La Rocque lui répond.
The radius of a cone 2006-09-08
Hermanson pose la question :
I know the cone is 20 degrees at the top and 80 degrees at the bottom. What is the formula for finding the radius?
Stephen La Rocque lui répond.
The velocity of a pendulum, part II 2006-09-07
Erin pose la question :
We saw the question in your database about the velocity of a pendulum swinging.....It is the same exact question....but there is another question......it says....

"estimate the instantaneous rate of change of d with respect to t when t = 1.5. At this time, is the pendulum moving toward or away from the wall? Explain."

Harley Weston lui répond.
How many thousands are in ten million. 2006-08-24
Echoe pose la question :
How many thousands are in ten million.
Stephen La Rocque lui répond.
Differentiate Y= sin3x + cos7x 2006-08-22
james pose la question :
Differentiate the function of x using the basic rules.

Y= sin3x + cos7x

Stephen La Rocque lui répond.
How fast is the water level rising 2006-08-12
Erin pose la question :
Water runs into a conical tank at the rate of 9ft3/min. The tank stands point down and has a height of 10 ft. and a base radius of 5 ft. How fast is the water level rising when the water is 6 ft. deep? (V=1/3 pi r2 h).
Penny Nom lui répond.
An Integral 2006-07-30
Aniket pose la question :

I am Aniket studing in 12 th standard At Mumbai
I have following integration problem please give me a solution

integral of 1/under root of (5x2 - 2x) dx


Penny Nom lui répond.
Find the radius knowing the chord length and... 2006-07-28
Jim pose la question :
If I know the length of a chord and its distance from the diameter, how do I calculate the radius of the circle?
Stephen La Rocque lui répond.
Minimizing a cost 2006-07-25
Edward pose la question :
The cost of running a car at an average speed of V km/h is given by c= 100 + (V2 / 75) cents per hour. Find the average speed (to the nearest km/h) at which the cost of a 1000 km trip is a minimum.
Stephen La Rocque lui répond.
The area of a house 2006-06-28
Michael pose la question :
I would like to know how to measure the area of a house?
Penny Nom lui répond.
Fahrenheit and Celsius 2006-06-12
Doris pose la question :
I just know how to do problems with fahrenheit or celcius. Could you show me step by step how to do these? Then I can do it if you can show me each step of the way.
Stephen La Rocque and Penny Nom lui répond.
What is the sum of the first 100 whole numbers? 2006-05-31
Jo pose la question :
what is the sum of the first 100 whole numbers?
Natasha Glydon, Paul Betts and Penny Nom lui répond.
Simultaneous Equations 2006-05-24
Angie pose la question :

Question: solve the equations
2x-3y-z=0
3x-2y+z=-5
x+3y-2z=14

for x,y,z


Stephen La Rocque lui répond.
differentiate the volume of a cylinder with V respect to h 2006-05-24
A student pose la question :
differentiate the volume of a cylinder with V respect to h
Stephen La Rocque lui répond.
integral of tan^4 x 2006-05-14
Aqil pose la question :
integral of tan4 x
Penny Nom lui répond.
How many thousands make 1million? 2006-05-10
Raj pose la question :
How many thousands make 1million?
Penny Nom lui répond.
Rate of ladder falling 2006-04-30
Harsh pose la question :
A ladder 4 m long rests against a vertical wall. If the bottom of the ladder slides away from the wall at a speed of 30 cm/s, how quickly is the top of the ladder sliding down the wall when the bottom of the ladder is 2 m from the wall?
Stephen La Rocque lui répond.
School bus reliability - a probability question 2006-04-27
Peggy pose la question :
The school bus arrives at Janet's stop on time on 75% of school mornings. What is the probability it will arrive on time each day in a 5-day week?
Stephen La Rocque lui répond.
Three circles inside a larger circle 2006-04-16
Meghan pose la question :
Given three congruent circles tangent to one another (radii = 1), what is the radius of a circle circumscribed around them?
Stephen La Rocque lui répond.
2x+5y=3 And -x+3y=-7 2006-04-03
Lloyd pose la question :
simplify 2x+5y=3 And -x+3y=-7
Penny Nom lui répond.
The centre and radius of a circle 2006-04-02
Kaye pose la question :
I need to calculate Dimension E and F. I am given A, B, C, (or over all A+B+C), D, G. The radius is one continuous unknown radius. Example: A = 23.50
B = 35.50
C = 0.50
D = 11.50
G = 23.50
I have calculated this for angles but my mind is drawing a blank for the radius calculation. I can draw it but I need to put into Excel spreadsheet.

Harley Weston lui répond.
Find the point of inflexion for the curve y = e^x/(x^2-1) 2006-03-31
Sam pose la question :
Hi, i am trying to find the point of inflexion for the curve y = ex/(x2-1) and i got a really complex expression for y". I can't seem to solve x4-4x3+4x2+4x+3=0 so does that mean there is no point of inflexion?
Penny Nom lui répond.
A fence around a pen 2006-03-30
Daryl pose la question :
I hope you can help me out with the attached problem, It has been driving me crazy.
Stephen La Rocque and Penny Nom lui répond.
Can an equilateral triangle have an obtuse angle? 2006-03-26
Chris pose la question :
Can an equilateral triangle have an obtuse angle?

I'm thinking not, because all sides must be equal, but
does that also imply that all angles are equal?


Stephen La Rocque lui répond.
2x+3y=0 and 3x-y=0 2006-03-14
Lisa pose la question :
my name is lisa I am doing math for work and i have a math problem to solve and i need help with it here is the question 2x+3y=0 and 3x-y=0 this is one question can you help me please
Penny Nom lui répond.
A nine digit number 2006-03-06
Ryan pose la question :
What is the total number of possible combinations of a nine digit number (ie., social security number) including repeating numbers?
Stephen La Rocque and Penny Nom lui répond.
The path of a submarine 2006-02-26
Meadow pose la question :
Suppose that a submarine has been ordered to follow a path that keeps it equidistant from a circular island of radius r and a straight line shoreline that is 2 units from the edge of the island. Derive an equation of the submarine path, assuming that the shoreline has equation x = -p and that the center of the island is on the x-axis.
Penny Nom lui répond.
A locus problem 2006-02-08
Jason pose la question :
If the distance from p(1,5) is 3/4 that of the distance of a(4,-3)? find the equation of a locus.
Penny Nom lui répond.
The box of maximum volume 2006-02-01
Elizabeth pose la question :
A box factory has a large stack of unused rectangular cardboard sheets with the dimensions of 26 cm length and 20 cm width.
The question was to figure what size squares to remove from each corner to create the box with the largest volume.
I began by using a piece of graph paper and taking squares out. I knew that the formula L X W X H would give me volume. After trial and error of trying different sizes I found that a 4cm X 4cm square was the largest amount you can take out to get the largest volume. My question for you is two parts

First: Why does L X H X W work? And second, is their a formula that one could use, knowing the length and width of a piece of any material to find out what the largest possible volume it can hold is without just trying a bunch of different numbers until you get it. If there is, can you explain how and why it works.

Penny Nom lui répond.
how can i find the height of a triangle if i have the base and the hypotenuse 2006-01-27
Kelsey pose la question :
how can i find the height of a triangle if i have the base and the hypotenuse
Penny Nom lui répond.
The diameter of a pipe 2006-01-27
An other pose la question :

I know the base length of my arc (10 inches) - I also know the height at the center to the arc (2 inches). I don't think the end is at the midpoint tho. How do I figure out how long the arc length is?

My question involves being able to cut a round pipe into an arc that is 10 inches wide and 2 inches tall. I need to know the smallest diameter pipe to buy in order to fulfill these requirements.


Penny Nom lui répond.
Measuring an octagon 2006-01-26
Travis pose la question :
I am looking to do a project for work where I must find the radius of an octagon but I cannot directly measure it. I've found that on a regular hexagon I can find the radius by using the distance between the bolts to find the radius to the line connecting the bolts but also to the outside of a circle to cut it out. I do not understand however how this works for an octagon. What do I do to find the radius of an octagon with only the ability to measure the distance of the bolts? The center has a cutout in it and is mounted currently and I cannot get accurate measurements.
Penny Nom lui répond.
One boundary of a pond is parabolic in shape. 2006-01-20
Glenn pose la question :
One boundary of a pond is parabolic in shape. The boundary passes through the points A(-20,45), B(40,40) and E(30,35). The equation of the parabola is of the form y=ax2+bx+c. Find the equation of the parabola and the coordinates of the vertex of the parabola. Any assistance you could provide would be greatly appreciated.
Penny Nom lui répond.
Differentiation, powers and logs 2006-01-06
Claudia pose la question :

Question: how do I find the derivative of

x* ln(x+(e^2))^2

x^lnx

x^(e^(-x^2))


Penny Nom lui répond.
Extraneous solutions 2006-01-01
Liz pose la question :

Question: solve and check for extraneous solution

3(w + 1)1/2 = 6


Penny Nom lui répond.
Two related rates problems 2005-12-29
Shimaera pose la question :

#1. A manufacturer determines that the cost of producing x of an item is C(x)=0.015x2+12x+1000 and the price function is p(x)=250+2x/10. Find the actual and marginal profits when 500 items are produced.

#2. At 9 a.m a car is 10km directly east of Marytown and is traveling north at 100 km/h. At the same time, a truck leaves Marytown traveling east at 70 km/h. At 10 a.m, how is the distance between the car and the truck changing?


Penny Nom lui répond.
The Mean Value Theorem 2005-12-22
Candace pose la question :
Verify that the function satisfies the hypotheses of the Mean Value Theorem on the given interval. Find all numbers "c" that satisfy the Mean Value Theorem. 11. f(x)=3x2 + 2x +5 [-1, 1]
Penny Nom lui répond.
Simultaneous Equations 2005-12-21
Matt pose la question :
I have these two equations,
336 = 60a + 10b
and
432 = 84a + 6b
Am I right in saying both a and b are 4.8?

Penny Nom lui répond.
A max-min problem 2005-12-16
Julie pose la question :
A car travels west at 24 km/h. at the instant it passes a tree, a horse and buggy heading north at 7 km/h is 25 km south of the tree. Calculate the positions of the vessels when there is a minimum distance between them.
Penny Nom lui répond.
Mrs. Faria lives on an island 2005-12-15
Julie pose la question :
Mrs. Faria lives on an island 1 km from the mainland. She paddles her canoe at 3 km/h and jogs at 5 km/h. the nearest drug store is 3 km along the shore from the point on the shore closest to the island. Where should she land to reach the drug store in minimum time?
Penny Nom lui répond.
Inclusive definitions 2005-12-14
Layla pose la question :

recently the solvable quandary of 5+5+5=550 came up (the question says that you have to put 1 straight line somewhere in the equation to make it true with out turning the "=" into a "not=" sign).

So two answers were put forward:
545+5=550 (the use of a line converting a + into a 4)
AND
5+5+5(less than or equal to)550

There is currently an argument about the second solution. The disagreement is about whether this sign can be used. One person is arguing that the "less than or equal to" sign defines that the number on the left is in the range 550 and below. The other is saying that since the number (which is clearly defined with no variables) can never equal 550, then the "less than or equal to" sign cannot be used in this case.

Which one is the correct definition?


Walter Whiteley lui répond.
A point is twice the distance from y = 5 + 2x as it is from y = 5 - 2x 2005-12-09
Hazel pose la question :
A point moves so that its distance from the line y=5+2x is twice its distance from the line y=5-2x. Find the general form of the equation of its locus.
Penny Nom lui répond.
Four tangent circles 2005-12-06
Ananth pose la question :

I have one bigger circle A with radius 15.

Inside this bigger circle i have another circle B with radius 3 which touch this bigger circle. Have another circle C with radius 4 which touches A and B. I would like to draw a biggest circle which touches A,B and C.


Chris Fisher lui répond.
Notation for the second derivative 2005-11-08
Mussawar pose la question :
my question is d/dx( dy/dx) = d2y/dx2. why it is not equal to d2y/d2x.
Penny Nom lui répond.
Percent or percentage 2005-11-03
Kenneth pose la question :

Which word should be used in the following?

Change a (percent or percentage) to a decimal.

Should the word percent be used only when a number precedes it as in 45 percent?


Harley Weston and Chris Fisher lui répond.
Velocity and acceleration 2005-10-27
Candace pose la question :
When taking the integral of the position function, you get the velocity function, and the same for velocity to acceleration. So when you do each of these, you get a function. But when you integrate on a graph, you get an area under a curve. The area is un units squared- where do the units go when you make it an equation? How can a function be an area?
Harley Weston lui répond.
Can we take the derivative of independent variable 2005-10-18
Mussawar pose la question :
why we take derivative of dependent variable with respect to independent variable .can we take the derivative of independent with respect to dependent.if not why.
Walter Whiteley lui répond.
How would I find the length of the radius? 2005-10-15
Stace pose la question :
If given the length of a chord (121") and the distance from the midpoint of the arc to the midpoint of a chord (12"), how would I find the length of the radius?
Penny Nom lui répond.
Simultaneous equations 2005-10-13
Daniel pose la question :
5x + 3y = 22 4x - 7y = -20
Penny Nom lui répond.
Prove that a rhombus' diagonals are perpendicular 2005-10-02
Tania pose la question :
How do you prove that a rhombus' diagonals are perpendicular using the 2 column proof method?
Walter Whiteley lui répond.
U'(X) - U(X) = 0; U(0) = 2 2005-09-23
David pose la question :
Out of interest could you please answer the following questions?

U'(X) - U(X) = 0; U(0) = 2

and

U''(X) - U'(X) = 0; U'(0) = U(0) = 2


Harley Weston lui répond.
A point is moving on the graph of x^3 + y^2 = 1 in such a way that 2005-09-17
Gina pose la question :
A point is moving on the graph of x3 + y2 = 1 in such a way that its y coordinate is always increasing at a rate of 2 units per second. At which point(s) is the x coordinate increasing at a rate of 1 unit per second.
Penny Nom lui répond.
How do you differentiate y=(x)^(x^x)? 2005-09-14
Calebius pose la question :
How do you differentiate y=(x)(xx)?
Penny Nom lui répond.
At what rate is the circumference of the circle increasing? 2005-08-08
John pose la question :

A mathematics professor is knitting a sweater. The main part of the sweater is knit in a large spiral, ending up with a diameter of 30 inches. She knits at a constant rate of 6/7 square inches per minute.

1. At what rate is the circumference of the circle increasing when the diameter is 2 inches?

2. How long will it take her to finish this piece of the sweater?


Penny Nom lui répond.
What is the radius of this planet 2005-08-05
Kelly pose la question :
Assuming that a North-South line has been established, you set up two camps that are 67 miles apart. You now set up poles at each camp perpendicular to the ground. On a certain day at "noon" the pole at the South camp casts no shadow, while at the North camp a shadow is cast. The shadow makes an angle of 89 degrees with the horizontal. What is the radius, diameter, and circumference of this planet?
Chris Fisher lui répond.
The equation of an ellipse 2005-07-17
Allan pose la question :
I working on a problem that asks me to give the equation of an ellipse when only the location of the directrix and the length of the latus rectum are given. No other points on the ellipse are given. Again, the only "givens" are:

Length of latus rectum = 12
Location of directrix is x = 16

If I could determine the eccentricity, I could proceed from there by taking the ratio of the distance from a focus to the latus rectum point to the distance of the point from the directrix, but I lack the x coordinate of c. I've searched the text, and feel I've "missed something" somewhere! I note that the latus rectum segment is unique in one respect in that it is parallel to the directrix, where any other line segment on the ellipse to the focus would not be. Please indicate where I'm going wrong.

Chris Fisher lui répond.
A lighthouse is located on a small island,... 2005-07-14
Brittnee pose la question :
A lighthouse is located on a small island, 3 km away from the nearest point P on a straight shoreline, and its light makes four revolutions per minute. How fast is the beam of light moving along the shoreline when it is 1 km from P?
Penny Nom lui répond.
The volume of a hopper 2005-05-28
Brian pose la question :
I would like to know the volume of this rectangular hopper. can you help
Penny Nom lui répond.
Logarithmic differentiation 2005-05-23
Richard pose la question :
I need to convince myself that I understand the process of differentiating y=xx.
The specific question is that if I have to take the logarithm of both sides of the equation how can differentiate the following?
y= {(x+2)(x+2)}/{(x+1)(x+1)} - {(x+1)(x+1)}/(xx),
I have an idea that the differential of this fairly complex function is itself ... am I right or wrong.

Penny Nom lui répond.
L'hopital's rule 2005-05-15
Abraham pose la question :
Find the limit of [(1/(x+4))-(1/4)]/x as x approaches zero. How do you use l"hopital's rule to find this limit. I know how to do it with multiplying everything by 4(x+4), and getting the answer, -1/16.But how do you apply derivatives with l'hopitals rule to this type of problem?
Penny lui répond.
A Taylor series for ln(x) 2005-04-16
Anood pose la question :
i have to represent ln(x) as a power series about 2

i`m not getting the final answer which is ln 2+ sigma (((-1)(n+1)/ (n*2n))*(x-2)n). i don`t get the ln 2 part

i show you my trial

f(x)= ln x.

f-(x)=(1/x) .

f--(x)= (-1/x2)*1/2!

f---(x)= (2/x3)*1/3!

f----(x)= (-6/x4)* 1/4!

so the pattern shows me that f(n)= ((-1)(n+1))/xn *n)

so f(2)= sigma ((-1)(n+1))/2n *n) *(x-2)n

so as you see i don`t get ln 2


Penny Nom lui répond.
A torus and a sphere 2005-03-27
Tony pose la question :
Is it possible to shrink a torus into a sphere?
Andrei Volodin and Penny Nom lui répond.
Dimensions of a roof 2005-03-18
A roofer? pose la question :
A right triangle (roof of a house) has a base of 7 feet and a 22 degree angle. What is the height of the roof and what is the hypothenus of the triangle.
Penny Nom lui répond.
The square root of 2 2005-03-12
Madhumita pose la question :
From Pythagoras theorem we can draw square root 2 as a finite distance but it is irrational number which is endless. Explain how we can equate these two.
Harley Weston lui répond.
Each interior angle of a particular polygon is an obtuse angle... 2005-02-22
Victoria pose la question :
Each interior angle of a particular polygon is an obtuse angle which is a whole number of degrees. What is the greatest number of sides the polygon could have?
Walter Whiteley lui répond.
Extraneous solutions 2005-02-04
Heather pose la question :
My teacher wants to know why there are extraneous solutions in logarithms?
Penny Nom lui répond.
Solve for x 2005-02-02
Christie pose la question :
Solve for x

.387 = (.40 - .265x)/(sqrt(1-x2))

Penny Nom lui répond.
Differentiating F(x,y) = 0 2005-01-23
Jacob pose la question :
In calculus, we often mention to the students that if F(x,y) = 0, then we can differentiate both sides and still get an equality. The problem is that we can't perform the same operation on F(x) = 0, say x = 0, otherwise 1 = 0, which is absurd. What is the reason?
Walyer Whileley and Harley Weston lui répond.
The radius of a circle 2005-01-18
A student pose la question :
find the radius of a circle whose area is 1256sq cm.Use pi as an appoximation for pi.
Penny Nom lui répond.
A line from the center of the patch to the periphery 2005-01-01
Sandrine pose la question :
I am currently researching a patch disease of grasses. These patches are roughly circular. I need a term for a line from the center of the patch to the periphery. Since the patches are not perfectly circular, my supervisors tell me I cannot use the word 'radius'. What else could I use?
Denis Hanson and Harley Weston lui répond.
Three calculus problems 2004-12-09
Ashley pose la question :
Hi, I am having a lot of trouble with three calculus questions and was wondering if you could help
Penny Nom lui répond.
Implicit differentiation 2004-10-24
Emily pose la question :
If x^3+3xy+2y^3=17, then in terms of x and y, dy/dx =
Penny Nom lui répond.
A 40% increase in garage space 2004-10-20
Dianna pose la question :
A bus company recently expanded and no longer has enough room in its garage for all of its buses. Twelve of the buses have to be stored outside. If the company decides to increase their garage space by 40%, this will give them enough room for all of their current buses, plus enough room to store another twelve in the future. How many buses does the company own?
Penny Nom lui répond.
The hypotenuse of a right triangle 2004-09-20
Shannon pose la question :
I am trying to find the hypotenuse of a right triangle with only the length of the opposite side. What is the formula as I don't have the length of the adjacent side? Can I compute it without knowing what the other two angles are?
Penny Nom lui répond.
The length of a cut 2004-09-17
Florita pose la question :
My daughhter, who is a 9th grader is attempting to cut a piece of wood after determining the length of the cut for the hypotenuse. These are the measures: a=4squared, b=6squared.

She determined that c should equal 52. But when she measured the actual piece to be cut, c measured 39.5 inches! Can you offer any insight as to what she is doing wrong? I have suggested that she may be working with an Acute rather than a Right angle . But she insists that it is a Right angle after using a "framing square".

Claude Tardif lui répond.
1+3+5+...+(2n+1) 2004-09-10
Emma pose la question :
Prove that 1+3+5+...+(2n+1)= (n+1)2
Penny Nom lui répond.
The radius of a circle 2004-08-24
Peter pose la question :
If you slice any circle with a line, and call the distance of the line between intersections the "y" length and the perpendicular length to the shorter side of the curve the "x" length, what is the resulting equation for the radius?
Penny Nom lui répond.
A division symbol 2004-08-18
William pose la question :
i was wondering what the mathematical name for this division sign (÷).
Penny Nom lui répond.
The integrating factor method 2004-08-05
A student pose la question :
Whilst using the integrating factor method, I am required to integrate a function multipled by another function.

say f(t) = exp(kt) and some other function g(t); where exp = exponential and k is some constant.

Integral f(t)*g(t) dt or Integral exp(kt)*g(t) dt

What would the result of this integral be? I have never met an integral like this before. Would it simply be exp(kt)*g(t)/k? More specifically, the problem and my attempted answer is in PDF format:

In my attempted solution, I am unsure about the last two lines I have written out, as it relates to integrating a function multipled by another function.

Harley Weston lui répond.
Integrating e^sin(x) 2004-08-04
A student pose la question :
I need to know that how to solve the integral " e^sin x",
Harley Weston lui répond.
Differentiation 2004-08-04
A parent pose la question :
I am a parent trying to understand higher level of maths and would be very grateful if you could help with differentiating the following functions, identifying general rules of calculus:

a) f(x)=e^2^xIn(cos(8x))

b) f(x)=secx/SQRTx^4+1

Penny Nom lui répond.
Extraneous solutions 2004-07-28
Nicole pose la question :
When I have a problem like (2x + 3)/5 = (x + 1)/6 and the question asks to check for extraneous solutions, how do you solve that?
Penny Nom lui répond.
An Octagonal playhouse 2004-07-13
Levi pose la question :
I'm building an octagon playhouse for my son that is 8 feet wide. what would be the measurements of each of the eight sides.
Harley Weston lui répond.
The circle through three points 2004-07-06
Jim pose la question :
I am a student trying to solve math problem. I'd like to calculate the radius of the circle that exactly fits any three points. If the points are (X1,Y1), (X2,Y2), and (X3,Y3), what is the radius of the circle that contains those three points?
Penny Nom lui répond.
The sum of some positive integers 2004-06-07
A student pose la question :
Find the sum of all positive integers not greater than 10000 that are divisible by neither 3 nor 7.
Penny Nom lui répond.
Maximizing the angle to the goal mouth 2004-05-15
Yogendra pose la question :
You are running down the boundary line dribbling the ball in soccer or hockey. Investigate where in your run the angle the goal mouth makes with your position is at a maximum.
Penny Nom lui répond.
Three dice 2004-05-10
A student pose la question :
If one has 3, 6 sided dice what is the probability of the numbers that are rolled to total 4 through 10 inclusively?

Subsequent to this, what is the probability to do this consecutively...say 3 times?

Peny Nom lui répond.
Related rates and baseball 2004-04-26
Bethany pose la question :
A baseball diamond is the shape of a square with sides 90 feet long. A player running from second to third base at a speed of 28 feet/ second is 30 feet from second base. At what rate is the player's distance from home plate changing?
Penny Nom lui répond.
The problem of Apollonius 2004-04-25
Mitja pose la question :
There are given 2 circles lying one out of another and one point out of both circles. How to construct a circle passing through a given point and internally tangent to one and externally tangent to the other cirlce?
Chris Fisher lui répond.
A changing rectangle 2004-04-03
A student pose la question :
The width x of a rectangle is decreasing at 3 cm/s, and its length y is increasing at 5 cm/s. At what rate is its area A changing when x=10 and y=15?
Penny Nom lui répond.
Some calculus problems 2004-04-01
Weisu pose la question :

I have questions about three word problems and one
regular problem, all dealing with derivatives.

  1. Find all points on xy=exy where the tangent line
    is horizontal.
  2. The width x of a rectangle is decreasing at 3 cm/s,
    and its length y is increasing at 5 cm/s. At what rate
    is its area A changing when x=10 and y=15?
  3. A car and a truck leave the same intersection, the
    truck heading north at 60 mph and the car heading west
    at 55 mph. At what rate is the distance between the
    car and the truck changing when the car and the truck
    are 30 miles and 40 miles from the intersection,
    respectively?
  4. The production P of a company satisfies the
    equation P=x2 + 0.1xy + y2, where x and y are
    the inputs. At a certain period x=10 units and y=8
    units. Estimate the change in y that should be made to
    set up a decrease of 0.5 in the input x so that the
    production remains the same.

If you could just give me some hints on these
questions, I'd really appreciate it. Thanks!


Penny Nom lui répond.
A partial derivative 2004-03-19
Penny Nom pose la question :
Is it possible to differentiate the following equation, if so could you please explain.

S=SQRT(T-(5/X^2))

I would like the derivative of S with respect to X.

Harley Weston lui répond.
The radius of a circle 2004-03-06
A student pose la question :
what is the radius of a circle with the circumference of 12 inches?
Penny Nom lui répond.
Billions and more! and even more! 2004-03-01
Steph pose la question :
What comes after undecillion?
Penny Nom lui répond.
Rearranging an expression 2004-02-24
Corey pose la question :

use the following formula to answer the question F=9c divided by 5 plus 32

rearange the formula to solve for c
how many degrees fahrenheit would it be it it was:
0 degrees celcius
-10 degrees celcius
-46 degrees celcius


Penny Nom lui répond.
The derivative of x to the x 2004-02-14
Cher pose la question :
what about the derivative of x to the power x?
Penny Nom lui répond.
A pyramid-shaped tank 2004-02-13
Annette pose la question :
The base of a pyramid-shaped tank is a square with sides of length 9 feet, and the vertex of the pyramid is 12 feet above the base. The tank is filled to a depth of 4 feet, and water is flowing into the tank at a rate of 3 cubic feet per second. Find the rate of change of the depth of water in the tank. (Hint: the volume of a pyramid is V = 1/3 B h , where B is the base area and h is the height of the pyramid.)
Harley Weston lui répond.
Some trig problems 2004-01-18
Weisu pose la question :

I have some questions about pre-calculus.

(1) (2(cos(x))^2)+3sin(x)-1=0
(2) sin(x)cos(x)=(1/2)
(3) 3sin(x)=1+cos(2x)
(4) tan(x)*csc(x)=csc(x)+1
(5) sin(arccsc(8/5))
(6) tan(arcsin(24/25))
(7) arccos(cos(11pi/6))
the last problem uses radian measure.


Penny Nom lui répond.
Unusual occurances 2004-01-08
Martin pose la question :
My wife and I have a question about the probability of something that happened to us a few years ago. So far, no one has been able to give me even an approximate answer. On my 32nd birthday, my wife and I went out to eat at local Japanese hibachi style restaurant. At the restaurant, couples/families are sat together around the hibachi where the cook performs a show. There was a fifteen minute or so wait, so my wife and I sat in the lounge waiting for our name to be called. When they called our names for the reservation, this is what happened. The first group called was the Martin family. Then they called the Francis family. We were the next family to be called, the Ashton family. My full name is Martin Francis Ashton! I think the odds of that happening to someone are very unlikely, but it did, and there is more. Next, we were all sat at the same table in that order, "Martin" family, "Francis" family, then us, the "Ashton" family. Again, it formed my full name!
Penny Nom lui répond.
Business trip 2003-12-19
Ameer pose la question :
A businnessman drives from Washington, D.C., to Boston, a distance of 442 miles, and then makes the return trip. On the way to Boston, he drives 65 miles per hour, taking an 1-hour rest stop during the drive. After finishing his business in Boston, he make the return trip driving at 60 miles per hour and takes a 45-minute rest stop halfway through the trip. Which leg of the journey, Washington, D.C. to Boston, or Boston to Washington, D.C., takes the longer time?
Penny Nom lui répond.
A locus 2003-12-02
Tash pose la question :

Question:

a)Find the equation of the locus of the point P which moves so that its distance from A(1,2) is always three times its distance from B(5,6)

b) Show that this locus is a circle and states the coordinates of its centre and the length of its radius


Penny Nom lui répond.
A riddle 2003-11-19
Sarah pose la question :
Ok, our teacher gave us this riddle, and I cannot for the life of me figure it out. He said that there are three problems with the following proof:
Penny Nom lui répond.
Symmetries of a rhombus 2003-11-02
Tonia pose la question :
why cant an equal sided rhombus have 3 lines of symmetry? you have one line of symmetry on each of the diagonals, and there should be one vertically on an angle. can you please explain the rules of symmetry to me?
Walter Whiteley lui répond.
The sketch of a graph 2003-10-07
A student pose la question :
I was wondering how do you figure out if a graph has a horizontal tangent line. One of my homework problem was to sketch the graph of the following function; (4/3)x3-2x2+x. I set f''(x) ( the second derivative) of the function equal to zero and got the inflection point:(1/2,1/6). Also i am having trouble finding the concavity for x>1/2 and x<1/2, i am getting a different answer from the back of the book, the graph i draw looks completely different from the correct answer.
Penny Nom lui répond.
Indeterminate forms 2003-10-06
A teacher pose la question :
Is it possible for me to find any geometrical interpretation without using calculus to explain indeterminate forms?
Chris Fisher lui répond.
Functions, graphs and derivatives 2003-10-05
Jathiyah pose la question :
I wanted to know how would you tell (on a graph diplaying two funtions), which funtion is the derivative of the other?
Walter Whiteley lui répond.
The slope of a tangent 2003-10-01
A student pose la question :

find the slope of the tangent to each curve at the given point

f(x)=square root 16-x, where y=5


Penny Nom lui répond.
The mean house price 2003-09-10
Carol pose la question :

Question:
I have to find the mean from the following example:

Price Range £000 No of Houses
55 and under 60 3
60 and under 65 6
65 and under 70 13
70 and under 80 21
80 and under 100 15
100 and under 130 7
130 upwards 1

I know when calculating the mean you use the mid points of the classes, but how does this work for the 130 upwards class? Also, does this still work given the difference in the classes (ie. 1st class is 5, 5th class is 20, etc). Any help to get me started would be greatly appreciated.


Penny Nom lui répond.
A helicopter rises vertically 2003-09-02
Kate pose la question :
A helicopter rises vertically and t seconds after leaving hte ground its velocity is given in feet per second by v(t) = 8t + 40 / (t+2)2 How far above the ground will the helicopter be after 3 seconds?
Penny Nom lui répond.
Two precalculus problems 2003-08-04
Kate pose la question :

Please help me verify the identity:
cos2x(sec2x-1)=sin2x

Also I am having trouble withdetermining whether f(x) is odd, even, or neither
f(x)=x3-x


Penny Nom lui répond.
Natural logarithms 2003-07-22
Amanda pose la question :
I'm going into my senior year of high school. I will be taking AP calculus, and my teacher gave us some homework over the summer. However, there are two things that I do not understand how to do. The first is, she wants us to be able to generate a unit circle by hand using 30, 60 and 90 degree triangles. I have used the unit circle in trigonometry, however I was never taught how to draw it. Secondly, I need to know how to do natural logarithms without a calculator. I was not taught how to do this, and the worksheet I was given only showed me how to complete them using a calculator.
Claude Tardif lui répond.
Odd powers of sine and cosine 2003-06-25
Antonio pose la question :
Can you please tell me how to integrate a trig function involving sine and cosine? I know if the powers of both the sine and cosine are even and nonnegative, then I can make repeated use of the power-reducing formulas. But for the question I have on my hand, the powers of both sine and cosine are odd: ( sin3x + cos7x ) dx.
Harley Weston lui répond.
Circumference 2003-05-09
A parent pose la question :
Find the circumference use 3 1/7 for pi

1. r= 28 ft.
2. D=98 cm

Penny Nom lui répond.
Integrating e^x sin(x) 2003-05-03
Lech pose la question :
I am having trouble integrating the following expression by parts:

ex sin(x)

I used the integrator at http://www.integrals.com/ to find the solution,

? 1/2 ex cos(x) + 1/2 ex sin(x).

This is easy to confirm by differentiation, however I am confounded as how to arrive at the answer.


Penny Nom and Claude Tardif lui répond.
The volume of air flowing in windpipes 2003-05-02
James pose la question :
The volume of air flowing in windpipes is given by V=kpR4, where k is a constant, p is the pressure difference at each end, R is the radius. The radius will decrease with increased pressure, according to the formula: Ro - R = cp, where Ro is the windpipe radius when p=0 & c is a positive constant. R is restricted such that:
0 < 0.5*Ro < R < Ro,
find the factor by which the radius of the windpipe contracts to give maximum flow?

Penny Nom lui répond.
The square of my age was the same as the year 2003-04-14
Pat pose la question :
Augustus de Morgan wrote in 1864, "At some point in my life, the square of my age was the same as the year." When was he born?
Penny Nom lui répond.
Uses of conic sections 2003-04-01
William pose la question :
My name is William and I am doing a research paper on conic sections for my 12th grade math class. Part of the project is to find two conic sections in our world today and explain what there purpose is. I really need help in this area because I've been searching the internet for where conic sections are used in our world today and I really can't find anything. If you can tell me specific building or a pyramid that contains conic sections that would be great. Or even something in the universe would be helpful.
Leeanne Boehm lui répond.
A royal flush 2003-03-24
Vikki pose la question :

A poker hand consists of 5 cards selected randomly from an ordinary deck of cards: find the probability of a ROYAL FLUSH : the 10 , jack, queen,king and ace of the same suit.

I was thinking somewhere along the lines of:

*the number of ways to get the suit is 4C1
*the number of ways to get a 10 out of the 13 cards etc....
...but Im not sure I am going about this the right way, could you help?


Andrei Volodin lui répond.
Surface area of a sphere 2003-03-11
Kim pose la question :
a sphere has a surface area of 128 pi sq. units. What is its exact radius?
formula is 4 pi r2 I believe but how do I get radius

Penny Nom lui répond.
Can a square be a rhombus? 2003-03-04
Beth pose la question :
Can a square be a rhombus? Some sources say yes, some say no. Some sources define a rhombus as a quadrilateral and parallelogram with equal sides, but without right angles. Some sources say a square is a special case of a rhombus. Clarity, please!
Walter Whiteley lui répond.
Hundreds, thousands, millions,... 2003-02-19
Karissa and Jasmeen pose la question :
hundreds, thousands, millions, - can you help us with the rest of this sequence - we are trying to find the largest number
Penny Nom lui répond.
Wrap a rope around the equator 2003-02-12
Ali pose la question :
If you take a rope and wrap it around the equator and then overlap it 6ft. and cut off the extra then you loosen it up so the ends meet how far would it stand off the ground?
Penny Nom lui répond.
Extraneous solutions 2003-01-24
Paul pose la question :

What is an extraneous solution and in what cases do you get one?

How do you know it is extraneous?


Penny Nom lui répond.
Integration of 1/(2+cos(x)) 2003-01-07
A student pose la question :
integral from pi to 0 of

dx/(2+cos x)

i used the substitution t=tan(x/2) and i ended up with

integral from +infinity to 0 of

2dt/(t2+3)

which looks like an inverse tan function , and i ended up with sqr(27)/2 pi , which is not the same as my calculator's answer , so i suspct i am doing some thing wrong. can some one tell me where i am going wrong please.
Penny Nom lui répond.

Constructions of polygons 2003-01-03
Garrett pose la question :
Our teacher just finished the constructions unit, and he mentioned briefly about odd sided figures such as pentagons and septagons, only that they're very hard. My question is, how do you draw, with a compass and a straight edge, a pentagon and septagon?
Chris Fisher lui répond.
How many billions equal one trillion? 2002-12-07
Ryan and Aylah pose la question :
How many billions equal one trillion?
We believe that the answer is one thousand times one billion equals one trillion. Please help us with the answer to this question.

Penny Nom lui répond.
The length of an arc 2002-11-27
Nancy pose la question :
If all I have is the length between 2 ends of an arc (72"), how do I find the length of the arc at its apex and the radius?
Penny Nom lui répond.
Differentiating inverses 2002-11-20
Amy pose la question :
f(x)= x3+x+1, a=1 find g'(a) (g = f -1). I am having trouble finding g(a).
Penny Nom lui répond.
Round to hundredths 2002-11-19
Brittany pose la question :
Can u tell me how to do a problem like this:

35 divded by 4.8= Round to hundreths.

Penny Nom lui répond.
A bus is 60% occupied 2002-11-09
Joe pose la question :
A bus with a seating capacity of 60 people is 60% occupied. At the next stop one-third of the people get off the bus and 3 people get on the bus. The bus is now ___% occupied.
Penny Nom lui répond.
Mathematics and Music 2002-11-01
Hannah pose la question :
I am looking for a science fair project to compare math and music and how they relate. If you have any project ideas for me, they would be greatly appreciated.
Walter Whiteley lui répond.
How would you find the length of the chord? 2002-10-31
A draftsperson pose la question :
If given the length of an arc and the distance from the midpoint of the arc to the midpoint of a chord, how would you find the length of the chord and the radius of the arc. The chords endpoints are the same as the the arcs endpoints.
Penny Nom lui répond.
Nixon, Jefferson, and Madison 2002-10-08
Lisa pose la question :
The longest-lived US presidents are John Adams(age90), Herbert Hoover (also90), and Harry Truman (88). Behind them are James Madison, Thomas Jefferson, and Richard Nixon. The lattter three men lived a total of 249 years, and their ages at the time of death form consecutive odd integers. For how long did Nixon, Jefferson, and Madison live?
Penny Nom lui répond.
A max/min problem 2002-09-21
Evelina pose la question :
A window is the shape of a rectangle with an equilateral triangle on top. The perimeter of the window is 300 cm. Find the width that will let the maximum light to enter.
Penny Nom lui répond.
The entire earths' population would fit in the state of Texas 2002-09-18
Roz pose la question :
I have been told that the entire earths' population would fit in the state of Texas and each person would have 1/2 acre. Is this true.
Chris Fisher lui répond.
A Circle is evenly divided into six equal triangles 2002-09-16
Marilynn pose la question :
A Circle is evenly divided into six equal triangles leaving an area between the outside of the circle and the one side of the triangle. This area is measured as 3.14. What is the length of the radius, one line on the triangle?
Paul Betts lui répond.
Sums of evens 2002-09-14
Rosa pose la question :
How do I find a geometric way to easily compute sums of consecutive even numbers 2 + 4 + 6 + ....
Leeanne Boehm and Harley Weston lui répond.
The sum of the first one hundred even numbers 2002-09-10
Arthur pose la question :
What is the sum of the first one hundred even numbers?
Penny Nom lui répond.
Two equations 2002-07-26
Derek pose la question :
1. 3x + 2y = 4
2. -7x + 2y = 24

finding x and y.


Penny Nom lui répond.
Musical Scales 2002-07-24
Terence pose la question :
Given that there are 12 notes in a musical octave, what is the maximum number of musical scales possible within that octave, if each scale has a minimum of 5 notes and a maximum of 9 and we start all the scales from the same note?

In case you don't know anything about music, a scale is a progression of notes where you start on a specific note and end on that same note an octave higher. There are twelve different notes between these two similar notes. Which notes you choose to play determine the sound of the scale. Anything less than five notes would not make for a very interesting scale. Anything more than nine and you would be playing almost 'every' note in the scale, not leaving much room for distinction in how you organize these notes.

I assume you first have to figure out the maximum number of variations possible in a 5-note scale (with 12 notes at your disposal). Then do the same for a 6-note scale, then a 7-note, then an 8-note, and so on. Then add up the results. How to find this maximum number of variations for each scale size though is what I don't know.


Leeanne Boehm lui répond.
Integrating x^x 2002-06-18
Jeremy pose la question :
I am a student at the University of Kansas and I am wondering if there is a general anti-derivative for x x (i.e. the integral of x x dx)? I've looked in a bunch of Table of Integrals and have found nothing (can you guys find it?), so I'm sort of wondering if this may be a research type question.
Claude Tardif lui répond.
A good rule of thumb when driving 2002-06-13
Lisa pose la question :
A good rule of thumb when driving is that you should be about one car length away from the car in front of you for every 10 miles per hour that you are travelling. Suppose you follow this rule perfectly (so you are exactly the correct distance away). You are waiting at a stop light with your front bumper just touching the car in front of you. The light turns green and the car in front accelerates at a constant rate "r". Calculate how you should accelerate in order to follow the rule.
Penny Nom lui répond.
Overlapping circles 2002-05-29
Naman pose la question :
There are two circles, big circle with radius R and small one with radius r. They intersect and overlap in such a way that the common area formed is 1/2 pi r 2 (half the area of the small circle) If r=1, find the Radius of the big circle (R)?
Harley Weston lui répond.
A spotlight shines on a wall 2002-05-25
Barb pose la question :
A spotlight on the ground shines on a wall 12m away. If a man 2m tall walks from the spotlight toward the bldg at a speed of 1.6 m/s, how fast is his shadow on the bldg decreasing when he is 4m from the bldg?
Penny Nom lui répond.
What is Calculus About? 2002-05-13
A student pose la question :
I am a high school senior and have to write an essay answering the question "What is Calculus?" I need some ideas. Thanks
Walter Whiteley lui répond.
The law of cosines and obtuse angles 2002-05-09
Bryant pose la question :
The question that I am pondering is that I need to derive the law of cosines for a case in which angle C is an obtuse angle.
Penny Nom lui répond.
A rectangular marquee 2002-05-07
Alyaa pose la question :
a marquee with rectangular sides on a square base with a flat roof is to be constructed from 250 meters square of canvas. find the maximum volume of the marquee. i find this topic so hard
Harley Weston lui répond.
How will I use calculus in my career? 2002-05-06
Meridith pose la question :
How will I, hopefully a future secondary mathematics teacher, use calculus in my career if I'm not teaching calculus?
Walter Whiteley lui répond.
Arithmetic progressions 2002-04-24
David pose la question :
I have been searching everywhere for the formula to mathamatical progression.
Penny Nom lui répond.
Arc length 2002-04-17
Vix pose la question :
Find the point on the curve r(t)=(12sint)i-(12cost)j+5tk at a distance 13pi units along the curve from the point (0,-12,0) when t=0 in the direction opposite to the direction of increasing arc length.
Harley Weston lui répond.
Related rates 2002-04-17
Molly pose la question :
A tanker spilled 30 ft cubed of chemicals into a river, causing a circular slick whose area is expanding while its thickness is decreasing. If the radius of the slick expands at the rate of 1 foot per hour, how fast is them thickness of the slick decreasing when the area is 100 feet squared?
Penny Nom lui répond.
Pairs of equations 2002-04-04
A student pose la question :

high school level
student is asking

y=4x x=-4y 
x+y=5 3x+2y=20   

y=x-1 3x-y=4 
x+y=3 2x-3y=-9   

x+5y=4 
3x+15y=-1 

. . . 

Penny Nom lui répond.
Some 5 card hands 2002-03-28
A student pose la question :
From a standard deck of cards how many 5 card hands are possible consisting of a. exactly 4 hearts

b. two cards of one kind and three of another(like a full house).

Penny Nom lui répond.
The slope of a tangent line 2002-03-04
Ridley pose la question :
Suppose a function f(x) has the line 3x+4y=2 as its tangent line at x=5. Find f'(5).
Harley Weston lui répond.
The substitution method 2002-02-24
Joe pose la question :
whats the answer to this question? 3x+y=11
x+2y=-3

its substitution method i am having alot of trouble figuring it out. send the answer as soon as possible. thank you


Penny Nom lui répond.
Alfredos house number 2002-02-21
Aunt Patty pose la question :
Alfredos house number is between 20 and 35. The sum of the digits is less than 5. If you subtract 1 from it you would get a multiple of 3. If you add three, you get a multiple of 5. What is Alfredos house number.
Penny Nom lui répond.
Diameter of a pipe 2002-02-16
Landry pose la question :
I am trying to calculate the dia. of a pipe 60 inches long that will hold a gallon of water. What is the formula?
Penny Nom lui répond.
Getting to B in the shortest time 2001-12-19
Nancy pose la question :
A motorist in a desert 5 mi. from point A, which is the nearest point on a long, straight road, wishes to get to point B on the road. If the car can travel 15 mi/hr on the desert and 39 mi/hr on the road to get to B, in the shortest possible time if......

A.) B is 5 mi. from A

B.) B is 10 mi. from A

C.) B is 1 mi. from A


Penny Nom lui répond.
Simultaneous equations 2001-12-17
Matthew pose la question :
4x + y = 4
2x - 3y = 5

what is x and y


Penny Nom lui répond.
Two equations in two unknowns 2001-12-04
Courtney pose la question :
y = 3x + 2
y = 4x - 5

solve for x

Penny Nom lui répond.
3, 6, 10, 15, 21 2001-11-29
Patrick pose la question :
we are trying to find the expression to solve for the nth term in the pattern

3, 6, 10, 15, 21


Denis Hanson lui répond.
A lighthouse and related rates 2001-11-29
Melissa pose la question :
A lighthouse is located on a small island 3 km away from the nearest point P on a straight shoreline, and its light makes 4 revolutions per minute. How fast is the beam of light moving along the shoreline when it is 1 km from P?
Penny Nom lui répond.
A tangent line 2001-11-21
A student pose la question :
write an equation of the line tangent to the graph of

ey + ln(xy) = 1 + e at (e,1)

Harley Weston lui répond.
Gallons 2001-11-19
Shawn pose la question :
Do we use British or American gallons in Canada?
Chris Fisher lui répond.
Asymptotes 2001-11-09
Frank pose la question :

given the function:

f(x) = (x2) / (x-1)

the correct answer to the limit of f(x) as x approaches infinity is:

y = x+1

all math references point to this answer and the method they all use is long division of x-1 into x2

however if one were to multiply both the numerator and denominator by 1/x and then take the limit, one gets:

y=x

how can the descrepency between the two answers be explained?


Chris Fisher and Penny Nom lui répond.
A lighthouse problem 2001-11-02
A student pose la question :
A lighthouse at apoint P is 3 miles offshore from the nearest point O of a straight beach. A store is located 5 miles down the beach from O. The lighthouse keeper can row at 4 mph and walk at 3.25 mph.

a)How far doen the beach from O should the lighthouse keeper land in order to minimize the time from the lighthouse to the store?

b)What is the minimum rowing speed the makes it faster to row all the way?

Harley Weston lui répond.
Three problems 2001-10-28
Brenda pose la question :
  1. Joseph is planting bushes around the perimeter of his lawn. If the bushes must be planted 4 feet apart and Joseph's lawn is 64 feet wide and 124 feet long, how many bushes will Joseph need to purchase?

  2. The cost of a long distance phone call is $1.50 for the first two minutes and $0.60 for each additional minute. How much will Maria pay for a 24 minute phone call?

  3. Find the next three numbers in the pattern. 1,3,7,15,31,___,____,___.

Penny Nom lui répond.
Concavity of f(g) 2001-10-25
Troy pose la question :
Suppose f & g are both concave upward on (-infinity,infinity). Under what condition on f will the composite function h(x)= f(g(x)) be concave upward?
Walter Whiteley lui répond.
Tenths, hundredths, and thousandths 2001-10-17
Cheri pose la question :
I would like the French terms for the decimal place values of tenths, hundredths, and thousandths. (I would be interested in the Spanish terms also if you had them.)
Maria Torres and Claude Tardif lui répond.
Odd plus even is odd 2001-10-14
James pose la question :
Why is the sum of an odd number and an even number always odd?
Peny Nom lui répond.
Maximize the area 2001-10-13
Mike pose la question :

I have no clue how to do this problem. Here is what the professor gave to us:

A=LW

C=E(2L+2W) + I(PL)

Where P = # of partitions

E= cost of exterior of fence

I = cost of interior of fence

C = total cost of fence

.
.
.

Harley Weston lui répond.
4 sinx cosy = 1 2001-10-10
A student pose la question :
How would i differentiate the following example in terms of t (x and y are functions of t)

4 sinx cosy = 1


Claude Tardif lui répond.
Locusts 2001-10-08
A parent pose la question :
JOHN CONJECTURED THAT BOTH 13-YR.&17-YR. LOCUSTS CAME OUT THE SAME YEAR. ASSUME IT IS CORRECT. HOW MANY YEARS PASS BETWEEN THE YRS. WHEN BOTH 13 YR. & 17YR. LOCUSTS ARE OUT AT THE SAME TIME? EXPLAIN. NEXT, SUPPOSE THERE WERE 12YR.,14 YR.,& 16YR. LO- CUSTS,& THEY ALL COME OUT THIS YR. HOW MANY YRS. WILL IT BE BEFORE THEY ALL COME OUT TOGETHER AGAIN?
Leeanne Boehm lui répond.
The height of the lamppost 2001-10-02
Werner pose la question :
I am working on question 51,section 3.7 ,page 191 of Stewart's Single Variable Calculus. The question involves a lamp post which is casting a shadow around the eliipse whose formula is x2 + 4*y2 = 5. I have found the derivative of the elllipse both explicitly: x/4(((5-x2)/4)0.5) and implicitly : y' = - x/(4*y).
Harley Weston lui répond.
(x^2-5x-6)/(x-6) 2001-10-02
Bill pose la question :
given f(x) = (x2-5x-6)/(x-6) find f'(6).
Harley Weston lui répond.
1+2+3+...+1000 2001-10-01
Louise pose la question :
Find a quick way to add all Intergers (whole Numbers) between 1 and 1000?
Walter Whiteley lui répond.
Sharing a donut 2001-09-06
Amanda pose la question :
You have invited 11 people over to your house one day, and your friends are hungry. You go into the kitchen and find out you only have 1 donut (with a hole in the middle). In order to feed 12 people (including you), you must cut the donut into 12 pieces with only using a straight knife and cutting 3 times. This is NOT a trick question.
Claude Tardif lui répond.
The radius of a planet 2001-07-30
Jessica pose la question :
A satellite is orbiting the earth at an altitude of 100 miles. If the angle of depression from the satellite to the horizon is 50 degrees, what is the radius (to the nearest mile) of the planet?
Harley Weston lui répond.
The Mean Value Theorem 2001-07-23
Corrie pose la question :
I need to find if the mean value theorem exists. and if so, find all values c guaranteed by the theorem.

f(x) = |x2-25| on the interval [-10,0]


Harley Weston lui répond.
Rhombus 2001-07-16
William pose la question :
Calculate the internal angles of a rhombus given measurments of all four sides only.
Walter Whiteley lui répond.
Area between curves 2001-06-13
Phil pose la question :

question 1

find the area bound by the curves y = x2 + 2x + 3 and y = 2x + 4

question 2

Find the volume generated by rotating the curve x2 + y2 = 9 about the x-axis

Harley Weston lui répond.
Mutually exclusive 2001-06-05
Marje pose la question :
What does the mathmatical term "mutually exclusive" mean. Pleas diagram if possible.
Penny Nom lui répond.
Common solution 2001-06-02
Samantha pose la question :
  1. Solve for common solution: x+y=6 2x-3y=2

  2. Solve for y in terms of x: 3x-y=4

Penny Nom lui répond.
National consumption function 2001-05-09
Brian pose la question :
If consumption is $11 billion when disposable income is 0 and the marginal propensity to consume is dC/dy = 1/(2y+4)1/2+0.3(in billions of dollars), find the national consumption function.
Harley Weston lui répond.
Maximize profit 2001-05-09
Brian pose la question :
The marginal cost for a certain product is given by MC = 6x+60 and the fixed costs are $100. The marginal revenue is given by MR = 180-2x. Find the level of production that will maximize profit and find the profit or loss at that level.
Harley Weston lui répond.
The average value of a continuous function 2001-05-08
Esther pose la question :
The average value of a continuous function y = f(x) on the interval [a,b] is given by ________________?
Harley Weston lui répond.
A Taylor series 2001-04-27
Karan pose la question :
Given the following information of the function
  1. f''(x) = 2f(x) for every value of x

  2. f(0) = 1

  3. f(0) = 0
what is the complete Taylor series for f(x) at a = 0

Harley Weston lui répond.
Oil revenue 2001-04-21
Brian pose la question :
Suppose that t months from now an oil well will be producing crude oil at the rate of r(t), not a constant, barrels per month and that the price of crude oil will be p(t), not a constant, dollars per barrel. Assume that the oil is sold as soon as it is extracted from the ground.
  1. Find an expression for the total revenue from the oil well, R(t).

  2. A certain oil well that currently yields 400 barrels of crude oil a month will run dry in 2 years. The price of crude oil is currently $18 per barrel and is expected to rise at a constant rate of 3 cents per barrel per month. What will be the total revenue from this well? {Hint: Model the degraded production rate with the equation:
    r(t) = (A-Bt)e0.04t}

Harley Weston lui répond.
Differentiation 2001-04-17
Esther pose la question :
Could you please tell me what the first derivative is of the following:

y = 2/(2x+e2x)

Is it (1+xe2x)/(2x+e2x)2 or perhaps -4(1+e2x)/(2x+e2x)2 ? I am a little confused between the two!


Harley Weston lui répond.
Integration by parts 2001-04-09
A student pose la question :
how do you integrate x tan-1x dx, i know it can be done by integration by parts maybe, but i'm not sure....
Claude Tardif and Harley Weston lui répond.
The domain of a function 2001-04-08
Mina pose la question :
Let f(x) = (2x2+3x-17)/(x+5)
What is the domain of f? What are the values of x for which f'(x) does not = 0?

Harley Weston lui répond.
The normal to a curve 2001-04-08
Varenne pose la question :
I am having SO much trouble tackling this question and don't know what the right answer is... can you help me out? The question is

Find the equation of the normal to the curve y=(x-2)2/(1-x)2 that is parallel to the line x+4y+7=0

Harley Weston lui répond.
Common tangents 2001-04-08
Anne pose la question :
I have been working on this problem for a while but I'm not sure I'm getting the right answer: Find the common tangents of 2y=x2 and 2y=-x2-16 Thanks for the help. :)
Harley Weston` lui répond.
Where do the lines y=2x-4 and y=x-1 intesect? 2001-04-06
Bryce pose la question :
solve the following problem by setting them equal to each other. Solve for x and y. Where do the lines y=2x-4 and y=x-1 intesect?
Penny Nom lui répond.
12 RTV's 2001-03-27
Christine pose la question :
  1. In order to promote the 2000 Census and encourage participation, 12 Road Tour Vehicles(RTV's) set out from 12 locations across the US for a 10 week promotion tour last February. The RTV's reportedly traveled a total combined distance of 97,911 miles. What was the aberage number of miles traveled in a day by each RTV?

  2. 2. The 12 RTV's mentioned above traveled a total combined distance equivalent to traveling 12.35 times around the earth at the equator. To the closest mile, what is the radius of the earth? Use 3.14 as your approximation for pi.

Leeanne Boehm lui répond.
Airflow in windpipes 2001-03-25
Ena pose la question :
The volume of air flowing in windpipes is given by V=kpR4, where k is a constant, p is the pressure difference at each end, R is the radius. The radius will decrease with increased pressure, according to the formula: Ro - R = cp, where Ro is the windpipe radius when p=0 & c is a positive constant. R is restricted such that:
0 < 0.5*Ro < R < Ro,
find the factor by which the radius of the windpipe contracts to give maximum flow?

Harley Weston lui répond.
A suspension bridge 2001-03-24
Janna pose la question :
The cables of a suspension bridge hang in a curve which approximates a parabola. The road bed passed through the vertex. If the supporting towers are 720m apart and 60m high, find:

a) an equation of the parabola (it's y = 1/2160x2)

b)the height of the cables at a point 30m from the vertex. I substituted 30 in for the x value and got 0.42 and the answer is 42. What did I do wrong?
Denis Hanson and Claude Tardif lui répond.

The repetend in repeating decimals 2001-03-21
Sharon pose la question :
What is the name for the bar over the repetend in repeating decimals? Also, what is the name of the long division "house"?
Penny Nom lui répond.
Systems of equations 2001-03-16
joy pose la question :
How do u solve problems using systems of equations?
~ finding x and y~

ex:

26 = 3x - 2y
42 = 4x + y


Penny Nom lui répond.
A jogger 2001-03-12
Bill pose la question :
At time t=0 a jogger is running at a velocity of 300 meters per minute. The jogger is slowing down with a negative acceleration that is directly propotional to time t. This brings the jogger to a stop in 10 minutes.

a) write an expression for the velocity of the jogger at time t.

b) what is the total distance traveled by the jogger in that 10-minute interval.


Harley Weston lui répond.
Two locus problems 2001-03-08
Janna pose la question :
A point P moves such that it is always equidistant from the point G(2,5) and the line defined by y=3. Find the equation of the locus. I got as far as the equation: 3y2 -4y = -x2 + 4x - 16 and didn't know what to do from there. Of, course that whole equation could be wrong.

Question 2: P is always twice as far from A(8,0) as it is from B(2,0). Find the equation of the locus. Once again, I got as far as y2 = -x2 -8x -56, and got stuck.


Harley Weston lui répond.
The substitution method 2001-03-05
A student pose la question :
Solve each system of equations by the substitution method. Show your work.
  1. y = 8
    7x = 1 - y

  2. y = x - 1
    4x - y = 19

Penny Nom lui répond.
The domain of the derivative 2001-02-22
Wayne pose la question :
I know that the domain of f'(x) is a subset of the domain of f(x). Is it necessarily true that the subset will have at most one less element than the domain of the original function?
Harley Wesston lui répond.
Differentiation of y = x n 2001-02-17
Jashan pose la question :
i am studying differentation at the moment i have drawn some graphs such as y=x 2. i have found the formula for the gradient of this curve, this being 2x obtained by using differentation, but i need to know the general case for the formula where y=xn in order for me to understand this topic more throughly, i would also like to know how u derived this general formula
Harley Weston lui répond.
A quartic equation 2001-02-15
George pose la question :
Let P(x) = x4 + ax3 + bx2 + cx + d. The graph of y = P(x) is symmetric with respect to the y-axis, has a relative max. at (0,1) and has an absolute min. at (q, -3)

a) determine the values for a, b c, and d using these values, write an equation for P(x)
b) find all possible values for q.


Harley Weston lui répond.
Find an exprression for f(x) 2001-02-07
A 12th grade AP Calc student pose la question :
Let f be the function defined for all x > -5 and having the following properties. Find an expression for f(x).

i) f ''(x) = 1/ (x+5)1/3 for all x in the domain of f

ii) the line tangent to the graph of f at (4,2) has an angle of inclination of 45 degress.


Harley Weston lui répond.
1 + 1 = 1 2001-01-23
Stephanie pose la question :
My friend has this as a bonus question the other day and I want to figure it out. I don't know how 1+1 in any form could equal 1. Please let me know how you come about geting that.
Claude Tardif lui répond.
The hypotenuse of a right triangle 2001-01-22
Phillipe pose la question :
How do you find the hypotenuse of a right triangle?
Penny Nom lui répond.
Polynomials and exponents 2001-01-15
A student pose la question :
I am duing a project in math on polynomials and exponents. I need a real life usage of polynomials and exponents for my project.
Penny Nom lui répond.
Height of the lamp 2000-12-31
Joey pose la question :
The figure shows a lamp located three units to the right of the y-axis and a shadow created by the elliptical region x2 + 4y2 < 5. If the point (-5,0) is on the edge of the shadow, how far above the x-axis is the lamp located?
Harley Weston lui répond.
Bush fractals 2000-12-30
Anita Wisecup pose la question :
My son has a report due on fractals. He needs information on bush fractals, but we cannot seem to find anything out about them.
Penny Nom lui répond.
How do you integrate secant(theta)? 2000-12-22
Robert Williamson pose la question :
How do you integrate secant(theta)?

I know the answer is ln [sec(theta) + tan(theta)] but how do you get there?


Claude tardif lui répond.
A limit using l'hopital's rule 2000-12-13
Wassim pose la question :
I need to know how to solve the:
limits of (x ( to the power lamda) -1 )/LAMDA when lamda tends to zero ( the answer is that the functional form is ln x) and I still don't know how using hopital rule leads to this answer.

Harley Weston lui répond.
Optical illusions 2000-12-06
Jessica pose la question :
Hi, my name is Jessica, 7th grade, and Im doing a Math Fair project on optical illusions. As one of the required factors, we need a "mathmatical significance" paragraph. Unfortunatley, I can only think of one way that optical illusions have to do with math, and thats time because some optical illusions tell you to look at the picture for a certain amount of time.
Penny Nom lui répond.
A non-integrable function 2000-12-03
Mark Spilker pose la question :
I have a proof that I cannot do here it goes.
Let F(x)= 1 if x is a rational number 0 if x is an irrational number Prove the function is not intregrable on the interval (0,1). Hint: Show that no matter how small the norm of the partition, the Riemann Sum for the SigmaNotation F(wi) DeltaiX is not unique.

Harley Weston lui répond.
Comparing an integral and a sum 2000-11-21
Douglas Norberg pose la question :
A fellow teacher asked me about a problem she wanted to give to her students. It involved whether to take a million dollars or a penny doubled a number of times. I was able to determine the number must have been .01 * 230 which is about $10 million and a lot more than $1 million. To check that I was right I used a spreadsheet and did a Riemann sum.

When I finished I reasoned that I had done the task in several steps and I could have done it in 1 step. Thus I integrated .01 * 2x from 0 through 30 but the number I got was $15,490,820.0324. Why the difference?


Harley Weston lui répond.
Bridges and parabolas 2000-11-18
Lauren pose la question :
My name is Lauren, and Im a secondary school student in Ontario. For my gr11 advanced math class I have to find out how and why parabolics are used in arch bridges and write 3 paragraphs on it. People who cohse satelites and whatnot are lucky - I've found a ton of info, but for arch bridges there seems to be nothing.
Harley Weston lui répond.
Inscribing a circle in a rhombus 2000-11-16
Jacky pose la question :
A rhombus ABCD is drawn in which the diagonals are 12 and 20 units long. A circle is inscribed in the quadrilateral with the centre of the circle right on the intersection point of the 2 diagonals. The circumference of the circle touches all 4 sides of the rhombus. Is it possible to find the radius of the inscribed circle? If so, how and what is it?
Chris Fisher lui répond.
Rhombuses 2000-11-07
Melissa pose la question :
What in real life is the shape of a rhombus?
Chris Fisher and Walter Whiteley lui répond.
Concavity 2000-10-22
Alex pose la question :
the question is: on what interval is f(x)=(x2)(ex)?

ive found the 2nd derivative which is ex(x2+4x+2) and i did the quadratic to get -2-20.5 and -2+20.5, but i dont know what the interval is.
Harley Weston lui répond.

A chord length 2000-10-17
Al Paas pose la question :
How to determine the length of a chord given the diameter of the circle and the maximum distance from the chord to The circle?
Chris Fisher lui répond.
Pillows and Cushions 2000-09-27
Fiona pose la question :

The following problem was given to grade eleven algebra students as a homework assignment. To manufacture cushions and pillows, a firm uses two machines A and B. The time required on each machine is shown. Machine A is available for one full shift of 9.6 hours. Machine B is available for parts of two shifts for a total of 10.5 hours each day.
Harley Weston lui répond.

Dividing a region in half 2000-09-21
Kerry pose la question :
There is a line through the origin that divides the region bounded by the parabola y=x-x2 and the x-axis into two regions with equal area. What is the slope of the line?
Penny Nom lui répond.
A cycloid in Cartesian form 2000-09-20
Billy pose la question :
The parametric equation of cycloid is given:
x=r(t-sint)
y=r(1-cost)

How to eliminate t?


Harley Weston lui répond.
A proof that 1=2 2000-09-19
sporky pose la question :
Why does the proof for 1=2 not work?

x = 1
x2 = 1
x = x2
1 = 2x (derivitive)
1 = 2(1)
1 = 2 ???

please tell me where the false logic is.


Walter Whiteley lui répond.
Two linear equations 2000-09-14
David Dean pose la question :
2a + 1b = 3.39 3a + 3b = 6.59
What formula do I use to find what a = ?

Harley Weston lui répond.
1 + 2 + 3 + ... + 50 2000-09-14
Vicki Charron pose la question :
How can you calculate the total of the numbers one through fifty, without adding up the individual numbers?
Penny Nom lui répond.
Derivatives, there must be an easier way 2000-09-06
Brad Goorman pose la question :
The direction read: Take the derivative of each expression.

y = {1+[x+(x2 +x3)4]5}6


Harley Weston lui répond.
Velocity of a pendulum 2000-08-28
Mekca pose la question :
A pendulum hangs from the ceiling. as the pendulum swings, its distance,d cm, form one wall of the room depends on the number of seconds,t, since it was set in motion. assume that the equation for d as a function of t is: d=80+30cos3.14/3t, t>0.

estimate the instantaneous rate of change of d at t=5 by finding the average rates for t=5 to 5.1, t=5 to 5.01, and t=5 to 5.001.


Harley Weston lui répond.
Some trigonometry 2000-08-11
Angela pose la question :
I have some PreCal questions. I am a student at the secondary level. I would be very grateful for your help.

Solve the equation for theta (0 <= theta < 2pi).

tan2(theta) = 3 I know sec2(theta) -1 = tan2(theta)

.
.
.
Harley Weston lui répond.

L'Hospital's Rule 2000-07-19
Dan Krymkowski pose la question :
The limit of the following as x goes to infinity is 2*y. Y is a constant.

lim 2*x*log(x/(x-y)) = 2*y


Harley Weston lui répond.
Divisors of 2000 2000-06-06
Amanda Semi pose la question :
  1. find the product of all the divisors of 2000
  2. dog trainer time has 100m of fencing to enclose a rectangular exercise yard. One side of the yard can include all or part of one side of his building. iff the side of his building is 30 m, determine the maximum area he can enclose

Claude Tardif lui répond.
A derivative problem 2000-06-04
Jeff Ellis pose la question :
If F(x)=(4+x)(3+2x2)2(2+3x3)3, find F'(0)
Harley Weston lui répond.
Calculus Research Questions 2000-05-22
William Wright pose la question :
I am a Calculus Teacher, and me and my class ran into these two problems without solutions in my manual, we got answers, but are unable to check them. If anyone gets this email and can respond to this with the solutions it be greatly appreciated.
.
.
.

Harley Weston lui répond.
Radioactive decay 2000-05-18
Catherine Sullivan pose la question :
Please help me with the following: The radioactive isotope carbon-14 is present in small quantities in all life forms, and it is constantly replenished until the organism dies, after which it decays to carbon-12 at a rate proportional to the amount of C-14 present, with a half life of 5730 years. Suppose C(t) is the amount of C-14 at time t.
  1. Find the value of the constant k in the differential equation: C'=-kC
  2. In 1988 3 teams of scientists found that the Shroud of Turin, which was reputed to be the burial cloth of Jesus, contained 91% of the amount of C-14 contained in freshly made cloth of the same material. How old is the Shroud according to the data?

Harley Weston lui répond.
Related Rates 2000-05-07
Derek pose la question :
How can you show that if the volume of a balloon is decreasing at a rate proportional to its surface area, the radius of the balloon is shrinking at a constant rate.
Harley Weston lui répond.
An improper integral 2000-05-04
A high school senior pose la question :
Hi, I am a high school senior and I need help stugying for a final. I am stuck on one of the questions on my review sheet. Does the improper integral from 5 to infinity of (38/97)x converge or diverge? If it converges I also need to know how to find the approximate value accurate to .01 of its actual value.
Harley Weston lui répond.
Thearcius Functionius 2000-05-03
Kevin Palmer pose la question :
With the Olympics fast approaching the networks are focusing in ona new and exciting runner from Greece. Thearcius Functionius has astounded the world with his speed. He has already established new world records in the 100 meter dash and looks to improve on those times at the 2000 Summer Olympics.

Thearcius Functionius stands a full 2 meters tall and the networks plan on placing a camera on the ground at some location after the finish line(in his lane) to film the history making run. The camera is set to film him from his knees(0.5 meters up from the ground) to 0.5 meters above his head at the instant he finishes the race. This is a total distance of two meters(the distance shown by the camera's lens).
Harley Weston lui répond.

An indefinite integral 2000-05-03
Bonnie Null pose la question :
I am to find the indefinite integral of: (ex - e-x)2 dx
Claude Tardif lui répond.
Minimizing the metal in a can 2000-05-02
May Thin Zar Han pose la question :
A can is to be made to hold 1 L of oil. Find the dimensions that will minimize the cost of the metal to manufacture the can.
Harley Weston lui répond.
Two calculus problems 2000-05-01
Kaushal Shah pose la question :
How Do WE Integrate the following Functions,
  1. Integral xtanx dx
  2. How was natural base "e" discovered and why e=2.7.......

Claude Tardif lui répond.
The area of a triangle using calculus 2000-04-15
Todd Bowie pose la question :
Hi, I am not a student but am reviewing calculus for an upcoming interview. I would like to know how to derive the area of a triangle using calculus. Thanks!
Patrick Maidorn lui répond.
y = x^x^x^x... 2000-04-05
Michael Hackman pose la question :
Find the derivative of: y = x^x^x^x... on to infinity.
Claude Tardif lui répond.
Riemann sums 2000-03-30
Joshua D. Parham pose la question :
If n is a positive integer, then
 
     lim   (1/n)[1/(1+1/n) + 1/(1+(2/n) + ... + 1/(1+n/n)] 
n->infinity 

can be expressed as the integral from 1 to 2 of 1/x dx
Penny Nom lui répond.

Compounding continuously 2000-03-21
Gina pose la question :
You deposit $1500 in an account that pays 6.5% annual interest, compounded continuously. Find the balance after 10 years.

I'm not sure what to do with the "compounded continuously" part.


Penny Nom lui répond.
Functions that satisfy f' = f 2000-03-16
Kevin Palmer pose la question :
Recently my calculus teacher asked his students to try and find any functions whose derivatives where the exact same as the original function.

The only function then I have determined that statement to be accurate in is all the natural exponential functions. Ex. f(x) = ex, f'(x) = ex

If possible could you please email me all the functions that you can find in which the original function and its derivative is identical.


Claude Tardif lui répond.
Maximize 2000-03-12
Tara Doucet pose la question :
My question is Maximize Q=xy^2 (y is to the exponent 2) where x and y are positive integers such that x + y^2 ( y is to the exponent 2)=4
Harley Weston lui répond.
Simultaneous equations 2000-03-11
Laura Molck pose la question :
My name is Laura Molck and I am in Year 11 in Australia. Please help me with the following. I know that they are all simultaneous equations which I can do but I have trouble with the formulae to work the equations. Can you please help!!

1. A tent manufacturer produces 2 models, Outback and Bushwalker. From earlier sales records it is known that 20% more of the Outback model is sold than the Bushwalker. A profit of $200 is made on each Outback sold, but $350 is made on each Bushwalker. If during the next year a profit of $177,000 is planned how many of each model must be sold?


Penny Nom lui répond.
Systems of linear equations 2000-03-10
Ann Marie Devereux pose la question :
hi there!!, I guess I have a problem!!!
  • 3x+4y=10 (over)
    4x+y=9

  • 2x=5y+3 (over)
    x=3y+1


Penny Nom lui répond.
A mixture problem 2000-03-06
Rebecca Edwards pose la question :
A tank in which cholocate milk is being mixed contains a mixture of 460 liters of milk and 40 liters of chocolate syrup initially. Syrup and milk are then added to the tank at the rate of 2 liters per minute of syrup and 8 liters of milk per minute. Simultaneously the mixture is withdrawn at the rate of 10 liters per minute. Find the function giving the amount of syrup in the tank at time t.
Harley Weston lui répond.
Two calculus problems 2000-03-03
Tara Doucet pose la question :
  1. The height of a cylinder with a radius of 4 cm is increasing at rate of 2 cm per minute. Find the rate of change of the volume of the cylinder with respect to time when the height is 10 cm.

  2. A 24 cm piece of string is cut in two pieces. One piece is used to form a circle and the other to form a square. How should the string be cut so the sum of the areas is a maximum?


Harley Weston lui répond.
Slant height of a cone 2000-02-24
Jocelyn Wozney pose la question :
I need help with this problem for my high school calculus class. Any help you can give me will be greatly appreciated-I am pretty stumped. "Express the volume of a cone in terms of the slant height 'e' and the semi-vertical angle 'x' and find the value of 'x' for which the volume is a maximum if 'e' is constant.
Harley Weston lui répond.
Some integration problems 2000-02-23
Tim Valentine pose la question :
I am having a great deal of difficulty with the following integrals, can you help? I think they need the use of trig substitution or integration by parts but I cannot figure out how to begin. Thanks!

The integral of 1/(2+3x2) dx.

and

The integral of x * square root of (4x+5) dx.


Harley Weston lui répond.
A moving point on the graph of y=sinx 2000-02-22
Veronica Patterson pose la question :
Find the rate of change of the distance between the origin and a moving point on the graph of y=sinx if dx/dt=2 centimeters per second.
Harley Weston lui répond.
The quotient rule 2000-02-21
Charlene Anderson pose la question :
Question: I came across a question in our book that states: Let Q(x) = N(x) / D(x) Then re-write Q(x) in a form that can utilize the Power and Product Rules. Use this rearranged form to derive the Quotient Rule.

The Quotient Rule can be derived from the Power Rule and the Product Rule.

One must also use the chain rule too, right?


Harley Weston lui répond.
Filbert Family Circus 2000-02-04
Sarah pose la question :
As Clyde moves his broom around the circus ring, he thinks that he has finally found a job where he can make a clean sweep of things. Clyde is sweeping the ring where the lions perform in the Filbert Family Circus. The ring is 76 feet across and Clyde is using a broom 3 feet wide. He starts at the outside edge and works his way to the middle, making circles around the ring. After sweeping 3/4 of the ring, Clyde sees the lions coming with their trainer and scurries out of the ring. How many trips around the ring did he make?
Penny Nom lui répond.
Play ball 2000-02-03
Jessie pose la question :
Here's a calc question that is probably a lot easier than I am making it. If you have a legendary "baseball problem" for the related rates section of Calc I, and you are given that the runner is running from 2nd to 3rd base at a given rate, and the umpire is standing at home plate, and you are given the distance between the bases on the field, how do you find the rate of change of the angle between the third base line (from the point of the umpire) and the runner? Here is a sample prob: Runner is moving from 2nd to 3rd base at a rate of 24 feet per second. Distance between the bases is 90 feet. What is the rate of change for the angle (theta, as described previously) when the runner is 30 feet from 3rd base?
Harley Weston lui répond.
A problem with a radius. 2000-02-01
Howard B Davis pose la question :
We start a Line that goes up 1 unit, then it goes to the Right for 5 units long, and then goes down 1 unit which is the end point. If we draw a circle that is tangent to both ends as well as the mid-point of the horizontal line: How do we find the radius of the arc; in Mathematics, with only this information?
Chris Fisher lui répond.
Functions 2000-01-23
Tara pose la question :
Hi my name is Tara, I have two math problems that I need help with in my calculus math class.

  1. If f(x)= x - 2 show that (x+3)f(x)-(x+2)f(x+1)+4=0

  2. Graph this function and use the graph to determine the range y=2x2 - 8x - 3

Harley Weston lui répond.
The limit of f(x)/x 2000-01-22
Laurent Jullien pose la question :
I would appreciate help to prove that a twice continuously differentiable convex function from R+ to R has the property that f(x)/x has a limit when x tends to infinity.
Claude Tardif lui répond.
Why study calculus? 2000-01-05
Trlpal pose la question :
I am a high school senior enrolled in a pre-calculus class. Could you tell me what the benefits of taking calculus are and why it would be important to take the class.
Walter Whiteley and Harley Weston lui répond.
zero 2000-01-01
Jason pose la question :
What civilization first used zero?
Penny Nom lui répond.
A decreasing ellipsoid 1999-12-15
A student instructor pose la question :
The volume of an ellipsoid whose semiaxes are of the lengths a,b,and c is 4/3 *pi*abc. Suppose semiaxes a is changing at a rate of A cm/s , the semiaxes b is changing at B cm/s and the semiaxes c is changing at C cm/s . If the volume of the ellipsoid is decreasing when a=b=c what can you say about A,B,C? Justify.
Harley Weston lui répond.
Two calculus problems 1999-12-13
Alan pose la question :
I have 2 questions that are very new to me, they were included on a quiz and the material was never covered. Our teacher never explained the purpose and detailed explanation of how to solve the problem. Could you help? Thanks.

Question 1:
A ball is falling 30 feet from a light that is 50 feet high. After 1 sec. How fast is the shadow of the ball moving towards the light post. Note that a ball moves according to the formula S=16t^2

Question 2:
How many trapezoids must one use in order for the error to be less than 10^-8 if we want to find the area under the curve Y=1/X from 1 to 2. Find the exact area, Graph the function and use the trap rule for the "N" that you found.


Harley Weston lui répond.

A calculus problem 1999-12-08
JT Wilkins pose la question :
These are the questions:

  1. Show that there exists a unique function that meets the following requirements:

    a) f is differentiable everywhere
    b) f(0)= f'(0)= 0
    c) f(x+y)= f(x)+ f(y), for all real values of x,y

  2. Consider the function F: R-->R (All Reals)

    F(x) = 0, for x irrational & 1/q, x=p/q gcd(p,q)=1 q > 0

    a)determine the values x where f is continuous, respectively discontinuous.
    b)determine the values x when f is differentiable and for each of these values compute f'(x).


Penny Nom lui répond.
Advanced Calculus 1999-12-07
Kay pose la question :
Hi, my name is Kay. Please help---these problems are driving me crazzzzy!!!! Your help would be greatly appreciated!
  1. Let a,b be contained in R, a
  2. .
    .
    .

Claude Tardif lui répond.
Systems of equations 1999-12-06
Roger Hornbaker pose la question :
I am having problem figuring out x and y solutions.
  1. 5x + y = 4
    3x - y = 4

  2. 3x + 2y = 6
    - 3x + y = 0

Penny Nom lui répond.
The chain rule 1999-12-03
Jennifer Stanley pose la question :
This problem is making me dizzy. I would greatly appreciate a little help!

Express the derivative dy/dx in terms of x.

y=u^2(u-u^4)^3 and u=1/x^2
Harley Weston lui répond.

Two calculus problems 1999-12-01
O'Sullivan pose la question :
Question #1 Assume that a snowball melts so that its volume decreases at a rate proportional to its surface area. If it takes three hours for the snowball to decrease to half its original volume, how much longer will it take for the snowball to melt completely? It's under the chain rule section of differentiation if that any help.

I've set up a ratio and tried to find the constant but am stuck.

Question #2 The figure shows a lamp located three units to the right of the y-axis and a shadow created by the elliptical region x^2 + 4y^2 < or= 5. If the point (-5,0) is on the edge of the shadow, how far above the x axis is the lamp located?

The picture shows an x and y axis with only the points -5 and 3 written on the x axis. the lamp is on the upper right quadrant shining down diagonally to the left. There's an ellipse around the origin creating the shadow. It's formula is given as x^2 + 4y^2=5.


Harley Weston lui répond.
Two derivatives 1999-11-16
Gina Renicker pose la question :
The derivative of:

y=e(xlnx) and y=x2arctan(x1/2)
Harley Weston lui répond.

Parabolic mirrors 1999-11-07
Andy White pose la question :
I am working on a project concerning parabolic mirrors. I need to create a mirror to focus sunlight on a focal point, but I don't know how to do it. Is there some equation that tells where a focal point will be in relation to a parabola?

What is a directrix?
Penny Nom lui répond.

Area of a circle and an inequality 1999-10-30
Adam Anderson pose la question :
I have two problems.

The first: prove that the area of a cirlce is pi times radius squared without using calculus.

The second: show that ln(x) < x - 1 for all x > 0.


Harley Weston lui répond.

Clockwise or Counterclockwise? 1999-10-27
Tim pose la question :
A particle moves around the circle x2 + y2 = 1 with an x-velocity component dx/dt = y
  1. Find dy/dt

  2. Does the particle travel clockwise or counterclockwise around the circle? Why?

Harley Weston lui répond.
Derivatives with logs 1999-10-26
Kate pose la question :
What is the derivative of 5 to the 5x-2 at x equals 0.8?
Harley Weston lui répond.
-log(a) 1999-10-22
Brenda Miskimmin pose la question :
I need to know the mercury concentration in mg/L or ng/L for the following:

-log M (Hg) = 8.5

where mw of Hg=200.59

(it's the negative sign in front of log that confuses me).
Harley Weston lui répond.

l'Hospital's Rule 1999-10-18
Yannick Gigandet pose la question :
How can I solve these two limits :
  1. lim when n approches 1 of n[a1/n -1]

  2. lim when x approches 0 of (eax - ebx) / x
Thanks for the answer!

Harley Weston lui répond.
A famous mathematician 1999-10-12
Yvette Perez pose la question :
Another way to write 3/15. Remove 0 add a line, unscramble, you have the name of a famous mathematician.
Claude Tardif lui répond.
Length of a line 1999-10-10
Dagmara Sarudi pose la question :
My question has to do with the length of a diagonal. This problem came up when I thought about the shortest distance between two points, for example walking from one point to another in my neighborhood. I can choose a zig zag route and assuming the blocks I walk are exactly the same length, it shouldn't matter what route I took, the distance I travel should still be the same when I reached my goal. If, on the other hand I could travel in a diagonal line, the distance would be shorter. But what if, in my zig zag motion, the sections get so small the route approaches a diagonal. Shouldn't it be that each separate section added together equals the value of the two original sides? Or would it suddenly equal the value of the diagonal (which, of course was shorter than the two sides added together)?
What gives?


Chris Fisher and Harley Weston lui répond.

A trig limit 1999-10-06
Yannick Gigandet pose la question :
What is the limit, as x approaches pi/3, of (1-2cosx) / sin(x-(pi/3)) ?
Penny Nom lui répond.
The circumference of a circle 1999-10-05
Mara Frost pose la question :
what is the formula to find the circumference of a circle, or if there is no formula, how do you find the circumference of a circle?
Penny Nom lui répond.
Two limits 1999-10-02
Jennifer pose la question :
How do I find

lim (1-cosx)/(x^2) as x-> 0

and

lim (tan3x)/x as x->0
Harley Weston lui répond.

Temperatures 1999-09-27
Eula pose la question :
How do you cahnge farenheit degrees to celsius degrees?
Penny Nom lui répond.
Numbers with the digit 2 in 1...1000 1999-09-20
Jessica pose la question :
Is there a trick to finding out how many numbers containing the digit two is there from 1 to 1000?
Walter Whiteley lui répond.
Distance between the windows 1999-09-19
Lawrence pose la question :
An observer on level ground is at distance d from a building. The angles of elevation to the bottom of the windows on the second and third floors are a and b respectively. Find the distance h between the bottoms of the windows in terms of a b and d
Harley Weston lui répond.
2 to the x and x squared 1999-09-17
John pose la question :
For what values of x is 2 to the exponent x greater than x squared?
Harely Weston lui répond.
Y2K? 1999-09-03
Mike Putzakulish pose la question :
In Y2K, the "K" stands for thousand, but where did the "K" come from? I know it's not a Roman Numeral, but what is it??
Chris Fisher and Harley Weston lui répond.
A double negative 1999-09-01
Dennis pose la question :
If b = -2 what does -b = ? As in (a + 8.5) - [(-b) + |c|] a = 1.5, c = -1.7
Penny Nom lui répond.
Parametric Equations 1999-08-06
Nicholas Lawton pose la question :
Show that an equation of the normal to the curve with parametric equations x=ct y=c/t t not equal to 0, at the point (cp, c/p) is :

y-c/p=xp^2-cp^3
Harley Weston lui répond.

A calculus problem 1999-07-22
Nicholas Lawton pose la question :
The curve y= e^x(px^2+qx+r) is such that the tangents at x=1 and x=3 are parallel to the x-axis. the point (0,9) is on the curve. Find the values of p,q and r.
Harley Weston lui répond.
The shortest ladder 1999-06-26
Nicholas pose la question :
A vertical wall, 2.7m high, runs parallel to the wall of a house and is at a horizontal distance of 6.4m from the house. An extending ladder is placed to rest on the top B of the wall with one end C against the house and the other end, A, resting on horizontal ground. The points A, B, and C are in a vertical plane at right angles to the wall and the ladder makes an angle@, where 0<@
Harley Weston lui répond.
Even and Odd Function 1999-06-17
Kent pose la question :
There is one function with the domain of all real numbers that is both even and odd. Please give me the answer to this question before I go insane.
Penny Nom lui répond.
A circle in a square 1999-05-26
Jose V Peris pose la question :
A circle is inscribed in a square. The circumference of the circle is increasing at a constant rate of 6 inches per second. As the circle expands, the square expands to maintain the condition of tangency.

find the rate at which the perimeter of the square is increasing.

find the rate of increase in the area enclosed between the circle and the square at the instant when the area of the circle is 25(pi) square inches.
Harley Weston lui répond.

Related rates 1999-05-13
Tammy pose la question :
The sides of a rectangle increase in such a way that dz/dt=1 and dx/dt=3*dy/dt. At the instant when x=4 and y=3, what is the value of dx/dt? (there is a picture of a rectangle with sides x and y, and they are connected by z, which cuts the rectangle in half)
Harley Weston lui répond.
A Polar Plot 1999-05-06
Irene pose la question :
Consider the polar equation r=2-3Cos(theta/2) In the interval [o, 4Pi], how would you find the area of one of the leaves and also the length of one of the edges of a leaf?
Harley Weston lui répond.
Radius of an arc 1999-04-22
Rusty Riddleberger pose la question :
I need to find the equation for finding the radius of an arc; I know the length of the arc (i.e the distance of the line connecting the two ends of the arc) and the height; (i.e the rise of the arc at its apex,) I had the formula years ago but it has lost me; this would be invaluable for work in new homes i.e. where we need to build an "arch" with a rise of 21" between two columns 11 feet apart
Chris Fisher lui répond.
Radius of convergence 1999-04-21
Nowl Stave pose la question :
Why is the radius of convergence of the first 6 terms of the power series expansion of x^(1/2) centered at 4 less than 6?
Harley Weston lui répond.
Circles 1999-04-21
Alex Elkins pose la question :
How do you find the circumference of a circle if you only know the radius and the square feet or inches of the circle if the radius is 18 inches, If done in inches do you multiply by 12 to get the square feet?
Jack Lesage and Harley Weston lui répond.
The average rate of change of a function 1999-04-20
Tammy pose la question :
Suppose that the average rate of change of a function f over the interval from x=3 to x=3+h is given by 5e^h-4cos(2h). what is f'(3)? I would appreciate any help with this question.
Harley Weston lui répond.
A Frustum 1999-03-29
Monica Armour pose la question :
What do you call a square pyramid that has had the top chopped off?
Chris Fisher and Jack LeSage lui répond.
Rhomboid 1999-03-25
Monica Armour pose la question :
I need to see a net of a rhomboid. Where can I find one on the net? Is it like a square paramid with the top chopped off? Help! This has me stumped.
Jack LeSage lui répond.
Graphing the Derivative 1999-01-18
Milena Ghebre pose la question :
This question has been nagging me for sometime now.

Is there a way of finding out the derivative of a function, just by looking at the graph of it?
Walter Whiteley lui répond.

Calculus 1999-01-16
Kaylea Rankin pose la question :
Differentiate the following.

y = 1 /(2+3/x)
Jack LeSage and Penny Nom lui répond.

Absolute value of i 1999-01-06
Wayne Bagley pose la question :
I would like to know what is the absolute value of i. I need an answer suitable for the secondary level.
Harley Weston lui répond.
The area and the circumference of a circle. 1998-08-27
Jason Wright pose la question :
I was looking at the relationship of the area of a circle and the circumference when I realized that 2*pi*r is the derivative of pi*r^2. I was wondering if there is any connective deep dark meaning as to why this appears to be related. Thanks for any help you can give me!
Walter Whiteley lui répond.
Parabolas 1998-07-24
Danica pose la question :
how do you find the focus, vertex, and directrix of 4x-y^2-2y-33=0
Penny Nom lui répond.
Volumes of Revolution 1998-07-24
Lorraine Wall pose la question :
I'm on the section fpr The Computation of Volumes of Solids of Revolution and the following question is giving me problems:

-Consider the region in the first quadrant bounded by the x-and y-axes, the vertical line x=3, and the curve y=1/(xsquared + 3) I can determine the volume of the solid by rotating the region about the y-axis using the shell method but I can't seem to be able to get started with the volume when rotated about the x-axis.
Harley Weston lui répond.

Calculus problems 1998-07-13
Lorraine pose la question :
I'm stuck again. Can you help?

This involves integration using the method of partial fractions

the integral of:
7x(to the 5th) - 2x(cubed) + 3 dx
--------------------------------------
x(to the fourth) - 81

Do I have to do long division to reduce the numerator to the fourth power?

the integral of:
4- 16x +21x(squared) + 6x(cubed) - 3x(fourth) dx
----------------------------------------------------
x(cubed)(x - 2)(squared)

Lorraine
Harley Weston lui répond.

A Calculus Problem 1998-06-28
Lorraine pose la question :
I'm a post-secondary student taking calculus by correspondence. I'm stuck on the following question (and similar ones) Can you help?

Evaluate the following indefinite integral:

d(theta)
----------
1 + sin (theta)

(It says to multiply both numerator and denominator by: 1 - sin(theta)

Thanks
Lorraine
Harley Weston lui répond.

A Logic Problem 1998-06-07
Anthony Bacigalupo pose la question :
My name is Anthony Bacigalupo and I take Sequential ][ Math and am taking a practice regents. When doing a logic problem, I encountered the following statements, where I am trying to prove P ( I left out steps unrelated to the question)....
Chris Fisher lui répond.
A trig limit 1998-05-28
Ann pose la question :
This problem is a calculus 1 limit problem-high school level. I'm teaching myself calc over the summer and I'm already stumped.

find the limit

 lim sec^(2)[(sqrt2)(p)]-1 p-->0 --------------------- 1-sec^(2)[(sqrt3)(p)] 
I'm Ann.

Harley Weston lui répond.
A Tightrope Walker. 1998-02-19
Amy Zitron pose la question :
A tightrope is stretched 30 feet above the ground between the Jay and the Tee buildings, which are 50 feet apart. A tightrope walker, walking at a constant rate of 2 feet per second from point A to point B, is illuminated by a spotlight 70 feet above point A....
Harley Weston lui répond.
Pi 1997-10-31
Ryan McKinnon pose la question :
What Is Pi?
Chris Fisher lui répond.
Some Calculus Problems. 1997-10-30
Roger Hung pose la question :
  1. What real number exceeds its square by the greatest possible amount?

  2. The sum of two numbers is k. show that the sum of their squares is at least 1/2 k^2.

  3. .
    .
    .

Penny Nom lui répond.
A Trigonometric Limit 1997-09-18
Brian Ray pose la question :
What is the limit, as x approaches 0, or tan^23x/x^2? (read, tan squared 3x over...)?
Harley Weston lui répond.
A Limit Problem 1997-09-16
Robert Reny pose la question :
what is the limit, as x approaches 0, of 3x/2x-[x]? [] means absolute value.
Harley Weston lui répond.
The Division Bracket. 1997-04-09
Judy Riley pose la question :
A fellow teacher recently asked if I remembered the exact word for a division bracket (not the symbol with dots, the horizontal line in a fraction, or a solidus). I couldn't. Can you help?
Walter Whiteley and Harley Weston lui répond.
Mathematical Induction and the Derivative 1997-03-18
Shuling Chong pose la question :
"Obtain a formula for the nth derivative of the product of two functions, and prove the formula by induction on n."

Any educated tries are appreciated.
Penny Nom lui répond.

Parabolic Mirrors 1997-01-28
Megan Wennberg pose la question :
Consider a ray of light that passes through a chord of a parabola (the chord is above the focus and parallel to the directrix), hits the parabola at a point (x,y) and is reflected through the focus. If d1 is the distance from the chord to the point of incidence (x,y) and d2 is the distance from (x,y) to the focus, can you prove that the sum of the distances d1+d2 is constant, independent of the particular point of incidence.
Penny Nom lui répond.
Foci of an Ellipse 1997-01-22
David Gilliam pose la question :
How do I find the focii of the following equation? 4x^2 + 9y^2 = 36
Harley Weston lui répond.
A tangent to a circle is perpendicular to the radius at the point of contact. 1996-10-22
Rita Leung pose la question :
I wonder if there is any proof for this theorem - A tangent to a circle is perpendicular to the radius at the point of contact. If there is any proof for that, can you tell me please?
Chris Fisher and Harley Weston lui répond.
Could you tell me the name for the bar in a division problem? 1996-10-21
Linda pose la question :
Could you tell me the name for the bar in a division problem. Not the line with dots on either side but the line that divides the two numbers? My name is Linda. I am asking for my niece who is in 8th grade.
Chris Fisher lui répond.
Thousands, millions, billions and trillions 1996-08-22
Blaine pose la question :
What are the names of the periods in groups of numbers like 123,456,789. I need to know the names of them for school. I already know the first ones: units thousands millions billions trillions.
Penny Nom and Diane Hanson lui répond.
What are fractals and are they of any practical use? 1996-06-26
Ron pose la question :
What are fractals and are they of any practical use?
Chris Fisher lui répond.
Show that this construction yields a rhombus. 1996-06-16
LennyB pose la question :
Hello, I have a problem. I doubt you can help me. If you have an isosceles trapezoid, and you connect the midpoints of the four sides of the isosceles trapezoid forming a quadrilateral, how do you prove that it's a rhombus in a 2 column proof??
Penny Nom lui répond.
Area of an annulus 1996-04-04
Ryan Levering pose la question :
What is the area of this annulus?
Penny Nom lui répond.
(-3)x(-2) 1995-10-25
Azmat pose la question :
Why is (-3)x(-2) = 6?
Herley Weston and Ed Giesbrecht lui répond.
 
Page
1/1

 

 


Centrale des maths reçoit une aide financière de l’Université de Regina et de The Pacific Institute for the Mathematical Sciences.

CMS
.

 

accueil centre de ressources accueil Société mathématique du Canada l'Université de Regina PIMS